Exam 2 review questions ( chapters 11,13,14,49,27,38,39,43,44,48,10,33,34,35,36,37,48,15,16,17,18,19,20,21,26

अब Quizwiz के साथ अपने होमवर्क और परीक्षाओं को एस करें!

Which individual is most likely to need the nurse's assistance in coping with identity confusion? a. A 49-year-old male with stable employment b. A 35-year-old recently divorced mother of twins c. A 22 year old in the third year of college d. A 50-year-old self-employed woman

A 35-year-old recently divorced mother of twins rational: Identity confusion results when people do not maintain a clear, consistent, and continuous consciousness of personal identity. A newly divorced woman would be trying to adapt to a new lifestyle of being single while handling parenting of twins as a single parent. This situation could lead to identity confusion. A college sophomore would have had at least 2 years to adjust to the new life setting, and a self-employed woman would likely be content with creating her own employment opportunity. There is no indication that the middle-aged man with stable employment should have identity confusion.

A nurse is preparing to assess a patient for orthostatic hypotension. Which piece of equipment will the nurse obtain to assess for this condition? a. Thermometer b. Elastic stockings c. Blood pressure cuӄ d. Sequential compression devices

Blood pressure cuff A blood pressure cuӄ is needed. Orthostatic hypotension is a drop of blood pressure greater than 20 mm Hg in systolic pressure or 10 mm Hg in diastolic pressure and symptoms of dizziness, light-headedness, nausea, 10/15/2016 Chapter 28: Immobility | Nursing Test Banks http://boostgrade.info/chapter28immobility/ 5/34 tachycardia, pallor, or fainting when the patient changes from the supine to standing position. A thermometer is used to assess for fever. Elastic stockings and sequential compression devices are used to prevent thrombus.

A nurse is assessing young and middle-aged adults for work-related conditions. Match the job to the work-related conditions that the nurse is assessing. Dye workers

Bladder cancer

A parent asks about the human papillomavirus (HPV) vaccine. Which information will the nurse include in the teaching session? a. It is recommended for girls 6 to 9 years old. b. It is recommended for females ages 11 to 26. c. It is recommended that booster injections be given. d. It is recommended to receive four required injections.

It is recommended for females ages 11 to 26. rational: The vaccine is safe for girls as young as 9 years old and is recommended for females ages 11 to 26 if they have not already completed the three required injections. Booster doses currently are not recommended. The vaccine is most effective if administered before sexual activity or exposure.

The nurse is caring for a patient after an open abdominal aortic aneurysm repair. The nurse requests an abdominal binder and carefully applies the binder. Which is the best explanation for the nurse to use when teaching the patient the reason for the binder? a. It reduces edema at the surgical site. b. It secures the dressing in place. c. It immobilizes the abdomen. d. It supports the abdomen.

It supports the abdomen rationale: The patient has a large abdominal incision. This incision will need support, and an abdominal binder will support this wound, especially during movement, as well as during deep breathing and coughing. A binder can be used to immobilize a body part (e.g., an elastic bandage applied around a sprained ankle). A binder can be used to prevent edema, for example, in an extremity but in this case is not used to reduce edema at a surgical site. A binder can be used to secure dressings such as elastic webbing applied around a leg after vein stripping.

A patient is admitted with a stroke. The outcome of this disorder is uncertain, but the patient is unable to move the right arm and leg. The nurse starts passive range-of-motion (ROM) exercises. Which finding indicates successful goal achievement? a. Heart rate decreased. b. Contractures developed. c. Muscle strength improved. d. Joint mobility maintained.

Joint mobility maintained rationale: When patients cannot participate in active ROM, maintain joint mobility and prevent contractures by implementing passive ROM into the plan of care. Exercise and active ROM can improve muscle strength. ROM is not performed for the heart but for the joints.

The nurse is caring for a patient who is recovering from a traumatic brain injury and frequently becomes disoriented to everything except location. Which nursing intervention will the nurse add to the care plan to reduce confusion? a. Keep a day-by-day calendar at the patient's bedside. b. Place a patient observer in the patient's room for safety. c. Assess the patient's level of consciousness and document every 4 hours. d. Prepare to discharge once the patient is awake, alert, and oriented.

Keep a day-by-day calendar at the patient's bedside. rational: Keeping a calendar in the patient's room helps to orient the patient to the dates. In the home meaningful stimuli include pets, music, television, pictures of family members, and a calendar and clock. The same stimuli need to be present in health care settings. Assessing the patient's level of consciousness is not an action that will directly affect the patient's confusion. A patient observer is unnecessary unless the patient is in danger from the confusion. The nurse should encourage the patient toward recovery but should be sensitive to the time it takes for progression.

An older patient with dementia and confusion is admitted to the nursing unit after hip replacement surgery. Which action will the nurse include in the plan of care? a. Keep a routine. b. Continue to reorient. c. Allow several choices. d. Socially isolate patient.

Keep a routine. rationale: Patients with dementia need a routine. Continuing to reorient a patient with dementia is nonproductive and not advised. Patients with dementia need limited choices. Social interaction based on the patient's abilities is to be promoted.

The patient is confused, is trying to get out of bed, and is pulling at the intravenous infusion tubing. These data would help to support a nursing diagnosis of a. Risk for poisoning. b. Knowledge deficit. c. Impaired home maintenance. d. Risk for injury.

Risk for injury rationale: The patient's behaviors support the nursing diagnosis of risk for injury. The patient is confused, is pulling at the intravenous line, and is trying to climb out of bed. Injury could result if the patient falls out of bed or begins to bleed from a pulled line. Nothing in the scenario indicates that this patient lacks knowledge or is at risk for poisoning. Nothing in the scenario refers to the patient's home maintenance.

The nurse is collaborating with the dietitian about a patient with a Stage III pressure ulcer. Which nutrient will the nurse most likely increase after collaboration with the dietitian? a. Fat b. Protein c. Vitamin E d. Carbohydrate

protein rationale: Protein needs are especially increased in supporting the activity of wound healing. The physiological processes of wound healing depend on the availability of protein, vitamins (especially A and C), and the trace minerals of zinc and copper. Wound healing does not require increased amounts of fats or carbohydrates. Vitamin E will not be increased for wound healing.

The nurse is providing teaching to an immobilized patient with impaired skin integrity about diet. Which diet will the nurse recommend? a. High protein, high calorie b. High carbohydrate, low fat c. High vitamin A, high vitamin E d. Fluid restricted, bland

High protein, high calorie rationale: Because the body needs protein to repair injured tissue and rebuild depleted protein stores, give the immobilized patient a high-protein, high-calorie diet. A high-carbohydrate, low-fat diet is not beneӄcial for an immobilized patient. Vitamins B and C are needed rather than A and E. Fluid restriction can be detrimental to the immobilized patient; this can lead to dehydration. A bland diet is not necessary for immobilized patients.

The nurse is teaching a patient how to use a condom. Which instructions will the nurse provide? a. Store in a warm lit space. b. Use massage oils for lubrication. c. Rinse and reuse the condom if needed. d. Hold onto the condom when pulling out.

Hold onto the condom when pulling out. rational: Teach patients to pull out right after ejaculating and to hold onto the condom when pulling out. Store condoms in a cool, dry place away from sunlight. Instruct patient to never reuse a condom or use a damaged condom. Instruct patient to only use water-based lubricants (e.g., K-Y jelly) to prevent the condom from breaking; do not use petroleum jelly, massage oils, body lotions, or cooking oil.

A nurse works on a pediatric unit and is using a psychosocial developmental approach to child care. In which order from the first to the last will the nurse place the developmental stages? 1. Initiative versus guilt 2. Trust versus mistrust 3. Industry versus inferiority 4. Identity versus role confusion 5. Autonomy versus shame and doubt a. 2, 5, 3, 1, 4 b. 2, 1, 3, 5, 4 c. 2, 3, 1, 5, 4 d. 2, 5, 1, 3, 4

2, 5, 1, 3, 4 rationale: Erikson uses a psychosocial approach to development. The stages are as follows: trust versus mistrust; autonomy versus shame and doubt; initiative versus guilt; industry versus inferiority; and identity versus role confusion.

The nurse is completing a skin risk assessment using the Braden Scale. The patient has slight sensory impairment, has skin that is rarely moist, walks occasionally, and has slightly limited mobility, along with excellent intake of meals and no apparent problem with friction and shear. Which score will the nurse document for this patient? a. 15 b. 17 c. 20 d. 23

20 rationale: With use of the Braden Scale, the total score is a 20. The patient receives 3 for slight sensory perception impairment, 4 for skin being rarely moist, 3 for walks occasionally, 3 for slightly limited mobility, 4 for intake of meals, and 4 for no problem with friction and shear.

A nurse is caring for a patient who is dealing with the developmental task known as initiative versus guilt. The nurse is providing care to which patient? a. A 3-week-old neonate b. A 5-year-old kindergarten student c. An 11-year-old student d. A 15-year-old high school student

A 5-year-old kindergarten student rational: The initiative versus guilt developmental stage occurs between the ages of 3 and 6 years. The patient is a 5-year-old kindergarten student. If a child shows initiative, the outcome of this developmental task is to develop purpose. A neonate developmental task is to develop trust. An 11-year-old is into new skill mastery (industry), and a 15-year-old is struggling with identity versus role confusion.

A home health nurse is providing care to a middle-aged couple with children at home. The patient has a debilitating chronic illness. Which areas will the nurse need to assess? (Select all that apply.) a. Adherence to treatment and rehabilitation regimens b. Coping mechanisms of patient and family c. Need for community services or referrals d. Knowledge base of patient only e. Use of a doula for care

Adherence to treatment and rehabilitation regimens Coping mechanisms of patient and family Need for community services or referrals rationale: Along with the current health status of the chronically ill middle-aged adult, you need to assess the knowledge base of both the patient and family. In addition, you must determine the coping mechanisms of the patient and family; adherence to treatment and rehabilitation regimens; and the need for community and social services, along with appropriate referrals. A doula is a support person to be present during labor to assist women who have no other source of support.

A formerly independent older adult becomes severely withdrawn upon admission to a nursing home. Which action should the nurse take first? a. Offer a reward to the patient for participation in all events. b. Encourage the patient to eat meals in the dining room with others. c. Allow the patient to incorporate personal belongings into the room. d. Advise the patient of the importance of attending mandatory activities.

Allow the patient to incorporate personal belongings into the room rationale: The nurse should first allow the patient to actively participate in an independent activity (the patient was formerly independent), such as preparing the room with personal belongings. Erikson's theory proposes that the older adult faces integrity versus despair. Offering a reward does not address the need for continued independence. Encouraging eating in the dining room would be a step after fixing the room since it does not address independence, and the question is asking for the first action. Advising the patient to attend all mandatory activities as the first intervention does not allow for the patient's independence. Some activities may be mandatory, but by first allowing the patient to decorate room, the nurse is fostering independence and is helping the patient feel welcome and more at home, fostering integrity.

The patient is in the intensive care unit (ICU), which has strict posted visiting hours and limits the number of visitors to two per patient at any one time. The patient is asking to see his wife and two daughters. The nurse should a. Tell the patient that they will be allowed to visit at the appropriate time. b. Allow the wife and one daughter to enter the ICU, but not the other daughter. c. Allow the two daughters to visit, and let the wife visit when they leave. d. Allow the wife and daughters to visit at the patient's request.

Allow the wife and daughters to visit at the patient's request. rationale: Use of support systems is important in any health care setting. When patients depend on family and friends for support, encourage them to visit the patient regularly. As long as no interference with active patient care is involved, there is no reason to limit visitation.

A nurse is assessing young and middle-aged adults for work-related conditions. Match the job to the work-related conditions that the nurse is assessing. Insulators

Asbestosis

A nurse is in the hallway assisting a patient to ambulate and hears an alarm sound. What is the best next step for the nurse to take? a. Seek out the source of the alarm. b. Wait to see if the alarm discontinues. c. Ask another nurse to check on the alarm. d. Continue ambulating the patient.

Ask another nurse to check on the alarm rationale: The nurse who heard the alarm has a duty to address it even though she is busy with another patient. Ask someone to check on the alarm. The nurse cannot leave the patient in the hallway to look for the source of the alarm and cause a potentially unsafe situation for this patient, but a patient on the unit may have an urgent need. Someone needs to seek out the source of the alarm and address it. Never ignore an alarm. Alarms are in place to maximize the safety of the patient. Waiting to see if an alarm stops may cause a delay in a possible emergency situation.

A nurse is teaching a group of older-adult patients. Which teaching strategy is best for the nurse to use? a. Provide several topics of discussion at once to promote independence and making choices. b. Avoid uncomfortable silences after questions by helping patients complete their statements. c. Ask patients to recall past experiences that correspond with their interests. d. Speak in a high pitch to help patients hear better.

Ask patients to recall past experiences that correspond with their interests. rationale: Teaching strategies include the use of past experiences to connect new learning with previous knowledge, focusing on a single topic to help the patient concentrate, giving the patient enough time in which to respond because older adults' reaction times are longer than those of younger persons, and keeping the tone of voice low; older adults are able to hear low sounds better than high-frequency sounds.

The nurse is caring for a patient who suddenly experiences chest pain. What is the nurse's first priority? a. Call the rapid response team. b. Start an intravenous (IV) line. c. Administer pain-relief medications. d. Ask the patient to rate and describe the pain.

Ask the patient to rate and describe the pain. rational: The nurse's ability to establish a nursing diagnosis, plan and implement care, and evaluate the effectiveness of care depends on an accurate and timely assessment. The other responses are all interventions; the nurse cannot know which intervention is appropriate until the nurse completes the assessment.

The nurse identifies that a patient has received Mylanta (simethicone) instead of the prescribed Pepto-Bismol (bismuth subsalicylate) for the problem of indigestion. The nurse's next intervention is to a. Do nothing, no harm has occurred. b. Assess and monitor the patient. c. Notify the physician, treat and document. d. Complete an incident report.

Assess and monitor the patient. rationale: After providing an incorrect medication, assessing and monitoring the patient to determine the effects of the medication is the first step. Notifying the physician and providing treatment would be the best next step. After the patient has stabilized, completing an incident report would be the last step in the process.

The nurse is caring for a patient in restraints. Which of the following pieces of information about restraints requires nursing documentation in the medical record? (Select all that apply.) a. The patient states that her gown is soiled and needs changing. b. Attempts to distract the patient with television are unsuccessful. c. The patient has been placed in bilateral wrist restraints at 0815. d. One family member has gone to lunch. e. Bilateral radial pulses present, 2+, hands warm to touch f. Released from restraints, active range-ofmotion exercises complete

Attempts to distract the patient with television are unsuccessful. The patient has been placed in bilateral wrist restraints at 0815. Bilateral radial pulses present, 2+, hands warm to touch Released from restraints, active range-of motion exercises complete rationale: Attempts at alternatives are documented in the medical record, as are type of restraint and time restrained. Assessments related to oxygenation, orientation, skin integrity, circulation, and position are documented, along with release from restraints and patient response. Comments about hygiene or the activities of one family member are not necessarily required in nursing documentation of restraints.

Transparent dressing

Barrier to external fluids/bacteria but allows wound to "breathe"

An older-adult patient is visiting the clinic after a fall during the night. The nurse obtains information on what medications the patient takes. Which medication most likely contributed to the patient's fall? a. Melatonin b. L-tryptophan c. Benzodiazepine d. Iron supplement

Benzodiazepine rationale: The most likely cause is a benzodiazepine. If older patients who were recently continent, ambulatory, and alert become incontinent or confused and/or demonstrate impaired mobility, the use of benzodiazepines needs to be considered as a possible cause. This can contribute to a fall in an older adult. Short-term use of melatonin has been found to be safe, with mild side effects of nausea, headache, and dizziness being infrequent. Iron supplements may be given to patients with restless legs syndrome. Some substances such as L- tryptophan, a natural protein found in foods such as milk, cheese, and meats, promote sleep; while it does promote sleep, it is not the most likely to cause mobility problems

A nurse is measuring an infant's head circumference and height. Which area is the nurse assessing? a. Moral development b. Cognitive development c. Biophysical development d. Psychosocial development

Biophysical development rationale: Biophysical development is how our physical bodies grow and change. Moral development is the difference between right and wrong. Cognitive development comprises changes in intelligence, use of language, and development of thinking. Psychosocial development consists of variations in emotions and relationships with others.

A nurse is assessing young and middle-aged adults for work-related conditions. Match the job to the work-related conditions that the nurse is assessing. Office computer workers

Carpal tunnel syndrome

Upon assessment a nurse discovers that a patient has erythema. Which actions will the nurse take? (Select all that apply.) a. Consult a dietitian. b. Increase fiber in the diet. c. Place on chest physiotherapy. d. Increase frequency of turning. e. Place on pressure-relieving mattress.

Consult a dietitian. Increase frequency of turning. Place on pressure-relieving mattress rationale: If skin shows areas of erythema and breakdown, increase the frequency of turning and repositioning; place the turning schedule above the patient's bed; implement other activities per agency skin care policy or protocol (e.g., assess more frequently, consult dietitian, place patient on pressure-relieving mattress). Increased ӄber will help constipation. Chest physiotherapy is for respiratory complications.

Location

Could you point to the area of pain?

Severity

Could you rate your pain on a scale of 0 to 10?

Lordosis

Exaggeration of anterior convex curve of lumbar spine

Timing

How often does it recur?

Role performance

I am a good mother

Low self-esteem

I am good for nothing.

A nurse is assessing a patient's self-concept. Which areas will the nurse include? (Select all that apply.) a. Identity b. Body image c. Coping behaviors d. Significant others' support e. Availability of insurance

Identity Body image Coping behaviors Significant others' support rational: Assessment of self-concept includes identity, body image, coping behaviors, and significant others' support. Availability of insurance is not a component of self-concept.

A nurse is assessing pressure points in a patient placed in the Sims' position. Which areas will the nurse observe? a. Chin, elbow, hips b. Ileum, clavicle, knees c. Shoulder, anterior iliac spine, ankles d. Occipital region of the head, coccyx, heels

Ileum, clavicle, knees rationale: In the Sims' position pressure points include the ileum, humerus, clavicle, knees, and ankles. The lateral position pressure points include the ear, shoulder, anterior iliac spine, and ankles. The prone position pressure points include the chin, elbows, female breasts, hips, knees, and toes. Supine position pressure points include the occipital region of the head, vertebrae, coccyx, elbows, and heels.

A patient has damage to the cerebellum. Which disorder is most important for the nurse to assess? a. Imbalance b. Hemiplegia c. Muscle sprain d. Lower extremity paralysis

Imbalance rationale: Damage to the cerebellum causes problems with balance, and motor impairment is directly related to the amount of destruction of the motor strip. A stroke can lead to hemiplegia. Direct trauma to the musculoskeletal system results in bruises, contusions, sprains, and fractures. A complete transection of the spinal cord can lead to lower extremity paralysis

A nurse is caring for an older adult. Which sensory change will the nurse identify as normal during the assessment? a. Impaired night vision b. Difficulty hearing low pitch c. Heightened sense of smell d. Increased taste discrimination

Impaired night vision rational: Night vision becomes impaired as physiological changes in the aging eye occur. Older adults lose the ability to distinguish high-pitched noises and consonants. Senses of smell and taste are also decreased with aging.

Torticollis

Inclining of head to aӄected side

Kyphosis

Increased convexity in curvature of thoracic spine

Scoliosis

Lateral-S- or C-shaped spinal column with vertebral rotation

A nurse is evaluating a patient's self-concept. Which key indicator will the nurse use? a. Drug abuse history b. Nonverbal behavior c. Personal journal entries d. Posts on social media

Nonverbal behavior rational: Key indicators of a patient's self-concept are nonverbal behaviors. A history of drug abuse does not necessarily indicate current self-concept, and people who do not have a drug abuse history may have a low self-concept. It would be an invasion of privacy and trust for a nurse to read a patient's personal journal or social media posts.

The mother of a child who died recently keeps the child's room intact. Family members are encouraging her to redecorate and move forward in life. Which type of grief will the home health nurse recognize the mother is experiencing? a. Normal b. End-of-life c. Abnormal d. Complicated

Normal rationale: Family members will grieve differently. One sign of normal grief is keeping the deceased individual's room intact as a way to keep that person alive in the minds of survivors. This is happening after the family member is deceased, so it is not end-of-life grief. It is not abnormal or complicated grief; the child died recently.

The nurse adds a nursing diagnosis of ineffective breathing pattern to a patient's care plan. Which sleep condition caused the nurse to assign this nursing diagnosis? a. Insomnia b. Narcolepsy c. Sleep deprivation d. Obstructive sleep apnea

Obstructive sleep apnea rationale: Obstructive sleep apnea (OSA) occurs when the muscles or structures of the oral cavity or throat relax during sleep. The upper airway becomes partially or completely blocked, diminishing airflow or stopping it for as long as 30 seconds. The person still attempts to breathe because chest and abdominal movements continue, resulting in snoring or snorting sounds. With narcolepsy, the person feels an overwhelming wave of sleepiness and falls asleep. Insomnia is characterized by chronic difficulty falling asleep. Sleep deprivation is a condition caused by dyssomnia. OSA is the only one of these conditions that results in blockage of the airway and impacts the ability to breathe.

Hiatal hernia

Offer a small meal several hours before bedtime

gauze

Oldest and most common absorbent dressing

Genu Varum

One or both legs bent outward at knee

The nurse and the patient have the same religious affiliation. Because of this, the nurse a. Can assume that they have the same spiritual beliefs. b. Should not impose her personal values on the patient. c. Must use an assessment tool to assess the patient's beliefs. d. Can skip the spiritual belief assessment.

Should not impose her personal values on the patient. rationale: The nurse can use an assessment tool or direct an assessment with questions based on principles of spirituality, but it is important not to impose personal value systems on the patient. This is particularly true when the patient's values and beliefs are similar to those of the nurse because it then becomes very easy to make false assumptions. It is important to conduct the spiritual belief assessment; conducting an assessment is therapeutic because it expresses a level of caring and support.

A nurse is assessing activity tolerance of a patient. Which areas will the nurse assess? (Select all that apply.) a. Skeletal abnormalities b. Emotional factors c. Pregnancy status d. Race e. Age

Skeletal abnormalities Emotional factors Pregnancy status Age rationale: Physiological, emotional, and developmental factors (age) influence the patient's activity tolerance. Factors influencing activity tolerance include physiological factors such as skeletal abnormalities, emotional factors such as anxiety/depression, developmental factors such as age and gender, and pregnancy status. Race is not a factor because people of all races are faced with similar factors that affect their activity tolerance.

A patient arrives at the emergency department experiencing a headache and rates the pain as 7 on a 0 to 10 pain scale. Which nonpharmacological intervention does the nurse implement for this patient while awaiting orders for pain medication from the health care provider? a. Reassures the patient that the provider will come to the emergency department soon b. Softly plays music that the patient finds relaxing c. Frequently reassesses the patient's pain scores d. Teaches the patient how to do yoga

Softly plays music that the patient finds relaxing rationale: The appropriate nonpharmacological pain-management intervention is to quietly play music that the patient finds relaxing. Music diverts a person's attention away from pain and creates relaxation. Reassessing the patient's pain scores is done during evaluation. Building the patient's expectation of the provider's arrival does not address the patient's pain. Although yoga promotes relaxation, nurses teach relaxation techniques only when a patient is not experiencing acute pain. Because the patient is having acute pain, this is not an appropriate time to provide patient teaching.

A nurse is assisting the patient to perform isometric exercises. Which action will the nurse take? a. Encourage wearing tight shoes. b. Set the pace for the exercise session. c. Stop the exercise if pain is experienced. d. Force muscles or joints to go just beyond resistance.

Stop the exercise if pain is experienced. rationale: Instruct the patient to stop the activity if pain, fatigue, or discomfort is experienced. Assess for pain, shortness of breath, or a change in vital signs; if present, stop the exercise. Let each patient move at his or her own pace. Assess for joint limitations, and do not force a muscle or a joint during exercise. Teach patient to wear comfortable shoes and clothing for exercise.

A nurse is caring for a patient who recently had a stroke and is going to be discharged at the end of the week. The nurse notices that the patient is having difficulty with communication and becomes tearful at times. Which intervention will the nurse include in the patient's plan of care? a. Teach the patient about special assistive devices. b. Make the patient talk as much as possible. c. Obtain an order for antidepressant medications. d. Place a consult for a home health nurse.

Teach the patient about special assistive devices. rational: Because a stroke often causes partial or complete paralysis of one side of a patient's body, the patient needs special assistive devices. The nurse should include interventions that help the patient adapt to this deficit while maintaining independence. Teaching the patient to use assistive devices allows the patient to care for him- or herself. Making the patient talk can be inappropriate and demeaning. A home health nurse is not necessary as long as the patient is able to care for him- or herself. Instead of placing the patient on antidepressants, assist the patient in attempting to adapt behavior to the sensory deficit.

The nurse is caring for a patient in the sleep lab. Which assessment finding indicates to the nurse that the patient is in stage 4 NREM? a. The patient awakens easily. b. The patient's eyes rapidly move. c. The patient is difficult to awaken. d. The patient's vital signs are elevated.

The patient is difficult to awaken Rationale: The patient is difficult to arouse, vital signs are significantly lower, and this stage lasts about 15 to 30 minutes. Stage 4 NREM is the deepest stage of sleep. Lighter sleep is seen in stages 1 and 2, where the patient awakens easily. REM sleep is characterized by rapid eye movement

Obstructive sleep apnea

Use continuous positive airway pressure

Hydrogel

Very soothing to the patient and do not adhere to the wound bed

Quality

What does the pain feel like?

The nurse is teaching a group of older adults at an assisted-living facility about age-related physiological changes. Which question would be the most important to ask this group? a. "Are you able to hear the tornado sirens in your area?" b. "Are you able to read your favorite book?" c. "Are you able to remember the name of the person you just met?" d. "Are you able to open a jar of pickles?"

"Are you able to hear the tornado sirens in your area?" rationale: The ability to hear safety alerts and seek shelter is imperative to life safety. Although agerelated changes may cause a decrease in sight that affects reading, and although difficulties in remembering short-term information and opening jars as arthritis sets in are important to patients and to those caring for them, being able to hear safety alerts is the priority.

Which assessment question should the nurse ask to best understand how visual alterations are affecting the patient's self-care ability? a. "Have you stopped reading books or switched to books on audiotape?" b. "What do you do to protect yourself from injury at work?" c. "Are you able to prepare a meal or write a check?" d. "How does your vision impairment make you feel?"

"Are you able to prepare a meal or write a check?" rational: To best understand how vision is affecting self-care ability, the nurse wants to target questions to encompass what self-care tasks the patient has difficulty doing, such as preparing meals and writing checks. Switching to books on audiotape gives the nurse an idea of the severity of the deficit but not its impact on activities of daily living. Assessing whether the patient is taking measures for protection is important, but this does not address self-care activities. Emotional assessment of a patient is also important but does not properly address the goal of determining the effect of visual alterations on selfcare ability

A nurse is planning care for an older-adult patient who is experiencing pain. Which statement made by the nurse indicates the supervising nurse needs to follow up? a. "As adults age, their ability to perceive pain decreases." b. "Older patients may have low serum albumin in their blood, causing toxic effects of analgesic drugs." c. "Patients who have dementia probably experience pain, and their pain is not always well controlled." d. "It is safe to administer opioids to older adults as long as you start with small doses and frequently assess the patient's response to the medication."

"As adults age, their ability to perceive pain decreases." rationale: Aging does not affect the ability to perceive pain. This misconception must be corrected by the supervising nurse. All the other statements are true and require no follow-up. Opioids are safe to use in older adults as long as they are slowly titrated and the nurse frequently monitors the patient. Patients with dementia most likely experience unrelieved pain because their pain is difficult to assess. Older adults frequently eat poorly, resulting in low serum albumin levels. Many drugs are highly protein bound. In the presence of low serum albumin, more free drug (active form) is available, thus increasing the risk for side and/or toxic effects.

An 18-year-old male patient informs the nurse that he isn't sure if he is homosexual because he is attracted to both genders. Which response by the nurse will help establish a trusting relationship? a. "Don't worry. It's just a phase you will grow out of." b. "Those are abnormal impulses. You should seek therapy." c. "At your age, it is normal to be curious about both genders." d. "Having questions about sexuality is normal but if these sexual activities make you feel bad you should stop."

"At your age, it is normal to be curious about both genders." rational: Adolescents have questions about sexuality. The patient will feel most comfortable discussing his sexual concerns further if the nurse establishes that it is normal to ask questions about sexuality. The nurse can then discuss in greater detail. Although it is normal for young adults to be curious about sexuality, the nurse should use caution in giving advice on taking sexual action. The nurse should promote safe sex practices. Telling the patient not to worry dismisses his concern. Telling the patient that he is abnormal might offend the patient and prevent him from establishing an open relationship.

The nurse is completing a skin assessment on a medical-surgical patient. Which nursing assessment questions should be included in a skin integrity assessment? (Select all that apply.) a. "Can you easily change your position?" b. "Do you have sensitivity to heat or cold?" c. "How often do you need to use the toilet?" d. "What medications do you take?" e. "Is movement painful?" f. "Have you ever fallen?"

"Can you easily change your position?" "Do you have sensitivity to heat or cold?" "How often do you need to use the toilet?" "Is movement painful?" rationale: Changing positions is important for decreasing the pressure associated with long periods of time in the same position. If the patient is able to feel heat or cold and is mobile, she can protect herself by withdrawing from the source. Knowing toileting habits and any potential for incontinence is important because urine and feces in contact with the skin for long periods can increase skin breakdown. Knowing whether the patient has problems with painful movement will alert the nurse to any potential for decreased movement and increased risk for skin breakdown. Medications and falling are safety risk questions.

A nurse is teaching young adults about health risks. Which statement from a young adult indicates a correct understanding of the teaching? a. "It's probably safe for me to start smoking. At my age, there's not enough time for cancer to develop." b. "My mother had appendicitis so this increases my chance for developing appendicitis." c. "Controlling the amount of stress in my life may decrease the risk of illness." d. "I don't do drugs. I do drink coffee, but caffeine is not a drug."

"Controlling the amount of stress in my life may decrease the risk of illness." rationale: Lifestyle habits that activate the stress response increase the risk of illness; so, controlling this will decrease risk. Smoking is a well-documented risk factor for pulmonary, cardiac, and vascular disease as well as cancer in smokers and in individuals who receive secondhand smoke. The presence of certain chronic illnesses (not acute illnesses—appendicitis) in the family increases the family member's risk of developing a disease. Caffeine is a naturally occurring legal stimulant that is readily available. Caffeine stimulates catecholamine release, which, in turn, stimulates the central nervous system; it also increases gastric acid secretion, heart rate, and basal metabolic rate.

The patient and the nurse discuss the need for sleep. After the discussion, the patient is able to state factors that hinder sleep. Which statements indicate the patient has a good understanding of the teaching? (Select all that apply.) a. "Drinking coffee at 7 PM could interrupt my sleep." b. "Staying up late for a party can interrupt sleep patterns." c. "Exercising 2 hours before bedtime can decrease relaxation." d. "Changing the time of day that I eat dinner can disrupt sleep." e. "Worrying about work can disrupt my sleep." f. "Taking an antacid can decrease sleep."

"Drinking coffee at 7 PM could interrupt my sleep." "Staying up late for a party can interrupt sleep patterns." "Changing the time of day that I eat dinner can disrupt sleep." "Worrying about work can disrupt my sleep." rationale: Caffeine, alcohol, and nicotine consumed late in the evening produce insomnia. Worry over personal problems or situations frequently disrupts sleep. Alterations in routines, including changing mealtimes and staying up late at night for social activities, can disrupt sleep. Exercising 2 hours before bedtime actually increases a sense of fatigue and promotes relaxation. Taking an antacid does not decrease sleep

An Orthodox Jewish rabbi has been pronounced dead. The nursing assistive personnel respectfully ask family members to leave the room and go home as postmortem care is provided. Which statement from the supervising nurse is best? a. "I should have called a male colleague to handle the body." b. "Family members stay with the body until burial the next day." c. "I wish they would go home because we have work to do here." d. "Family will quietly leave after praying and touching the rabbi's head."

"Family members stay with the body until burial the next day." rationale: Jewish culture calls for family members or religious officials to stay with the body until the time of burial. A male provider is unnecessary. Requesting or expecting the family to go home is not providing culturally sensitive care. Hindus and Muslims usually have persons of the same gender handle the body after death. Buddhists often say prayers while touching and standing at the head of the deceased.

The nurse is caring for a patient who recently had unprotected sex with a partner who has HIV. Which response by the nurse is best? a. "You should have your blood drawn today to see if you were infected." b. "If you have the virus, you will have flu-like symptoms in 6 months." c. "Highly active antiretroviral therapy has been shown effective in slowing the disease process." d. "I will set you up with a support group to help you cope with dying within the next 10 years."

"Highly active antiretroviral therapy has been shown effective in slowing the disease process. rational: Highly active retroviral therapy increases the survival time of a person with HIV or AIDS. HIV antibodies will not show up in blood work for 6 weeks to 3 months. The infection stage of HIV lasts for about a month after the virus is contracted; during that time, the patient may experience flu-like symptoms. A support group may be beneficial for a patient who contracts HIV; however, it is unknown whether the patient has contracted HIV, and antiretroviral therapy has helped people live beyond the 10 years expected if HIV goes untreated.

The nurse is beginning a sleep assessment on a patient. Which question will be most appropriate for the nurse to ask initially? a. "What is going on?" b. "How are you sleeping?" c. "Are you taking any medications?" d. "What did you have for dinner last night?"

"How are you sleeping?" rationale: Sleep is a subjective experience. Only the patient is able to report whether or not it is sufficient and restful. Asking patients how they are sleeping is an introductory question. After this beginning question is asked, problems with sleep such as the nature of the problem, signs and symptoms, onset and duration of the issue, severity, predisposing factors, and the effect on the patient can be assessed. What is going on is too broad and open ended for information about sleep to be obtained specifically. Medications and food intake can be part of the detailed assessment of sleep issues

The nurse is completing an assessment on a patient who has a Stage IV pressure ulcer. The wound is odorous, and a drain is currently in place. Which statement by the patient indicates issues with self-concept? a. "I am so weak and tired. I want to feel better." b. "I am thinking I will be ready to go home early next week." c. "I am ready for my bath and linen change right now since this is awful." d. "I am hoping there will be something good for dinner tonight."

"I am ready for my bath and linen change right now since this is awful." rationale: Body image changes can influence self-concept. The wound is odorous, and a drain is in place. The patient who is asking for a bath and change in linens and states that this is awful gives you a clue that he or she may be concerned about the smell in the room. Factors that affect the patient's perception of the wound include the presence of scars, drains, odor from drainage, and temporary or permanent prosthetic devices. The patient's stating that he or she wants to feel better, talking about going home, and caring about what is for dinner could be interpreted as positive statements that indicate progress along the health journey.

A patient has sleep deprivation. Which statement by the patient will indicate to the nurse that outcomes are being met? a. "I wake up only once a night to go to the bathroom." b. "I feel rested when I wake up in the morning." c. "I go to sleep within 30 minutes of lying down." d. "I only take a 20-minute nap during the day."

"I feel rested when I wake up in the morning." rationale: Being able to sleep and feeling rested would indicate that outcomes are being met for sleep deprivation. Limiting a nap to 20 minutes is an intervention to promote sleep. Going to sleep within 30 minutes indicates a goal for insomnia. Waking up only once may indicate nocturia is improving but does not relate to sleep deprivation

The nurse discussed threats to adult safety with a college group. Which of the following statements would indicate understanding of the topic? a. "Our campus is safe; we leave our dorms unlocked all the time." b. "As long as I have only two drinks, I can still be the designated driver." c. "I am young, so I can work nights and go to school with 2 hours' sleep." d. "I guess smoking even at parties is not good for my body."

"I guess smoking even at parties is not good for my body." rationale: Lifestyle choices frequently affect adult safety. Smoking conveys great risk for pulmonary and cardiovascular disease. It is prudent to secure belongings. When an individual has been determined to be the designated driver, that individual does not consume alcohol, beer, or wine. Sleep is important no matter the age of the individual and is important for rest and integration of learning. The average young adult needs 6 1/2 to 8 hours of sleep each night.

A nurse is interviewing a woman who uses a diaphragm. Which information from the patient will require the nurse to follow up? a. "I have lost 12 pounds on this diet." b. "I use the diaphragm to prevent pregnancy." c. "I use a contraceptive cream with my diaphragm." d. "I know this provides a barrier over the cervical opening."

"I have lost 12 pounds on this diet." rational: The woman needs to be refitted after a significant change in weight (10-pound gain or loss) or pregnancy. The diaphragm is a round, rubber dome that has a flexible spring around the edge. It is used with a contraceptive cream or jelly and is inserted in the vagina so it provides a contraceptive barrier over the cervical opening.

The nurse is instructing the student nurse regarding discharge teaching and medications. Which response by the student would indicate that learning has occurred? a. "I need to be precise when teaching a patient about Zyprexa (olanzapine) and Zyrtec (cetirizine)." b. "The medications can be picked up at the pharmacy on the way out of the hospital." c. "I need to be sure to give the patient leftover medications from the medication drawer." d. "I need to remember to teach the patient to take all medications at the same time of the day."

"I need to be precise when teaching a patient about Zyprexa (olanzapine) and Zyrtec (cetirizine)." rationale: Zyprexa and Zyrtec are sound-alike, look-alike medications. Zyprexa is an antipsychotic and Zyrtec an antihistamine; these agents treat two different conditions. Bringing the differences and similarities in spelling and sound to the attention of the patient is important for patient safety. Medications are not distributed by the hospital, and medications do not need to be administered at the same time each day.

A nurse is caring for a patient with chronic pain. Which statement by the nurse indicates an understanding of pain management? a. "This patient says the pain is a 5 but is not acting like it. I am not going to give any pain medication." b. "I need to reassess the patient's pain 1 hour after administering oral pain medication." c. "It wasn't time for the patient's medication, so when it was requested, I gave a placebo." d. "The patient is sleeping, so I pushed the PCA button."

"I need to reassess the patient's pain 1 hour after administering oral pain medication. rational: Be sure to evaluate after an appropriate period of time. For instance, oral medications usually peak in about 1 hour, whereas IVP medications peak in 15 to 30 minutes. Ask a patient if a medication alleviates the pain when it is peaking. Because oral medications usually peak in about an hour, you need to reassess the patient's pain within an hour of administration. Nurses must believe any patient report of pain, even if nonverbal communication is not consistent with pain ratings. The patient is the only person who should push the PCA button. Pushing the PCA when a patient is sleeping is dangerous and may lead to narcotic overdose or respiratory depression. Giving the patient a placebo and telling the patient it is medication is unethical.

Which patient statement is the most reliable indicator that an older adult has the correct understanding of health promotion activities? a. "I need to increase my fat intake and limit protein." b. "I still keep my dentist appointments even though I have partials now." c. "I should discontinue my fitness club membership for safety reasons." d. "I'm up-to-date on my immunizations, but at my age, I don't need the influenza vaccine."

"I still keep my dentist appointments even though I have partials now." rationale: General preventive measures for the nurse to recommend to older adults include keeping regular dental appointments to promote good oral hygiene, eating a low-fat, well-balanced diet, exercising regularly, and maintaining immunizations for seasonal influenza, tetanus, diphtheria and pertussis, shingles, and pneumococcal disease.

The older patient presents to the emergency department after stepping in front of a car at a crosswalk. After the patient has been triaged, the nurse interviews the patient. Which of the following comments would require follow-up by the nurse? a. "I try to exercise, so I walk that block almost every day." b. "I waited and stepped out when the traffic sign said go." c. "The car was going too fast, the speed limit is 20." d. "I was so surprised; I didn't see or hear the car coming."

"I was so surprised; I didn't see or hear the car coming." rationale: The patient did not see or hear the car coming. As patients age, sensory impairment can increase the risk for injury. This statement by the patient would require follow-up by the nurse. The patient needs hearing and eye examinations. Exercise is important at every stage of development. The patient seemed to comprehend how to cross an intersection correctly and was able to determine the speed of the car.

The nurse is teaching a new mother about the sleep requirements of a neonate. Which comment by the patient indicates a correct understanding of the teaching? a. "I can't wait to get the baby home to play with the brothers and sisters." b. "I will ask my mom to come after the first week, when the baby is more alert." c. "I can get the baby on a sleeping schedule the first week while my mom is here." d. "I won't be able to nap during the day because the baby will be awake.

"I will ask my mom to come after the first week, when the baby is more alert. rationale: The patient indicates an understanding when asking the mother to come after the first week. The neonate up to the age of 3 months averages about 16 hours of sleep a day, sleeping almost constantly during the first week. The baby will sleep rather than play. The baby will not be on a sleeping schedule the first week home. The mother will be able to nap since the baby sleeps 16 hours a day

An outcome for an older-adult patient living alone is to be free from falls. Which statement indicates the patient correctly understands the teaching on safety concerns? a. "I'll take my time getting up from the bed or chair." b. "I should dim the lighting outside to decrease the glare in my eyes." c. "I'll leave my throw rugs in place so that my feet won't touch the cold tile." d. "I should wear my favorite smooth bottom socks to protect my feet when walking around."

"I'll take my time getting up from the bed or chair." rationale: Postural hypotension is an intrinsic factor that can cause falls. Changing positions slowly indicates a correct understanding of this concept. Environmental hazards outside and within the home such as poor lighting, slippery or wet flooring, and items on floor that are easy to trip over such as throw rugs are other factors that can lead to falls. Impaired vision and poor lighting are other risk factors for falls and should be avoided (dim lighting). Inappropriate footwear such as smooth bottom socks also contributes to falls.

A patient cancels a scheduled appointment because the patient will be attending a Shivah for a family member. Which response by the nurse is best? a. "When families come together for end-of-life decisions, it provides connections." b. "We will reschedule so the appointment does not fall on the Sabbath." c. "Missionary outreach is so important for spiritual comfort." d. "I'm so sorry for your loss."

"I'm so sorry for your loss." rationale: A death has occurred and saying that you are sorry for their loss is appropriate. The Jewish mourning ritual of Shivah is a time period when normal life activities come to a stop. Those mourning welcome friends into the home as a way of honoring the dead and receive support during the mourning period. Cultural variables can influence a person's response to grief. It is not when families come together for end-of-life decisions. It is not because the appointment fell on the Sabbath. It is not about missionary outreach.

A nurse discusses the risks of repeated sun exposure with a young-adult patient. Which response will the nurse most expect from this patient? a. "I should consider participating in a health fair about safe sun practices." b. "I'll make an appointment with my doctor right away for a full skin check." c. "I've had this mole my whole life. So what if it changed color? My skin is fine." d. "I have a mole that has been bothering me. I'll call my family doctor for an appointment to get it checked."

"I've had this mole my whole life. So what if it changed color? My skin is fine." rationale: Most typically young adults would say that their skin is fine. Young adults often ignore physical symptoms and often postpone seeking health care. Making an appointment right away with the doctor and participating in health fairs are not typical behaviors of young adults for the same reason.

The nurse is providing information regarding safety and accidental poisoning to a grandmother who will be taking custody of a 1-year-old grandchild. Which of the following comments would indicate that the grandmother needs further instruction? a. "The number for poison control is 800-222-1222." b. "Never induce vomiting if my grandchild drinks bleach." c. "I should call 911 if my grandchild loses consciousness." d. "If my grandchild eats a plant, I should provide syrup of ipecac."

"If my grandchild eats a plant, I should provide syrup of ipecac." rationale: Syrup of ipecac to induce vomiting after ingestion of a poison has not been proven effective in preventing poisoning. This medication should not be administered to the child. The poison control number is 800-222-1222. After a caustic substance such as bleach has been drunk, do not induce vomiting. This can cause further burning and injury as the medication is eliminated. Loss of consciousness associated with poisoning requires calling 911.

The nurse is providing an educational session on sleep regulation for new nurses in the Sleep Disorder Treatment Center. Which statement by the nurses will best indicate that the teaching is effective? a. "If the patient has a disease process in the central nervous system, it can influence the functions of sleep." b. "If the patient has a disease process in the cranial nerves, it can influence the functions of sleep." c. "If the patient has an interruption in the urinary pathways, it can influence the functions of sleep." d. "If the patient has an interruption in the spinal reflexes, it can influence the functions of sleep.

"If the patient has a disease process in the central nervous system, it can influence the functions of sleep. rationale: Sleep involves a sequence of physiological states maintained by the central nervous system. It is associated with changes in the peripheral nervous, endocrine, cardiovascular, respiratory, and muscular systems. A disease process associated with the cranial nerves, urinary pathway, or spinal reflexes may influence a person's ability to sleep, but the best answer is the central nervous system

A nurse teaches the patient about the gate control theory. Which statement made by a patient reflects a correct understanding about the relationship between the gate control theory of pain and the use of meditation to relieve pain? a. "Meditation controls pain by blocking pain impulses from coming through the gate." b. "Meditation alters the chemical composition of pain neuroregulators, which closes the gate." c. "Meditation will help me sleep through the pain because it opens the gate." d. "Meditation stops the occurrence of pain stimuli."

"Meditation controls pain by blocking pain impulses from coming through the gate." rationale: According to this theory, gating mechanisms located along the central nervous system regulate or block pain impulses. Pain impulses pass through when a gate is open and are blocked when a gate is closed. Nonpharmacologic pain-relief measures, such as meditation, work by closing the gates, which keeps pain impulses from coming through. Meditation does not open pain gates or stop pain from occurring. Meditation also does not have an effect on pain neuroregulators.

A nurse is caring for an older-adult patient who was in a motor vehicle accident because the patient thought the stoplight was green. The patient asks the nurse "Should Istop driving?" Which response by the nurse is most therapeutic? a. "Yes, you should stop driving. As you age, your cognitive function declines, and becoming confused puts everyone else on the road at risk." b. "Yes, you should ask family members to drive you around from now on. Your reflex skills have declined so much you can't avoid an accident." c. "No, as you age, you lose the ability to see colors. You need to think about stoplights in a new way. If the top is lit up, it means stop, and if the bottom is lit up, it means go." d. "No, instead you should see your ophthalmologist and get some glasses to help you see better."

"No, as you age, you lose the ability to see colors. You need to think about stoplights in a new way. If the top is lit up, it means stop, and if the bottom is lit up, it means go." rational: Part of the normal aging process is reduced ability to see colors. The nurse should teach the patient new ways to adapt to this deficit. This patient's accident was not due to impaired cognitive function or reflexes. Glasses will not assist the patient in color discrimination.

The nurse is leading a seminar about menopause and age-related changes. Which response from a group member indicates the nurse needs to follow up? a. "Hormones of sexual regulation decrease with aging." b. "Orgasms are no longer achievable after menopause." c. "The excitement phase is prolonged as we age." d. "As men age, their erection may be less firm."

"Orgasms are no longer achievable after menopause." rational: Believing that orgasms are no long achievable requires follow-up to correct this misconception. Orgasms are achievable at any age; however, it may take longer with aging. All other statements indicate that the patient does have an understanding of age-related changes and needs no follow-up. Both genders experience a reduced availability of sex hormones. The excitement phase prolongs in both men and women. Men often have erections that are less firm and shorter acting.

A single parent is discussing the sleep needs of a preschooler with the nurse. Which information will the nurse share with the parent? a. "Most preschoolers sleep soundly all night long." b. "It is important that the 5-year-old get a nap every day." c. "On average, the preschooler needs to sleep 10 hours a night." d. "Preschoolers may have trouble settling down after a busy day.

"Preschoolers may have trouble settling down after a busy day rationale: The preschooler usually has difficulty relaxing or settling down after long, active days. By the age of 5, naps are rare for children, except those for whom a siesta is a custom. Preschoolers frequently awaken during the night. On average, a preschooler needs about 12 hours of sleep

A new nurse is caring for a patient who is undergoing chemotherapy for cancer. The patient is becoming malnourished because nothing tastes good. Which recommendation by the nurse will be most appropriate for this patient? a. "Rinse your mouth several times a day to hydrate your taste buds." b. "Avoid adding spices or lemon juice to food to prevent nausea." c. "Blend foods together in interesting flavor combinations." d. "Eat soft foods that are easy to chew and swallow."

"Rinse your mouth several times a day to hydrate your taste buds." rational: Good oral hygiene keeps the taste buds well hydrated. Having an unpleasant taste in the mouth discourages the patient from eating. Well-seasoned, differently textured food eaten separately heightens taste perception. Avoid blending foods together because this makes it difficult to identify tastes. Texturized, spicy, and aromatic foods stimulate and make eating more enjoyable. Flavored vinegar or lemon juice adds tartness to food.

A nurse is caring for a 15-year-old who in the past 6 months has had multiple male and female sexual partners. Which response by the nurse will be most effective? a. "Sexually transmitted infections and unwanted pregnancy are a real risk. Let's discuss what you think is the best method for protecting yourself." b. "Having sexual interaction with both males and females places you at higher risk for STIs. To protect yourself, you need to decide which orientation you are." c. "Your current friends are leading you to make poor choices. You should find new friends to hang out with." d. "I think it's best to notify your parents. They know what's best for you and can help make sure you practice safe sex."

"Sexually transmitted infections and unwanted pregnancy are a real risk. Let's discuss what you think is the best method for protecting yourself." rational: Some adolescents participate in risky behaviors. The nurse should acknowledge this feeling to the patient and offer education and alternatives, while giving the patient the autonomy to make his or her own decisions. Adolescents who engage in sexual risk behaviors experience negative health outcomes such as STIs and unintended pregnancy. In addition, the pattern of risk-taking behavior tends to be established and continue throughout life. The nurse should not force the patient to make a choice of orientation and should not pass judgment on a patient's sexual orientation or social network; this would make the patient feel defensive and would eliminate the trust in the relationship. Involving parents is not the first line of action; parents should be notified only if the child is in a life or death situation.

A nurse is caring for a patient who is experiencing pain following abdominal surgery. Which information is important for the nurse to share with the patient when providing patient education about effective pain management? a. "To prevent overdose, you need to wait to ask for pain medication until you begin to experience pain." b. "You should take your medication after you walk to make sure you do not fall while you are walking." c. "We should work together to create a schedule to provide regular dosing of medication." d. "When you experience severe pain, you will need to take oral pain medications."

"We should work together to create a schedule to provide regular dosing of medication." rationale: One way to maximize pain relief while potentially decreasing opioid use is to administer analgesics around the clock (ATC) rather than on a prn basis. This approach ensures a more constant therapeutic blood level of an analgesic. Working with the patient to design a schedule allows the patient to be a full partner in the care provided. The nurse should not wait until pain is experienced because it takes medications 10-30 minutes to begin to relieve pain. The nurse administers pain medications before painful activities, such as walking, and administers intravenous medications when a patient is having severe pain.

The nurse is assessing how a patient's pain is affecting mobility. Which assessment question is most appropriate? a. "Have you considered working with a physical therapist?" b. "What activities, if any, has your pain prevented you from doing?" c. "Would you please rate your pain on a scale from 0 to 10 for me?" d. "When does your pain medication typically take effect on your pain?"

"What activities, if any, has your pain prevented you from doing?" rational : Because the nurse is interested in knowing whether the patient's pain is affecting mobility, the priority assessment question is to ask the patient how the pain affects ability to participate in normal activities of daily living. Although a physical therapist is a good resource to have, especially if pain is severely affecting mobility, considering working with a physical therapist does not describe the effect of pain on the patient's mobility. Assessing quality of pain and effectiveness of pain medication does not help the nurse to understand how it is affecting the patient's mobility.

A nurse is caring for a patient who recently had abdominal surgery and is experiencing severe pain. The patient's blood pressure is 110/60 mm Hg, and heart rate is 60 beats/min. Additionally, the patient does not appear to be in any distress. Which response by the nurse is most therapeutic? a. "Your vitals do not show that you are having pain; can you describe your pain?" b. "OK, I will go get you some narcotic pain relievers immediately." c. "What would you like to try to alleviate your pain?" d. "You do not look like you are in pain."

"What would you like to try to alleviate your pain?" rationale: Be sure the patient is a partner in making decisions about the best approaches for managing pain. A patient knows the most about his or her pain and is an important partner in selecting successful pain therapies. The nurse must believe that a patient is in pain whenever the patient reports that he or she is in pain, even if the patient does not appear to be in pain. The nurse must be careful to not judge the patient based on vital signs or nonverbal communication and must not assume that the patient is seeking narcotics. The patient is a partner in pain management, so going to get narcotics to treat the pain without consulting with the patient first is not appropriate.

A home health nurse is performing a home assessment for safety. Which of the following comments by the patient would indicate a need for further education? a. "I will schedule an appointment with a chimney inspector next week." b. "Daylight savings is the time to change batteries on the carbon monoxide detector." c. "If I feel dizzy when using the heater, I need to have it inspected." d. "When it is cold outside in the winter, I can warm my car up in the garage."

"When it is cold outside in the winter, I can warm my car up in the garage." rationale: Allowing a car to run in the garage introduces carbon monoxide into the environment and decreases the available oxygen for human consumption. Garages should be opened and not just cracked to allow fresh air into the space and allay this concern. Checking the chimney and heater, changing the batteries on the detector, and following up on symptoms such as dizziness, nausea, and fatigue are all statements that would indicate that the individual has understood the education

The nurse has brought a patient the scheduled pain medication. The patient asks the nurse to wait to give pain medication until the time for the dressing change, which is 2 hours away. Which response by the nurse is most therapeutic? a. "This medication will still be providing you relief at the time of your dressing change." b. "OK, swallow this pain pill, and I will return in a minute to change your dressing." c. "Would you like medication to be given for dressing changes in addition to your regularly scheduled medication?" d. "Your medication is scheduled for this time, and I can't adjust the time for you. I'm sorry, but you must take your pill right now.

"Would you like medication to be given for dressing changes in addition to your regularly scheduled medication?" rational: Additional doses of medication can be given to patients in certain circumstances, as with an extensive dressing change, when the health care provider is notified that more medication is needed. It is the nurse's responsibility to communicate with the provider and with the patient about a pain-control plan that works for both. By asking to hold off on the dose, the patient is indicating that the dressing changes are extremely painful. The regularly scheduled dose might not be as effective for the patient 2 hours later when the dressing change is scheduled. Oral medications take 30 to 60 minutes to take effect. If the nurse began the dressing change right then, the medication would not have been absorbed yet. The patient has the right to refuse to take a medication.

The nurse is teaching a student nurse about pain assessment scales. Which statement by the student indicates effective teaching? a. "You cannot use a pain scale to compare the pain of my patient with the pain of your patient." b. "When patients say they don't need pain medication, they aren't in pain." c. "A patient's behavior is more reliable than the patient's report of pain." d. "Pain assessment scales determine the quality of a patient's pa

"You cannot use a pain scale to compare the pain of my patient with the pain of your patient." rational: Do not use a pain scale to compare the pain of one patient to that of another. Pain is subjective and cannot be compared to the pain of another patient. Some patients do not express their pain (stoic) or do not wish to take medications to relieve the pain. This does not mean they aren't in pain. A patient's behavior is not more reliable than the patient's report of pain. Pain scales help determine severity or intensity, not quality.

A nursing student is providing education to a group of older adults who are in an independent living retirement village. Which statement made by the nursing student requires the nurse to intervene? a. "Avoiding alcohol use will enhance your sexual functioning." b. "You need to tell your partner how you feel about sex and any fears you may have." c. "You do not need to worry about getting a sexually transmitted infection at this point in your life." d. "Using pillows and taking pain medication if needed before having sexual intercourse often help alleviate pain and improve sexual functioning."

"You do not need to worry about getting a sexually transmitted infection at this point in your life." rational: Research indicates many older adults are more sexuality active than previously thought and engage in high-risk sexual encounters, resulting in a steady increase HIV and STI rates over the past 12 years. Therefore, the nurse needs to intervene when the student tells the older adults that they are not at risk for developing an STI. Avoiding the use of alcohol; using pillows; taking pain medications before having intercourse if needed; and communicating thoughts, fears, and feelings about sex all enhance sexual functioning.

A nurse is providing discharge teaching for a patient with a fractured humerus. The patient is going home with hydrocodone. Which important patient education does the nurse provide? a. "You need to drink plenty of fluids and eat a diet high in fiber." b. "Narcotics can be addictive, so do not take them unless you are in severe pain." c. "Be sure to eat a meal high in fat before taking the medication, to avoid a stomach ulcer." d. "As your pain severity lessens, you will begin to give yourself once-daily intramuscular injections."

"You need to drink plenty of fluids and eat a diet high in fiber." rationale: A common side effect of opioid analgesics is constipation. Therefore, the nurse encourages the patient to drink fluids and eat fiber to prevent constipation. Although medications can be irritating to the stomach, eating a diet high in fat does not prevent gastric ulcers. To best manage pain, the patient needs to take pain medication before painful procedures or activities or before pain becomes severe. As the patient's pain gets better, the strength of the medications will decrease. IM, IV, and topical analgesics are used for more severe and chronic pain.

The nurse is caring for an elderly woman and notices that she is not using her cane properly. Which of the following statements by the nurse would most likely elicit a positive response from the patient? a. "You're doing that all wrong. Let me show you how to do it." b. "I don't know who showed how to use the cane like that, but you're not doing it right. Let me show you again." c. "You use the cane the way I did before I was shown a way to keep from tripping over it; do you mind if I show you?" d. "I used to use the cane the same way you are using it: the wrong way. I'll show you the right way to do it."

"You use the cane the way I did before I was shown a way to keep from tripping over it; do you mind if I show you?" rational: When the nurse is confident and skillful instead of coming across as an authority on the subject, the patient's defenses will be down, making the patient more willing to listen without feeling embarrassed. Respectful communication is necessary.

.A patient who has had type 2 diabetes for 26 years is beginning to experience peripheral neuropathy in the feet and lower leg. The nurse is providing education to the patient to prevent injury to the feet by wearing shoes or slippers when walking. Which statement made by the nurse best explains the rationale for this instruction? a. "Wearing shoes blocks pain perception and helps you adapt to pain, which ends up protecting your feet." b. "Shoes provide nonpharmacological pain relief to people with diabetes and peripheral neuropathy." c. "The neurological gates open when wearing shoes, which protects your feet." d. "If you step on something without shoes, you might not feel it; this could possibly cause injury to your foot."

. "If you step on something without shoes, you might not feel it; this could possibly cause injury to your foot." rationale: Any factor that interrupts or influences normal pain reception or perception (e.g., spinal cord injury, peripheral neuropathy, or neurological disease) affects a patient's awareness of and response to pain. The patient will no longer have protective reflexes to prevent injury to the feet. Wearing shoes prevents the patient from injuring the feet because they protect the feet. Shoes do not block pain perception, and they do not help people adapt to pain. Shoes are not a form of nonpharmacological pain relief. Wearing shoes will not have an effect on opening or closing the pain gates.

The nurse is caring for a patient who is a well-known surgeon at the hospital. The nurse notices the patient becoming more agitated and withdrawn with each group of surgeon visitors. The nurse and patient agree to place a "Do not disturb" sign on the door. A few hours later, the nurse notices a surgeon who is not involved in the patient's care attempting to enter the room. Which response by the nurse is most appropriate? a. Call for security to remove the surgeon. b. Allow the surgeon to enter. c. Firmly explain that the patient does not wish to have visitors at this time. d. Scold the surgeon for not obeying the sign and respecting the patient's wishes.

. Firmly explain that the patient does not wish to have visitors at this time. rational: The nurse acts as an advocate for the patient (who is experiencing sensory overload and would benefit from a quiet environment) by firmly and politely asking the surgeon to leave regardless of position in the hospital. A creative solution to decrease excessive environmental stimuli that prevents restful, healing sleep is to institute "quiet time" in ICUs. Data collected from one hospital that implemented 1 hour of quiet time daily found decreased staff and unit noise and improved patient satisfaction. The nurse should not allow anyone to enter unless the patient approves it. Security is not a necessary measure at this time. The nurse should handle the situation with professionalism when addressing the surgeon; scolding the visitor is not appropriate.

The nurse is performing a moist-to-dry dressing. The nurse has prepared the supplies, solution, and removed the old dressing. In which order will the nurse implement the steps, starting with the first one? 1. Apply sterile gloves. 2. Cover and secure topper dressing. 3. Assess wound and surrounding skin. 4. Moisten gauze with prescribed solution. 5. Gently wring out excess solution and unfold. 6. Loosely pack until all wound surfaces are in contact with gauze. a. 4, 3, 1, 5, 6, 2 b. 1, 3, 4, 5, 6, 2 c. 4, 1, 3, 5, 6, 2 d. 1, 4, 3, 5, 6, 2

1, 3, 4, 5, 6, 2 rationale: The steps for a moist-to-dry dressing are as follows: (1) Apply sterile gloves; (2) assess appearance of surrounding skin; (3) moisten gauze with prescribed solution. (4) Gently wring out excess solution and unfold; apply gauze as single layer directly onto wound surface. (5) If wound is deep, gently pack dressing into wound base by hand until all wound surfaces are in contact with gauze; (6) cover with sterile dry gauze and secure topper dressing.

The nurse is teaching a patient how to sit with crutches. In which order will the nurse present the instructions starting with the first step? 1. Place both crutches in one hand. 2. Grasp arm of chair with free hand. 3. Completely lower self into chair. 4. Transfer weight to crutches and unaffected leg. a. 4, 1, 2, 3 b. 1, 4, 2, 3 c. 1, 2, 4, 3 d. 4, 2, 1, 3

1, 4, 2, 3 rationale: A patient is sitting in a chair with crutches. Both crutches are held in one hand. The patient then transfers weight to the crutches and the unaffected leg. Next, the patient grasps the arm of the chair with the free hand and begins to lower self into chair. Finally, the patient completely lowers self into chair.

A nurse is caring for a patient with dyspareunia. In which order will the nurse provide care, starting with the first step? 1. Determine which signs and symptoms of dyspareunia the patient has. 2. Mutually decide upon goals and objectives for dyspareunia. 3. Ask the patient if the dyspareunia is improving. 4. Develop a nursing diagnosis for the patient. 5. Use resources to help resolve the problem. a. 5, 3, 1, 4, 2 b. 1, 4, 2, 5, 3 c. 3, 1, 4, 2, 5 d. 4, 2, 5, 3, 1

1, 4, 2, 5, 3 rational: The nurse should use the nursing process when caring for patients with sexual dysfunction. Determine signs and symptoms (assessment); develop a nursing diagnosis (diagnosis); mutually decide upon goals (planning); use resources to help resolve the problem (implementation); and ask if the dyspareunia is improving (evaluation).

A nurse is describing the transmission of sound to a patient. In which order will the nurse list the pathway of sound, beginning with the first structure? 1. Eardrum 2. Perilymph 3. Oval window 4. Bony ossicles 5. Eighth cranial nerve a. 1, 5, 2, 4, 3 b. 1, 3, 4, 2, 5 c. 1, 2, 4, 5, 3 d. 1, 4, 3, 2, 5

1, 4, 3, 2, 5 rational: Vibration of the eardrum transmits through the bony ossicles. Vibrations at the oval window transmit in perilymph within the inner ear to stimulate hair cells that send impulses along the eighth cranial nerve to the brain.

The patient has a risk for skin impairment and has a 15 on the Braden Scale upon admission. The nurse has implemented interventions. Upon reassessment, which Braden score will be the best sign that the risk for skin breakdown is removed? a.12 b.13 c.20 d.23

23 rationale: The best sign is a perfect score of 23. The Braden Scale is composed of six subscales: sensory perception, moisture, activity, mobility, nutrition, and friction and shear. The total score ranges from 6 to 23, and a lower total score indicates a higher risk for pressure ulcer development. The cutoff score for onset of pressure ulcer risk with the Braden Scale in the general adult population is 18.

A nurse is developing an exercise plan for a middle-aged patient. In which order will the nurse instruct the patient to execute the plan, beginning with the first step? 1. Design the fitness program. 2. Assemble equipment. 3. Assess fitness level. 4. Monitor progress. 5. Get started. a. 5, 1, 3, 2, 4 b. 1, 2, 3, 5, 4 c. 2, 5, 3, 1, 4 d. 3, 1, 2, 5, 4

3, 1, 2, 5, 4 rationale: Five steps to beginning an exercise program are Step 1: Assess fitness level; Step 2: Design the fitness program; Step 3: Assemble equipment; Step 4: Get started; and Step 5: Monitor progress.

A nurse is teaching a health promotion class for older adults. In which order will the nurse list the most common to least common conditions that can lead to death in older adults? 1. Chronic obstructive lung disease 2. Cerebrovascular accidents 3. Heart disease 4. Cancer a. 4, 1, 2, 3 b. 3, 4, 1, 2 c. 2, 3, 4, 1 d. 1, 2, 3, 4

3, 4, 1, 2 rationale: Heart disease is the leading cause of death in older adults followed by cancer, chronic lung disease, and stroke (cerebrovascular accidents).

The nurse needs to move a patient up in bed using a drawsheet. The nurse has another nurse helping. In which order will the nurses perform the steps, beginning with the first one? 1. Grasp the drawsheet firmly near the patient. 2. Move the patient and drawsheet to the desired position. 3. Position one nurse at each side of the bed. 4. Place the drawsheet under the patient from shoulder to thigh. 5. Place your feet apart with a forward-backward stance. 6. Flex knees and hips and on count of three shift weight from the front to back leg. a. 1, 4, 5, 6, 3, 2 b. 4, 1, 3, 5, 6, 2 c. 3, 4, 1, 5, 6, 2 d. 5, 6, 3, 1, 4, 2

3, 4, 1, 5, 6, 2 rationale: Assisting a patient up in bed with a drawsheet (two or three nurses): (1) Place the patient supine with the head of the bed flat. A nurse stands on each side of the bed. (2) Remove the pillow from under the patient's head and shoulders and place it at the head of the bed. (3) Turn the patient side to side to place the drawsheet under the patient, extending it from shoulders to thighs. (4) Return the patient to the supine position. (5) Fanfold the drawsheet on both sides, with each nurse grasping ӄrmly near the patient. (6) Nurses place their feet apart with a forward-backward stance. Nurses should flex knees and hips. On the count of three, nurses should shift their weight from front to back leg and move the patient and drawsheet to the desired position in the bed.

A nurse is teaching a patient about vision. In which order will the nurse describe the pathway for vision, beginning with the first structure? 1. Lens 2. Pupil 3. Retina 4. Cornea 5. Optic nerve a. 2, 1, 4, 5, 3 b. 1, 2, 4, 3, 5 c. 4, 2, 1, 3, 5 d. 5, 2, 4, 1, 3

4, 2, 1, 3, 5 rational: Light rays enter the convex cornea and begin to converge. An adjustment of light rays occurs as they pass through the pupil and lens. Change in the shape of the lens focuses light on the retina. The sensory retina contains the rods and cones (i.e., photoreceptor cells sensitive to stimulation from light). Photoreceptor cells send electrical potentials by way of the optic nerve to the brain.

The patient has been diagnosed with a respiratory illness and complains of shortness of breath. The nurse adjusts the temperature to facilitate the comfort of the patient. What is the usual comfort range for most patients? a. 65° F to 75° F b. 60° F to 75° F c. 15° C to 17° C d. 25° C to 28° C

65° F to 75° F rationale: The comfort zone for most individuals is the range between 65° F and 75° F (18.3° C to 23.9° C). The other ranges do not reflect the average person's comfort zone.

A 55-year-old patient is preparing to start an exercise program. The health care provider wants 60% of maximum target heart rate. Calculate the heart rate that the nurse will add to the care plan as the target heart rate. Record answer as a whole number. _________ maximum heart rate

99 rationale: Teach patients to calculate their maximum heart rate by subtracting their current age in years from 220 and then obtaining their target heart rate by taking 60% to 90% of the maximum, depending on their health care provider's recommendation. 220 - 55 = 165 ´ 0.6 = 99.

The nurse is preparing pain medications. To which patient does the nurse anticipate administering an opioid fentanyl patch? a. A 15-year-old adolescent with a fractured femur b. A 30-year-old adult with cellulitis c. A 50-year-old patient with prostate cancer d. An 80-year-old patient with a broken hip

A 50-year-old patient with prostate cancer rationale: Transdermal fentanyl (patch), which is 100 times more potent than morphine, is available for opioid-tolerant patients with cancer or chronic pain (prostate cancer). It delivers predetermined doses that provide analgesia for up to 72 hours. The other patients are expected to experience acute pain (fractured femur, cellulitis, and broken hip). Therefore, they will most likely benefit from oral or IV opioids for short-term pain relief.

The nurse is starting an exercise program in a local community as a health promotion project. Which information will the nurse include in the teaching session? a. A cool-down period lasts about 5 to 10 minutes. b. The purpose of weight training is to bulk up muscles. c. Resistance training is appropriate for warm-up and cool-down periods. d. Aerobic exercise should be done 3 to 5 times per week for about 20 minutes.

A cool-down period lasts about 5 to 10 minutes. rationale: The cool-down period follows the exercise routine and usually lasts about 5 to 10 minutes. The purposes of weight training from a health perspective are to develop tone and strength and to simulate and maintain healthy bone. Stretching and flexibility exercises are ideal for warm-up and cool-down periods. The recommended frequency of aerobic exercise is 3 to 5 times per week or every other day for approximately 30 minutes.

During an assessment of a patient, the nurse finds the patient experiences vertigo. Which sensory deficit will the nurse assess further? a. Neurological deficit b. Visual deficit c. Hearing deficit d. Balance deficit

Balance deficit rational: Vertigo is a result of vestibular dysfunction and often is precipitated by a change in head position. Neurological deficits include peripheral neuropathy and stroke. Visual deficits include presbyopia, cataracts, glaucoma, and macular degeneration. Hearing deficits include presbycusis and cerumen accumulation.

The nurse is caring for a patient who is immobile and needs to be turned every 2 hours. The patient has poor lower extremity circulation, and the nurse is concerned about irritation of the patient's toes. Which device will the nurse use? a. Hand rolls b. A foot cradle c. A trapeze bar d. A trochanter roll

A foot cradle rationale: A foot cradle may be used in patients with poor peripheral circulation as a means of reducing pressure on the tips of a patient's toes. A trochanter roll prevents external rotation of the hips when the patient is in a supine position. Hand rolls maintain the thumb in slight adduction and in opposition to the ӄngers. The trapeze bar is a triangular device that hangs down from a securely fastened overhead bar that is attached to the bedframe. It allows the patient to pull with the upper extremities to raise the trunk oӄ the bed, to assist in transfer from bed to wheelchair, or to perform upper arm exercises.

A 70-year-old patient who suffers from worsening dementia is no longer able to live alone. The nurse is discussing health care services and possible long-term living arrangements with the patient's only son. What will the nurse suggest? a. An apartment setting with neighbors close by b. Having the patient utilize weekly home health visits c. A nursing center because home care is no longer safe d. That placement is irrelevant because the patient is retreating to a place of inactivity

A nursing center because home care is no longer safe rationale: Some family caregivers consider nursing center placement when in-home care becomes increasingly difficult or when convalescence from hospitalization requires more assistance than the family is able to provide. An apartment setting and the use of home health visits are not appropriate because living at home is unsafe. Dementia is not a time of inactivity but an impairment of intellectual functioning.

A nurse is completing a history on a patient with role conflict. Which finding is consistent with role conflict? a. A patient is unsure about job expectations in a fast-paced company. b. A patient has to travel for work and misses children's birthdays. c. A patient feels less of a man after a leg amputation. d. A patient loses a job from the company's downsizing.

A patient has to travel for work and misses children's birthdays. rational: Role conflict results when a person has to simultaneously assume two or more roles that are inconsistent, contradictory, or mutually exclusive—for example, when a patient has to travel for work and misses children's birthdays. Role ambiguity is also common in employment situations. In complex, rapidly changing, or highly specialized organizations, employees often become unsure about job expectations. Feeling less of a man after a leg amputation is a body image and self-concept/self-esteem problem. Losing a job can lead to low self-esteem or loss of job identity.

The nurse is caring for a group of patients and is monitoring for sensory deprivation. Which patient will the nurse monitor most closely? a. A patient in the ICU under constant monitoring following a myocardial infarction b. A patient on the unit with tuberculosis on airborne precautions c. A patient who recently had a stroke and has left-sided weakness d. A patient receiving hospice care for end-stage lung cancer

A patient on the unit with tuberculosis on airborne precautions rational: A group at risk includes patients isolated in a health care setting or at home because of conditions such as active tuberculosis. Sensory deprivation occurs when a person has decreased stimulation and limited sensory input. A patient in isolation (airborne precautions) is at risk for sensory deprivation because of limited exposure to meaningful stimuli. A patient in the ICU would be at risk for sensory overload with all the monitors and visitors. A patient with a stroke may have difficulty with tactile sensation but is not at as high a risk for sensory deprivation as is one in isolation. A patient with lung cancer may have deficits, but hospice is present so the patient is at home with others.

The nurse is using the Snellen chart. Which patient is the nurse assessing? a. A patient who frequently reports the incorrect time from the clock across the room. b. A patient who is having difficulty remembering how to perform familiar tasks. c. A patient who turns the television up as loud as possible. d. A patient who has trouble saying words.

A patient who frequently reports the incorrect time from the clock across the room. rational: The Snellen chart is used to assess vision. Difficulty remembering how to perform familiar tasks indicates the need to further assess mental and cognitive status. Turning the television up louder indicates the need for a hearing assessment. For a patient having trouble saying words a picture board/chart may be used.

Which patient will cause the nurse to select a nursing diagnosis of Impaired physical mobility for a care plan? a. A patient who is completely immobile b. A patient who is not completely immobile c. A patient at risk for single-system involvement d. A patient who is at risk for multisystem problems

A patient who is not completely immobile rationale: The diagnosis of Impaired physical mobility applies to the patient who has some limitation but is not completely immobile. The diagnosis of Risk for disuse syndrome applies to the patient who is immobile and at risk for multisystem problems because of inactivity. Beyond these diagnoses, the list of potential diagnoses is extensive because immobility aӄects multiple body systems.

A nurse is caring for a group of patients. Which patient will the nurse see first? a. A patient who received morphine and has a pulse of 62 beats/min, respirations 10 breaths/min, and blood pressure 110/60 mm Hg b. A patient lying very still in bed who reports no pain but is pale with warm, dry skin c. A patient with severe pain who is nauseated and feels like he or she is about to vomit d. A patient writhing and moaning from abdominal pain after abdominal surgery

A patient who received morphine and has a pulse of 62 beats/min, respirations 10 breaths/min, and blood pressure 110/60 mm Hg rational: A respiratory rate of 10 indicates respiratory depression. A rare adverse effect of opioids in opioid-naïve patients (patients who have used opioids around the clock for less than approximately 1 week) is respiratory depression. Naloxone (Narcan) may be administered. While the other patients are experiencing pain and do need to be seen, they are not the priority since respirations are not affected.

A nurse is assigned most of the patients with pressure ulcers. The nurse leaves the pressure ulcer open to air and does not apply a dressing. To which patient did the nurse provide care? a. A patient with a clean Stage I b. A patient with a clean Stage II c. A patient with a clean Stage III d. A patient with a clean Stage IV

A patient with a clean Stage I rationale: Stage I intact pressure ulcers that resolve slowly without epidermal loss over 7 to 14 days do not require a dressing. A composite film, hydrocolloid, or hydrogel can be utilized on a clean Stage II. A hydrocolloid, hydrogel covered with foam, calcium alginate, and gauze can be utilized with a clean Stage III. Hydrogel covered with foam, calcium alginate, and gauze can be utilized with a clean Stage IV. An unstageable wound covered with eschar should utilize a dressing of adherent film or gauze with an ordered solution of enzymes.

The nurse is caring for a group of patients. Which patient will the nurse see first? a. A patient with chronic obstructive pulmonary disease doing stretching exercises b. A patient with diabetes mellitus carrying hard candy while doing exercises c. A patient with a heart attack doing isometric exercises d. A patient with hypertension doing Tai Chi exercises

A patient with a heart attack doing isometric exercises rationale: The nurse must see the myocardial infarction patient first to stop this type of exercise. It is important to understand the energy expenditure (increased respiratory rate and increased work on the heart) associated with isometric exercises because the exercises are sometimes contraindicated in certain patients' illnesses (e.g., myocardial infarction or chronic obstructive pulmonary disease). All the rest are appropriate. Stretching exercises are beneficial for patients with chronic obstructive pulmonary disease. Also instruct patients to perform low- to moderate-intensity exercises, carry a concentrated form of carbohydrates (sugar packets or hard candy), and wear a medical alert bracelet. The effect of a Tai Chi exercise program has demonstrated a significant reduction in systolic and diastolic blood pressures.

A nurse is providing care to a group of patients. Which patient will the nurse see first? a. A patient with a hip replacement on prolonged bed rest reporting chest pain and dyspnea b. A bedridden patient who has a reddened area on the buttocks who needs to be turned c. A patient on bed rest who has renal calculi and needs to go to the bathroom d. A patient after knee surgery who needs range of motion exercises

A patient with a hip replacement on prolonged bed rest reporting chest pain and dyspnea rationale: A patient on prolonged bed rest will be prone to deep vein thrombosis, which can lead to an embolus. An embolus can travel through the circulatory system to the lungs and impair circulation and oxygenation, resulting in tachycardia and shortness of breath. Venous emboli that travel to the lungs are sometimes life threatening. While the patient with a reddened area needs to be turned, a patient with renal calculi needing the restroom, and a patient needing range of motion, these are not as life threatening as the chest pain and dyspnea.

The nurse is caring for a group of patients. Which patient will the nurse see first? a. A patient with a Stage IV pressure ulcer b. A patient with a Braden Scale score of 18 c. A patient with appendicitis using a heating pad d. A patient with an incision that is approximated

A patient with appendicitis using a heating pad rationale: The nurse should see the patient with an appendicitis first. Warm applications are contraindicated when the patient has an acute, localized inflammation such as appendicitis because the heat could cause the appendix to rupture. Although a Stage IV pressure ulcer is deep, it is not as critical as the appendicitis patient. The total Braden score ranges from 6 to 23; a lower total score indicates a higher risk for pressure ulcer development. A score of 18 can be assessed later. A healing incision is approximated (closed); this is a normal finding and does not need to be seen first.

A nurse is supervising the logrolling of a patient. To which patient is the nurse most likely providing care? a. A patient with neck surgery b. A patient with hypostatic pneumonia c. A patient with a total knee replacement d. A patient with a Stage IV pressure ulcer

A patient with neck surgery rationale: A nurse supervises and aids personnel when there is a health care provider's order to logroll a patient. Patients who have suӄered from spinal cord injury or are recovering from neck, back, or spinal surgery often need to keep the spinal column in straight alignment to prevent further injury. Hypostatic pneumonia, total knee replacement, and Stage IV ulcers do not have to be logrolled.

Which of the following concepts are important to utilize when evaluating orders for restraints? (Select all that apply.) a. Behaviors that necessitate the use of restraint are part of the nursing plan of care. b. A physician's order is required for restraint and includes a face-to-face evaluation. c. The physician's preference for the format of the order can override agency policy. d. Orders are time limited. Restraints are not ordered prn (as needed). e. It should be specified that restraints are to be removed periodically. f. Restraint orders are time dated and signed by the physician.

A physician's order is required for restraint and includes a face-to-face evaluation. Orders are time limited. Restraints are not ordered prn (as needed). It should be specified that restraints are to be removed periodically. Restraint orders are time dated and signed by the physician. rationale: Physicians are responsible for writing restraint orders and conducting face-to-face evaluations, as well as for putting time limits, specifying when to remove, and time dating and signing orders. Behaviors that necessitate the use of restraint not only are part of the nursing documentation but are to be included as part of the order for restraint. The physician's formatting is not a consideration for evaluating restraint orders. Formatting of restraint orders typically follows state rules and regulations, as well as regulatory agency standards.

The nurse suspects the possibility of a bioterrorist attack. Which of the following factors is most likely related to this possibility? (Select all that apply.) a. A rapid increase in patients presenting with fever or respiratory or gastrointestinal symptoms b. Lower rates of symptoms among patients who spend time primarily indoors c. Large number of rapidly fatal cases of patients with presenting symptoms d. Shortage of personal protective equipment available from central supply e. An increase in the number of staff calling in sick for their assigned shift f. Patients with symptoms all coming from one location in the area

A rapid increase in patients presenting with fever or respiratory or gastrointestinal symptoms Lower rates of symptoms among patients who spend time primarily indoors Large number of rapidly fatal cases of patients with presenting symptoms Patients with symptoms all coming from one location in the area rationale: A rapid increase in patients presenting with a specific symptom, lower rates of symptoms among individuals indoors, and large numbers of fatalities with these symptoms all coming from one location are triggers that lead the nurse to suspect a bioterrorist attack. A shortage of personal protective equipment and an increase in the number of staff calling in sick can occur and does occur at times in the hospital setting and may have nothing to do with bioterrorism.

The nurse is caring for a patient with a spinal cord injury and notices that the patient's hips have a tendency to rotate externally when the patient is supine. Which device will the nurse use to help prevent injury secondary to this rotation? a. Hand rolls b. A trapeze bar c. A trochanter roll d. Hand-wrist splints

A trochanter roll rationale: A trochanter roll prevents external rotation of the hips when the patient is in a supine position. Hand rolls maintain the thumb in slight adduction and in opposition to the ӄngers. Hand-wrist splints are individually molded for the patient to maintain proper alignment of the thumb and the wrist. The trapeze bar is a triangular device that hangs down from a securely fastened overhead bar that is attached to the bedframe. It allows the patient to pull with the upper extremities to raise the trunk oӄ the bed, to assist in transfer from bed to wheelchair, or to perform upper arm exercises.

A nurse is caring for a young adult after surgery. Which action by the nurse will be priority? a. Allow involvement of peers b. Allow involvement of partner c. Allow involvement of volunteer activities d. Allow involvement of consistent schedule

Allow involvement of partner rationale: Nurses must understand that during hospitalization, a young adult's need for intimacy remains present; thus young adults benefit from the support of their partner or significant other during this time. Involvement of peers is priority for adolescents. Volunteer activities are priority for middle-aged adults. Consistent schedule is priority for infants and toddlers.

Hydrocolloid

Absorbs drainage through the use of exudate absorbers in the dressing

The patient reports being tired and weak and lacks energy. Upon assessment, the nurse finds that patient has gained weight, and blood pressure and pulse are elevated after climbing stairs. Which nursing diagnosis will the nurse add to the care plan? a. Fatigue b. Ineffective coping c. Activity intolerance d. Decreased cardiac output

Activity intolerance rationale: You consider nursing diagnoses of Activity intolerance or Fatigue in a patient who reports being tired and weak. Further review of assessed defining characteristics (e.g., abnormal heart rate and verbal report of weakness and the assessment findings occurring during the activity of climbing the stairs) leads to the definitive diagnosis (Activity intolerance). There is no data to support ineffective coping or decreased cardiac output.

The patient weighs 450 lbs (204.5 kg) and reports shortness of breath with any exertion. The health care provider has recommended beginning an exercise program. The patient states that she can hardly get out of bed and just cannot do anything around the house. Which nursing diagnosis will the nurse add to the care plan? a. Activity intolerance related to excessive weight b. Impaired physical mobility related to bed rest c. Imbalanced nutrition: less than body requirements d. Impaired gas exchange related to shortness of breath

Activity intolerance related to excessive weight rationale: In this case, activity intolerance is related to the patient's excessive weight. The patient is not on bed rest although claims that it is difficult to get out of bed, making this diagnosis inappropriate. Shortness of breath is a symptom, not a cause, of Impaired gas exchange, making this nursing diagnosis ineffective. The patient certainly has an imbalance of nutrition, but it is more than body requirements (obesity).

A nurse is caring for an older adult. Which goal is priority? a. Adjusting to career b. Adjusting to divorce c. Adjusting to retirement d. Adjusting to grandchildren

Adjusting to retirement rationale: Adjusting to retirement is one of the developmental tasks for an older person. A young or middle-aged adult has to adjust to career and/or divorce. A middle-aged adult has to adjust to grandchildren.

A nurse is assessing cognitive functioning of a patient. Which action will the nurse take? a. Administer a Mini-Mental State Examination (MMSE). b. Ask the patient to state name, location, and what month it is. c. Ask the patient's family if the patient is behaving normally. d. Administer the hearing handicap inventory for the elderly (HHIE-S).

Administer a Mini-Mental State Examination (MMSE). rational: The MMSE is a formal diagnostic tool that is used to assess a patient's level of cognitive functioning. The Mini-Mental State Examination (MMSE) is a tool you can use to measure disorientation, change in problem-solving abilities, and altered conceptualization and abstract thinking. Asking the patient orientation questions evaluates only the patient's orientation to self and surroundings, not abstract reasoning or critical thinking ability. Family members are not the most reliable source of information about the patient, although information received from the family should be considered. The HHIE-S is a 5- minute, 10-item questionnaire that assesses how the individual perceives the social and emotional effects of hearing loss. The higher the HHIE-S score, the greater the handicapping effect of a hearing impairment.

The nurse is caring for a group of patients. Which task may the nurse delegate to the nursing assistive personnel (NAP)? a. Administer a back massage to a patient with pain. b. Assessment of pain for a patient reporting abdominal pain. c. Administer patient-controlled analgesia for a postoperative patient. d. Assessment of vital signs in a patient receiving epidural analgesia.

Administer a back massage to a patient with pain. rational: A massage may be delegated to an NAP. Pain assessment is a nursing function and cannot be delegated to an NAP. Administration of patient-controlled analgesia (PCA) cannot be delegated to an NAP. Assessment of vital signs is a licensed nursing function; the NAP can take vital signs for a patient receiving epidural analgesia.

Cataplexy

Administer antidepressants

Insomnia

Administer benzodiazepine-like drugs

Narcolepsy

Administer modafinil (Provigel)

The nurse is presenting an educational session on safety for parents of adolescents. The nurse should include which of the following teaching points? a. Adolescents need unsupervised time with friends two to three times a week. b. Parents and friends should teach adolescents how to drive. c. Adolescents need information about the effects of beer on the liver. d. Adolescents need to be reminded to use seatbelts on long trips.

Adolescents need information about the effects of beer on the liver. rationale: Providing information about drugs and alcohol is important because adolescents may choose to participate in risk-taking behaviors. Adolescents need to socialize but need supervision. Parents can encourage and support learning processes associated with driving, but organized classes can help to decrease motor vehicle accidents. Seatbelts should be used all the time.

Which action should the nurse take when teaching a 5-year-old patient about a scheduled surgery? a. Do not discuss the procedure with the child to decrease anxiety. b. Let the child know the surgery will be at 9:00 AM in the morning. c. Insist that the parents wait outside the room to ensure privacy of the child. d. Allow the child to touch and hold medical equipment such as thermometers.

Allow the child to touch and hold medical equipment such as thermometers. rationale: Nursing interventions during the preoperational period (ages 2 to 7 years) should recognize the use of play (such as handling equipment) to help the child understand the events taking place. The nurse should talk to the child about the procedure in terms the child can understand. Children at this stage have difficulty conceptualizing time; telling the child surgery is at 9:00 AM in the morning is inappropriate. Parents should be allowed in the room.

A nurse is caring for an adult patient who has had a minor motor vehicle accident. The health history reveals that the patient is currently in the process of obtaining a divorce. Which of the following actions should the nurse take? (Select all that apply.) a. Agree upon and make time for the patient to talk. b. Use active listening skills and therapeutic touch as appropriate. c. Teach stress reduction strategies. d. Inform patient that stressed individuals are more likely to have accidents. e. Agree to witness telephone conversations with separated husband. f. Refer the patient to the nurse's church marriage counselor.

Agree upon and make time for the patient to talk. Use active listening skills and therapeutic touch as appropriate. Teach stress reduction strategies. Inform patient that stressed individuals are more likely to have accidents. rationale: Agreeing and making time for conversation, using active listening skills and therapeutic touch, teaching stress reduction strategies, and informing the patient of the risk to health associated with stress are interventions that are within the nurse's scope of practice. Agreeing to witness a telephone conversation could draw the nurse into divorce proceedings when the focus should be on the patient and his health. Referring the patient to the nurse's church counselor without a specific request from the patient may not take into consideration cultural care and could be considered unprofessional. If the patient requested a marriage counselor, a better solution would be to provide a referral to social services that may include a list of possible counselors from which the patient could choose.

The nurse is caring for a patient who has a Stage IV pressure ulcer with grafted surgical sites. Which specialty bed will the nurse use for this patient? a. Low-air-loss b. Air-fluidized c. Lateral rotation d. Standard mattress

Air-fluidized rationale: For a patient with newly flapped or grafted surgical sites, the air-fluidized bed will be the best choice; this uses air and fluid support to provide pressure redistribution via a fluid-like medium created by forcing air through beads as characterized by immersion and envelopment. A low-air-loss bed is utilized for prevention or treatment of skin breakdown by preventing buildup of moisture and skin breakdown through the use of airflow. A standard mattress is utilized for an individual who does not have actual or potential altered or impaired skin integrity. Lateral rotation is used for treatment and prevention of pulmonary, venous stasis and urinary complications associated with mobility.

A patient has developed a pressure ulcer. Which laboratory data will be important for the nurse to check? a. Vitamin E b. Potassium c. Albumin d. Sodium

Albumin rationale: Normal wound healing requires proper nutrition. Serum proteins are biochemical indicators of malnutrition, and serum albumin is probably the most frequently measured of these parameters. The best measurement of nutritional status is prealbumin because it reflects not only what the patient has ingested but also what the body has absorbed, digested, and metabolized. Zinc and copper are the minerals important for wound healing, not potassium and sodium. Vitamins A and C are important for wound healing, not vitamin E.

The home health nurse is caring for a patient in the home who is using an electrical infusion device. While visiting the patient, the nurse smells smoke and notices an electrical fire started by this device. The nurse uses the fire extinguisher and fights the fire when (Select all that apply.) a. All occupants have left the home. b. Fire department has been called. c. Fire is confined to one room. d. An exit route is available. e. The correct extinguisher is available. f. The nurse thinks she can use the fire extinguisher.

All occupants have left the home. Fire department has been called. An exit route is available. The correct extinguisher is available. rationale: In a home setting, if the nurse is present during a fire, she first should remove all occupants and then should call the fire department by dialing 911. If the fire is small— not confined to just one room (this could be too large for the fire extinguisher), if the correct extinguisher is available, and if the nurse knows (not thinks) that she can use it, the nurse may attempt to extinguish the fire. Utilize PASS (Pull the pin, Aim low, Squeeze the handles, Sweep area from side to side) to activate the extinguisher.

In preparation for the eventual death of a female hospice patient of the Muslim faith, the nurse organizes a meeting of all hospice caregivers. A plan of care to be followed when this patient dies is prepared. Which information will be included in the plan? a. Prepare the body for autopsy. b. Prepare the body for cremation. c. Allow male Muslims to care for the body after death has occurred. d. Allow female Muslims to care for the body after death has occurred.

Allow female Muslims to care for the body after death has occurred rationale: Muslims of the same gender prepare the body for burial. Muslim faith discourages cremation and autopsy to preserve the sanctity of the soul of the deceased and promote burial as soon as possible after death

A nurse is caring for an immobile patient. Which metabolic alteration will the nurse monitor for in this patient? a. Increased appetite b. Increased diarrhea c. Increased metabolic rate d. Altered nutrient metabolism

Altered nutrient metabolism rationale: Immobility disrupts normal metabolic functioning: decreasing the metabolic rate, altering the metabolism of carbohydrates, fats, and proteins; causing ӄuid, electrolyte, and calcium imbalances; and causing gastrointestinal disturbances such as decreased appetite and slowing of peristalsis, leading to constipation.

Which patient is most in need of a nurse's referral to adoption services? a. A woman considering abortion for an unwanted pregnancy b. An infertile couple religiously opposed to artificial insemination c. A woman who suffered miscarriage during her first pregnancy d. An infertile couple who has been attempting conception for 3 months

An infertile couple religiously opposed to artificial insemination rational: Adoption is an option for someone with infertility, especially if infertility treatments are unavailable owing to religious or financial constraints. A patient who wishes to have an elective abortion may be educated about all the possibilities, but the nurse should approach the patient in a nonjudgmental manner and should accept the patient's decision. When a patient has recently miscarried, the nurse should assess the patient's feelings about the loss and should address any concerns the patient may have about fertility. Infertility is the inability to conceive after 1 year of unprotected intercourse; therefore, talking about adoption after one miscarriage or after only 3 months of attempting conception would be too soon.

During a follow-up visit, a female patient is describing new onset of marital discord with her terminally ill spouse to the hospice nurse. Which Kübler-Ross stage of dying is the patient experiencing? a. Denial b. Anger c. Bargaining d. Depression

Anger rationale: Kübler-Ross's traditional theory involves five stages of dying. The anger stage of adjustment to an impending death can involve resistance, anger at God, anger at people, and anger at the situation. Denial would involve failure to accept a death. Bargaining is an action to delay acceptance of death by bartering. Depression would present as withdrawal from others.

The nurse is discussing lack of sleep with a middle-aged adult. Which area should the nurse most likely assess to determine a possible cause of the lack of sleep? a. Anxiety b. Loud teenagers c. Caring for pets d. Late night television

Anxiety rationale: During middle adulthood, the total time spent sleeping at night begins to decline. Anxiety, depression, and certain physical illnesses can affect sleep, and women can experience menopausal symptoms. Insomnia is common because of the changes and stresses associated with middle age. Teenagers, caring for pets, and late night television can influence the amount of sleep; however, these are not the most common causes of insomnia in this age group

The nurse is caring for a patient to ease modifiable factors that contribute to pain. Which areas did the nurse focus on with this patient? a. Age and gender b. Anxiety and fear c. Culture and ethnicity d. Previous pain experiences and cognitive abilities

Anxiety and fear rational: Some examples of modifiable contributors to pain are anxiety and fear. The nurse can take measures to ease the patient's anxiety and fear related to pain. Age, gender, culture, ethnicity, cognitive abilities, and previous pain experience are all nonmodifiable factors that the nurse can help the patient to understand, but the nurse cannot alter them.

A nurse is developing a plan of care concerning growth and development for a hospitalized adolescent. What should the nurse do? (Select all that apply.) a. Apply developmental theories when making observations of the adolescent's patterns of growth and development. b. Compare the adolescent's assessment findings versus normal findings. c. Recognize her own (the nurse's) moral developmental level. d. Stick with one developmental theory for consistency. e. Apply a unidimensional life span perspective.

Apply developmental theories when making observations of the adolescent's patterns of growth and development. Compare the adolescent's assessment findings versus normal findings. Recognize her own (the nurse's) moral developmental level. rationale: Today's nurses need to be knowledgeable about several theoretical perspectives when working with patients. These theories form the basis for meaningful observation of an individual's pattern of growth and development. They provide important guidelines for nurses to recognize deviations from the norm. Recognizing your own moral developmental level is essential in separating your own beliefs from those of others when helping patients with their moral decision-making process. No one theory successfully describes all the intricacies of human growth and development. Growth and development, as supported by a life span perspective, is multidimensional, not unidimensional.

The nurse is caring for a postoperative medial meniscus repair of the right knee. Which action should the nurse take to assist with pain management? a. Monitor vital signs every 15 minutes. b. Check pulses in the right foot. c. Keep the leg dependent. d. Apply ice.

Apply ice rationale: Ice assists in preventing edema formation, controlling bleeding, and anesthetizing the body part. Elevation (not dependent) assists in preventing edema, which in turn can cause pain. Monitoring vital signs every 15 minutes is routine postoperative care and includes a pain assessment but in itself is not an intervention that decreases pain. Checking the pulses is important to monitor the circulation of the extremity but in itself is not a pain management intervention.

The nurse is caring for a group of patients. Which task can the nurse delegate to the nursing assistive personnel? a. Assessing a surgical patient for risk of pressure ulcers b. Applying an elastic bandage to a medical-surgical patient c. Treating a pressure ulcer on the buttocks of a medical patient d. Implementing negative-pressure wound therapy on a stable patient

Applying an elastic bandage to a medical-surgical patient rationale: Applying an elastic bandage to a medical-surgical patient can be delegated to the nursing assistive personnel (NAP). Assessing pressure ulcer risk, treating a pressure ulcer, and implementing negative-pressure wound therapy cannot be delegated to an NAP.

The nurse is admitting a patient to the hospital. The patient states that he is a very spiritual person but does not practice any specific religion. The nurse understands that these statements a. Are contradictory. b. Indicate a strong religious affiliation. c. Indicate a lack of faith. d. Are reasonable.

Are reasonable. rational: These statements are reasonable and are not contradictory. Many people tend to use the terms spirituality and religion interchangeably. Although closely associated, these terms are not synonymous. Religious practices encompass spirituality, but spirituality does not need to include religious practice. When a person has the attitude of something to live for and look forward to, hope is present.

The nurse is caring for a patient who has had a stroke causing total paralysis of the right side. To help maintain joint function and minimize the disability from contractures, passive ROM will be initiated. When should the nurse begin this therapy? a. After the acute phase of the disease has passed b. As soon as the ability to move is lost c. Once the patient enters the rehab unit d. When the patient requests it

As soon as the ability to move is lost rationale: Passive ROM exercises should begin as soon as the patient's ability to move the extremity or joint is lost. The nurse should not wait for the acute phase to end. It may be some time before the patient enters the rehab unit or the patient requests it, and contractures could form by then.

A nurse receives an order from a health care provider to administer hydrocodone and acetaminophen (Vicodin ES 7.5/750), to a patient who is experiencing 8/10 postsurgical pain. The order is to give 2 tablets every 6 hours by mouth as needed for pain. What is the nurse's next best action? a. Give the Vicodin ES to the patient immediately because the patient is experiencing severe pain. b. Ask the health care provider for a nonsteroidal antiinflammatory drug (NSAID) order. c. Ask the health care provider to verify the dosage and frequency of the medication. d. Give the Vicodin ES in addition to playing soothing music for the patient.

Ask the health care provider to verify the dosage and frequency of the medication. rational: The maximum 24-hour dosage for acetaminophen is 4 grams. If the patient took 2 tablets of Vicodin ES every 6 hours, the patient would take in 6 grams of acetaminophen in 24 hours (2 tablets = 750 + 750 = 1500 4 [could have 4 doses in 24 hours every 6 hours] = 6000 mg = 6 g). This exceeds the safe dosage of acetaminophen, so the best action is to question this order. Giving the medication as ordered would possibly result in the patient's taking more acetaminophen than is considered a safe dose. Acetaminophen overdose can result in liver failure. NSAIDs are used to treat mild to moderate pain. At this moment, the patient is experiencing severe pain. Implementing music therapy is a nursing intervention and is an independent nursing action that can be instituted with pain medication, but the possible acetaminophen dose is the priority.

.An oriented patient has recently had surgery. Which action is best for the nurse to take to assess this patient's pain? a. Assess the patient's body language. b. Ask the patient to rate the level of pain. c. Observe the cardiac monitor for increased heart rate. d. Have the patient describe the effect of pain on the ability to cope.

Ask the patient to rate the level of pain. rationale: One of the most subjective and therefore most useful characteristics for reporting pain is its severity. Therefore, the best way to assess a patient's pain is to ask the patient to rate the pain. Nonverbal communication, such as body language, is not as effective in assessing pain, especially when the patient is oriented. Heart rate sometimes increases when a patient is in pain, but this is not a symptom that is specific to pain. Pain some-times affects a patient's ability to cope, but assessing the effect of pain on coping assesses the patient's ability to cope; it does not assess the patient's pain.

A 15-year-old patient is concerned because her mother wants her to receive the human papillomavirus (HPV) vaccination, but the patient is unsure if she wants it. Which response by the nurse is most therapeutic? a. Ask the patient what concerns she may have about the vaccination. b. Inquire about how many sexual partners she has had in the past year. c. Remind her that her mother knows best and that she should respect her parents' wishes. d. Promote the importance of the vaccine, and recommend that the patient get the vaccine as soon as possible.

Ask the patient what concerns she may have about the vaccination. rational: The nurse should encourage health promotion behaviors but first must consider the autonomy of the patient and assess the patient for more data. The nurse should value the input of the patient in making a decision and assess what the patient is thinking to address any concerns the patient may have. The HPV vaccine is a preventative treatment; whether or not the patient is sexually active (asking about how many sexual partners) does not matter in this case. The nurse should not make assumptions about a patient's home life (mother knows best); instead, the nurse should ask questions while establishing a therapeutic relationship. Recommending the patient get the vaccine as soon as possible is in violation of the patient's rights.

When the nurse views the family as context, the primary focus is on the health and development of an individual member existing within a specific environment (i.e., the patient's family). Although the focus is on the individual's health status, the nurse should a. Assess how much the family provides the patient's basic needs. b. Assess family patterns versus individual characteristics. c. Maintain distinctions between "family as patient" and "family as context." d. Plan care to meet not only the patient's needs, but those of his family as well.

Assess how much the family provides the patient's basic needs. rational: When the nurse views the family as context, the primary focus is on the health and development of an individual member existing within a specific environment (i.e., the patient's family). Although the focus is on the individual's health status, the nurse assesses how much the family provides the individual's basic needs. Family patterns are in the realm of "family as patient." It is important to understand that although the nurse is able to make theoretical and practical distinctions between "family as context" and "family as patient," they are not necessarily mutually exclusive. Often, the nurse will use the two simultaneously, as with the perspective of "family as system." "Family as patient" involves planning to meet the needs of the patient and those of his family as well.

The nurse is interviewing a patient who is being admitted to the hospital. The patient's family went home before the nurse's interview. The nurse asks the patient, "Who decides where to go on vacation?" In asking this, what is the nurse trying to do? a. Assess the family structure. b. Assess the family form. c. Assess the family function. d. Make a categorical generalization.

Assess the family structure. rational: To assess the family structure, the nurse asks questions that determine the power structure and patterning of roles and tasks (e.g., "Who decides where to go on vacation?"). When focusing on family form, the nurse should begin the family assessment by determining the patient's definition of family. Family function is the ability of the family to provide emotional support and to cope with health problems or situations. The question asked above will not assess that. It is imperative to remember that categorical generalizations are misleading and should be avoided.

The nurse is completing an admission history on a new home health patient. The patient has been experiencing seizures as the result of a recent brain injury. The nurse diagnoses risk for injury with a goal of keeping the patient safe in the event of a seizure. Which interventions should the nurse utilize for this patient? (Select all that apply.) a. Teach the family how to insert an oral airway during the seizure. b. Assess the home for items that could harm the patient during a seizure. c. Provide information on how to obtain a Medical Alert bracelet. d. Teach the patient to communicate to the caregiver plans for bathing. e. Discuss with family steps to take if the seizure does not discontinue. f. Demonstrate how to restrain the patient in the event of a seizure.

Assess the home for items that could harm the patient during a seizure. Provide information on how to obtain a Medical Alert bracelet. Teach the patient to communicate to the caregiver plans for bathing. Discuss with family steps to take if the seizure does not discontinue. rationale: Assessment of the home for safety, providing information on Medical Alert bracelets, teaching the patient to communicate before bathing, and discussing steps to take with status epilepticus are important interventions for the patient who is having seizures. Inserting an airway may harm the patient by forcing the object into the mouth or by biting down on a hard object. Never restrain a patient who is having a seizure, but protect the patient from hitting his body on objects around him to prevent traumatic injury.

During a routine physical assessment, the nurse obtaining a health history notes that a 50-year-old female patient reports pain and redness in the right breast. Which action is best for the nurse to take in response to this finding? a. Assess the patient as thoroughly as possible. b. Explain to the patient that breast tenderness is normal at her age. c. Tell the patient that redness is not a cause for concern and is quite common. d. Inform her that redness is the precursor to normal unilateral breast enlargement.

Assess the patient as thoroughly as possible. rationale: A comprehensive assessment offers direction for health promotion recommendations, as well as for planning and implementing any acutely needed intervention. Redness or painful breasts are abnormal physical assessment findings in the middle-aged adult. Increased size of one breast is an abnormal physical assessment finding in the middle-aged adult.

The nurse is caring for a patient who suddenly becomes confused and tries to remove an intravenous infusion. The nurse begins to develop a plan to care for the patient. Which nursing intervention should take priority? a. Gather restraint supplies. b. Try alternatives to restraint. c. Assess the patient. d. Call the physician for a restraint order.

Assess the patient. When a patient becomes suddenly confused, the priority is to assess the patient, including checking laboratory test and oxygen status and treating and eliminating the cause of the change in mental status. If interventions and alternatives are exhausted, the nurse working with the physician may determine the need for restraints.

The nurse is caring for a patient who has been diagnosed with a terminal illness. The patient states, "I just don't feel like going to work. I have no energy, and I can't eat or sleep." The patient shows no interest in taking part in his care. The nurse should a. Not be concerned about self-harm because the patient has not indicated any desire toward suicide. b. Ignore individual patient goals until the current crisis is over. c. Encourage the patient to purchase over-the-counter sleep aids to help him sleep. d. Assess the potential for suicide and make appropriate referrals.

Assess the potential for suicide and make appropriate referrals. rationale: A decreased appetite and level of energy and not wanting to be involved in care are signs of hopelessness. The nurse should assess for risk of the patient harming himself or others. The nurse should set goals that are important to the patient. Recommending good sleep hygiene habits is more appropriate than giving over-the-counter sleep aids.

11. The nurse is preparing to lift a patient. Which action will the nurse take first? a. Position a drawsheet under the patient. b. Assess weight and determine assistance needs. c. Delegate the task to a nursing assistive personnel. d. Attempt to manually lift the patient alone before asking for assistance.

Assess weight and determine assistance needs. rationale: When lifting, assess the weight you will lift, and determine the assistance you will need. The nurse has to assess before positioning a drawsheet or delegating the task. Manual lifting is the last resort, and it is used when the task at hand does not involve lifting most or all of the patient's weight; most facilities have a no-lift policy

The patient has the nursing diagnosis of Impaired physical mobility related to pain in the left shoulder. Which priority action will the nurse take? a. Encourage the patient to do self-care. b. Keep the patient as mobile as possible. c. Encourage the patient to perform ROM. d. Assist the patient with comfort measures.

Assist the patient with comfort measures. rationale: The diagnosis related to pain requires the nurse to assist the patient with comfort measures so that the patient is then willing and more able to move. Pain must be controlled so the patient will not be reluctant to initiate movement. The diagnosis related to reluctance to initiate movement requires interventions aimed at keeping the patient as mobile as possible and encouraging the patient to perform self-care and ROM.

The patient is admitted to a skilled care unit for rehabilitation after the surgical procedure of fixation of a fractured left hip. The patient's nursing diagnosis is Impaired physical mobility related to musculoskeletal impairment from surgery and pain with movement. The patient is able to use a walker but needs assistance ambulating and transferring from the bed to the chair. Which nursing intervention is most appropriate for this patient? a. Obtain assistance and physically transfer the patient to the chair. b. Assist with ambulation and measure how far the patient walks. c. Give pain medication after ambulation so the patient will have a clear mind. d. Bring the patient to the cafeteria for group instruction on ambulation.

Assist with ambulation and measure how far the patient walks. rationale: Assist with walking and measure how far the patient walks to quantify progress. The nurse should allow the patient to do as much for self as possible. Therefore, the nurse should observe the patient transferring from the bed to the chair using the walker and should provide assistance as needed. The patient should be encouraged to use adequate pain medication to decrease the eӄects of pain and to increase mobility. The patient should be instructed on safe transfer and ambulation techniques in an environment with few distractions, not in the cafeteria

An 18-month-old patient is brought into the clinic for evaluation because the parent is concerned. The 18-month-old child hits siblings and says only "No" when communicating verbally. Which recommendation by the nurse will be best for this situation? a. Assure the mother that the child is developmentally within specified norms. b. Encourage the mother to seek psychological counseling for the child. c. Consult the social worker because the child is hitting other children. d. Remove all toys from the child's room until this behavior ceases.

Assure the mother that the child is developmentally within specified norms rationale: Assure the mother that the child is displaying normal behavior. At 18 months, the child is in the sensorimotor period of development. Piaget describes hitting, looking, grasping, and kicking as normal schemas to deal with the environment. The social worker does not need to be consulted in this case nor is psychological counseling warranted, because the child is exhibiting normal behaviors. Play is an important part of all children's development. Removing toys and the opportunity to play with them may actually hinder the child's development.

The patient is being admitted to the neurological unit with a diagnosis of stroke. When will the nurse begin discharge planning? a. At the time of admission b. The day before the patient is to be discharged c. When outpatient therapy will no longer be needed d. As soon as the patient's discharge destination is known

At the time of admission rationale: Discharge planning begins when a patient enters the health care system. In anticipation of the patient's discharge from an institution, the nurse makes appropriate referrals or consults a case manager or a discharge planner to ensure that the patient's needs are met at home. Referrals to home care or outpatient therapy are often needed. Planning the day before discharge, when outpatient therapy is no longer needed, and as soon as the discharge destination is known is too late.

The nurse is assessing the patient for respiratory complications of immobility. Which action will the nurse take when assessing the respiratory system? a. Inspect chest wall movements primarily during the expiratory cycle. b. Auscultate the entire lung region to assess lung sounds. c. Focus auscultation on the upper lung fields. d. Assess the patient at least every 4 hours.

Auscultate the entire lung region to assess lung sounds. rationale: Auscultate the entire lung region to identify diminished breath sounds, crackles, or wheezes. Perform a respiratory assessment at least every 2 hours for patients with restricted activity. Inspect chest wall movements during the full inspiratory-expiratory cycle. Focus auscultation on the dependent lung ӄelds because pulmonary secretions tend to collect in these lower regions.

The word spirituality derives from the Latin word spiritus, which refers to breath or wind. Today, spirituality is a. Awareness of one's inner self and a sense of connection to a higher being. b. Less important than coping with the patient's illness. c. Patient centered and has no bearing on the nurse's belief patterns. d. Equated to formal religious practice and has a minor effect on health care.

Awareness of one's inner self and a sense of connection to a higher being. rational: Today, spirituality is often defined as an awareness of one's inner self and a sense of connection to a higher being, to nature, or to some purpose greater than oneself. Spirituality is an important factor that helps individuals achieve the balance needed to maintain health and well-being and to cope with illness. It positively affects and enhances health, quality of life, health promotion behaviors, and disease prevention activities. Nurses need an awareness of their own spirituality to provide appropriate and relevant spiritual care to others. The concepts of spirituality and religion are often interchanged, but spirituality is a much broader and more unifying concept than religion. The human spirit is powerful, and spirituality has different meanings for different people.

The nurse is working on an orthopedic rehabilitation unit that requires lifting and positioning of patients. Which personal injury will the nurse most likely try to prevent? a. Arm b. Hip c. Back d. Ankle

Back rationale: Back injuries are often the direct result of improper lifting and bending. The most common back injury is strain on the lumbar muscle group. While arm, hip, and ankle can occur, they are not as common as back.

A nurse is assessing a patient with activity intolerance for possible orthostatic hypotension. Which finding will help confirm orthostatic hypotension? a. Blood pressure sitting 120/64; blood pressure 140/70 standing b. Blood pressure sitting 126/64; blood pressure 120/58 standing c. Blood pressure sitting 130/60; blood pressure 110/60 standing d. Blood pressure sitting 140/60; blood pressure 130/54 standing

Blood pressure sitting 130/60; blood pressure 110/60 standing rationale: Orthostatic hypotension results in a drop of 20 mm Hg systolic or more in blood pressure when rising from sitting position (110/60). 120 to 140 means the blood pressure increased rather than dropped. 126 to 120 is only a six points' difference. 140 to 130 is only a 10 points' difference.

A verbally abusive partner has told a significant other many negative comments over the years. In the crisis center, the nurse would anticipate that the patient may have which self-concept deficits? a. Body image b. Role confusion c. Rigidity d. Yearning

Body image rational: Over the years of marriage, the significant other incorporates this devaluation into his or her own self-concept, negatively affecting body image. The way others view a person's body and the feedback offered are also influential on body image and self-concept. Role confusion is part of a developmental task (identity versus role confusion) for adolescents. Rigidity and yearning are not components of self-concept.

A nurse is using different strategies to meet older patients' psychosocial needs. Match the strategy the nurse is using to its description. a. Respecting the older adult's uniqueness b. Improving level of awareness c. Listening to the patient's past recollections d. Accepting describing of patient's perspective e. Offering help with grooming and hygiene 1. Body image 2. Validation therapy 3. Therapeutic communication 4. Reality orientation 5. Reminiscence

Body image- offering help with grooming and hygiene Validation therapy- accepting describing of patient's perspective Therapeutic communication- respecting the older adult's uniqueness Reality orientation- Improving level of awareness Reminiscence- listening to the patient's past recollections

The nurse is evaluating the body alignment of a patient in the sitting position. Which observation by the nurse will indicate a normal finding? a. The edge of the seat is in contact with the popliteal space. b. Both feet are supported on the ӄoor with ankles flexed. c. The body weight is directly on the buttocks only. d. The arms hang comfortably at the sides.

Both feet are supported on the ӄoor with ankles flexed. rationale: Both feet are supported on the ӄoor, and the ankles are comfortably ӄexed. Body weight is evenly distributed on the buttocks and thighs. A 1- to 2-inch space is maintained between the edge of the seat and the popliteal space on the posterior surface of the knee to ensure that no pressure is placed on the popliteal artery or nerve. The patient's forearms are supported on the armrest, in the lap, or on a table in front of the chair.

A patient's father died a week ago. Both the patient and the patient's spouse talk about the death. The patient's spouse is experiencing headaches and fatigue. The patient is having trouble sleeping, has no appetite, and gets choked up most of the time. How should the nurse interpret these findings as the basis for a follow-up assessment? a. The patient is dying and the spouse is angry. b. The patient is ill and the spouse is malingering. c. Both the patient and the spouse are likely in denial. d. Both the patient and the spouse are likely grieving.

Both the patient and the spouse are likely grieving. rationale: Both are likely grieving from the loss of the patient's father. Symptoms of normal grief include headache, fatigue, insomnia, appetite disturbance, and choking sensation. Different people manifest different symptoms. There is no data to support the spouse is angry or malingering. There is no data to support the patient is dying or ill. Denial is assessed when the person cannot accept the loss; both talked about the loss.

The nurse is caring for a hospitalized young-adult male who works as a dishwasher at a local restaurant. He states that he would like to get a better job but has no education. How can the nurse best assist this patient psychosocially? a. By providing information and referrals b. By focusing on the patient's medical diagnoses c. By telling the patient that he needs to go back to school d. By expecting the patient to be flexible in decision making

By providing information and referrals rationale: Support from the nurse, access to information, and appropriate referrals provide opportunities for achievement of a patient's potential. Health is not merely the absence of disease (focusing on medical diagnoses) but involves wellness in all human dimensions. Telling a patient what to do (go back to school) is inappropriate. Each person (not the nurse) needs to make these decisions based on individual factors. Insecure persons tend to be more rigid in making decisions.

The nurse is caring for a patient who has a wound drain with a collection device. The nurse notices that the collection device has a sudden decrease in drainage. Which action will the nurse take next? a. Call the health care provider; a blockage is present in the tubing. b. Chart the results on the intake and output flow sheet. c. Do nothing, as long as the evacuator is compressed. d. Remove the drain; a drain is no longer needed.

Call the health care provider; a blockage is present in the tubing rationale: Because a drainage system needs to be patent, look for drainage flow through the tubing, as well as around the tubing. A sudden decrease in drainage through the tubing may indicate a blocked drain, and you will need to notify the health care provider. The health care provider, not the nurse, determines the need for drain removal and removes drains. Charting the results on the intake and output flow sheet does not take care of the problem. The evacuator may be compressed even when a blockage is present.

When focusing on older adults, the nurse must be aware that a. Elder abuse happens in lower socioeconomic classes only. b. Elders have the same social networks as younger people. c. Caregivers may be spouses or middle-age children. d. Caregiver stress is minimal when caring for a parent.

Caregivers may be spouses or middle-age children. rational: Caregivers are typically spouses, who may be older adults with declining physical stamina, or middle-age children, who often have other responsibilities. Abuse of older adults in families occurs across all social classes. Later-life families have a different social network than younger families because friends and same-generation family members often have died or have been ill themselves. The nurse should assess for caregiver stress such as tension in relationships between family and care recipient, changes in level of health, changes in mood, and anxiety and depression.

A nurse determines that a middle-aged patient is a typical example of the "sandwich generation." What did the nurse discover the patient is caught between? a. Job responsibilities or family responsibilities b. Stopping old habits and starting new ones c. Caring for children and aging parents d. Advancing in career or retiring

Caring for children and aging parents rationale: Middle-aged adults also begin to help aging parents while being responsible for their own children, placing them in the sandwich generation. It does not include job and family responsibilities; old habits and new ones; or career and retiring.

The nurse is caring for a patient who has not been able to sleep well while in the hospital, leading to a disrupted sleep-wake cycle. Which assessment findings will the nurse monitor for in this patient? (Select all that apply.) a. Changes in physiological function such as temperature b. Decreased appetite and weight loss c. Anxiety, irritability, and restlessness d. Shortness of breath and chest pain e. Nausea, vomiting, and diarrhea f. Impaired judgment

Changes in physiological function such as temperature Decreased appetite and weight loss Anxiety, irritability, and restlessness Impaired judgment rationale: The biological rhythm of sleep frequently becomes synchronized with other body functions. Changes in body temperature correlate with sleep pattern. When the sleep-wake cycle becomes disrupted, changes in physiological function such as temperature can occur. Patients can experience decreased appetite, loss of weight, anxiety, restlessness, irritability, and impaired judgment. Gastrointestinal and respiratory/cardiovascular symptoms such as shortness of breath and chest pain are not symptoms of a disrupted sleep cycle

Social scientists have identified four threats facing the family in today's American society. Of the choices below, what are the four threats? (Select all that apply.) a. Changing economic status b. Homelessness c. Family violence d. Rise of homosexual families e. Presence of illness

Changing economic status Homelessness Family violence Presence of illness rational: Social scientists have identified four trends as threats facing the family. These include (1) changing economic status, (2) homelessness, (3) family violence, and (4) the presence of acute or chronic illness. Many homosexual couples define their relationship in family terms, but homosexual families are not listed as a threat facing the family.

5. A nurse is preparing to reposition a patient. Which task can the nurse delegate to the nursing assistive personnel? a. Determining the level of comfort b. Changing the patient's position c. Identifying immobility hazards d. Assessing circulation

Changing the patient's position rationale: The skill of moving and positioning patients in bed can be delegated to nursing assistive personnel (NAP). The nurse is responsible for assessing the patient's level of comfort and for any hazards of immobility and assessing circulation.

The nurse has been called to a hospital room where a patient is using a hair dryer from home. The patient has received an electrical shock from the dryer. The patient is unconscious and is not breathing. What is the best next step? a. Ask the family to leave the room. b. Check for a pulse. c. Begin compressions. d. Defibrillate the patient.

Check for a pulse rationale: In this scenario, the patient is in a hospital setting, and it has been determined that the patient is not conscious and is not breathing. The next step is to check the pulse. An electrical shock can interfere with the heart's normal electrical impulses and can cause arrhythmias. Checking the pulse helps to determine the need for cardiopulmonary resuscitation (CPR) and defibrillation.

A nurse is caring for a patient in the last stages of dying. Which finding indicates the nurse needs to prepare the family for death? a. Redness of skin b. Clear-colored urine c. Tense muscles tone d. Cheyne-Stokes breathing

Cheyne-Stokes breathing rationale: Altered breathing such as Cheyne-Stokes pattern, apnea, labored, or irregular breathing is a sign of impending death. Cyanotic, pallor, or mottling of skin occurs. Urine is decreased and a dark color. Decreased muscle tone, relaxed jaw muscles, and sagging mouth also occur.

The nurse is cleansing a wound site. As the nurse administers the procedure, which intervention should be included? a. Allow the solution to flow from the most contaminated to the least contaminated. b. Scrub vigorously when applying noncytotoxic solution to the skin. c. Cleanse in a direction from the least contaminated area. d. Utilize clean gauze and clean gloves to cleanse a site.

Cleanse in a direction from the least contaminated area. rationale: Cleanse in a direction from the least contaminated area, such as from the wound or incision, to the surrounding skin. While cleansing surgical or traumatic wounds by applying noncytotoxic solution with sterile gauze or by irrigations is correct, vigorous scrubbing is inappropriate and can cause damage to the skin. Use gentle friction when applying solutions to the skin, and allow irrigation to flow from the least to the most contaminated area.

The nurse is caring for a group of medical-surgical patients. The unit has been notified of a fire on an adjacent wing of the hospital. The nurse quickly formulates a plan to keep the patients safe. Which of the following should the nurse implement? (Select all that apply.) a. Close all doors. b. Note evacuation routes. c. Note oxygen shut-offs. d. Await direction from the fire department. e. Evacuate everyone from the building. f. Review "Stop, drop, and roll" with the nursing staff.

Close all doors. Note evacuation routes. Note oxygen shut-offs Await direction from the fire department. rationale: Closing all doors helps to contain smoke and fire. Noting the evacuation routes and oxygen shut-offs is important in case the direction to evacuate comes from established channels. Evacuation from the building is determined by the established chain of command or the fire department. Evacuation is done only when necessary. Review of "stop, drop, and roll," although important, is not a priority at this time.

A nurse is evaluating care of an immobilized patient. Which action will the nurse take? a. Focus on whether the interdisciplinary team is satisӄed with the care. b. Compare the patient's actual outcomes with the outcomes in the care plan. c. Involve primarily the patient's family and health care team to determine goal achievement. d. Use objective data solely in determining whether interventions have been successful.

Compare the patient's actual outcomes with the outcomes in the care plan. rationale: From your perspective as the nurse, you are to evaluate outcomes and response to nursing care and compare the patient's actual outcomes with the outcomes selected during planning. Ask if the patient's expectations (subjective data) of care are being met, and use objective data to determine the success of interventions. Just as it was important to include the patient during the assessment and planning phase of the care plan, it is essential to have the patient's evaluation of the plan of care, not just the patient's family and health care team.

The nurse is caring for a patient with a healing Stage III pressure ulcer. Upon entering the room, the nurse notices an odor and observes a purulent discharge, along with increased redness at the wound site. What is the next best step for the nurse? a. Complete the head-to-toe assessment, including current treatment, vital signs, and laboratory results. b. Notify the health care provider by utilizing Situation, Background, Assessment, and Recommendation (SBAR). c. Consult the wound care nurse about the change in status and the potential for infection. d. Check with the charge nurse about the change in status and the potential for infection.

Complete the head-to-toe assessment, including current treatment, vital signs, and laboratory results. rationale: The patient is showing signs and symptoms associated with infection in the wound. The nurse should complete the assessment: gather all data such as current treatment modalities, medications, vital signs including temperature, and laboratory results such as the most recent complete blood count or white cell count. The nurse can then notify the primary care provider and receive treatment orders for the patient. It is important to notify the charge nurse and consult the wound nurse on the patient's status and on any new orders.

A female nurse is called into the supervisor's office regarding her deteriorating work performance since the loss of her spouse 2 years ago. The woman begins sobbing and says that she is "falling apart" at home as well. Which type of grief is the female nurse experiencing? a. Normal grief b. Perceived grief c. Complicated grief d. Disenfranchised grief

Complicated grief rationale: In complicated grief, a person has a prolonged or significantly difficult time moving forward after a loss. Normal grief is the most common reaction to death; it involves a complex range of normal coping strategies. Disenfranchised grief involves a relationship that is not socially sanctioned. Perceived grief is not a type of grief; perceived loss is a loss that is less obvious to other people.

A patient who has had several sexual partners in the past month expresses a desire to use a contraceptive. Which contraceptive method should the nurse recommend? a. Condom b. Diaphragm c. Spermicide d. Oral contraceptive

Condom rational: Condoms are both a contraceptive and a barrier against STIs and HIV; proper use will greatly reduce the risk. Spermicides, diaphragms, and oral contraceptives all protect against pregnancy; however, they are not a barrier and do not prevent bodily fluids from coming in contact with the patient during sexual intercourse.

Which assessment finding of an older adult, who has a urinary tract infection, requires an immediate nursing intervention? a. Confusion b. Presbycusis c. Temperature of 97.9° F d. Death of a spouse 2 months ago

Confusion rationale: Confusion is a common manifestation in older adults with urinary tract infection; however, the cause requires further assessment. There may be another reason for the confusion. Confusion is not a normal finding in the older adult, even though it is commonly seen with concurrent infections. Difficulty hearing, presbycusis, is an expected finding in an older adult. Older adults tend to have lower core temperatures. Coping with the death of a spouse is a psychosocial concern to be addressed after the acute physiological concern in this case.

Which behavior indicates the nurse is using a team approach when caring for a patient who is experiencing alterations in mobility? a. Delegates assessment of lung sounds to nursing assistive personnel b. Becomes solely responsible for modifying activities of daily living c. Consults physical therapy for strengthening exercises in the extremities d. Involves respiratory therapy for altered breathing from severe anxiety levels

Consults physical therapy for strengthening exercises in the extremities rationale: The nurse should collaborate with other health care team members such as physical or occupational therapists when considering mobility needs. For example, physical therapists are a resource for planning ROM or strengthening exercises. Nurses often delegate some interventions to nursing assistive personnel, but assessment of lung sounds is the nurse's responsibility. Nursing assistive personnel may turn and position patients, apply elastic stockings, help patients use the incentive spirometer, etc. Occupational therapists are a resource for planning activities of daily living that patients need to modify or relearn. A mental health advanced practice nurse or psychologist should be used for severe anxiety.

The nurse is caring for a patient who will have both a large abdominal bandage and an abdominal binder. Which actions will the nurse take before applying the bandage and binder? (Select all that apply.) a. Cover exposed wounds. b. Mark the sites of all abrasions. c. Assess the condition of current dressings. d. Inspect the skin for abrasions and edema. e. Cleanse the area with hydrogen peroxide. f. Assess the skin at underlying areas for circulatory impairment.

Cover exposed wounds. Assess the condition of current dressings. Inspect the skin for abrasions and edema. Assess the skin at underlying areas for circulatory impairment. rationale: Before applying a bandage or a binder, the nurse has several responsibilities. The nurse would need to inspect the skin for abrasions, edema, and discoloration or exposed wound edges. The nurse also is responsible for covering exposed wounds or open abrasions with a dressing and assessing the condition of underlying dressings and changing if soiled, as well as assessing the skin of underlying areas that will be distal to the bandage. This checks for signs of circulatory impairment, so that a comparison can be made after bandages are applied. Marking the sites of all abrasions is not necessary. Although it is important for the skin to be clean, and even though it may need to be cleaned with a noncytotoxic cleanser, cleansing with hydrogen peroxide can interfere with wound healing.

The patient has been in bed for several days and needs to be ambulated. Which action will the nurse take first? a. Maintain a narrow base of support. b. Dangle the patient at the bedside. c. Encourage isometric exercises. d. Suggest a high-calcium diet.

Dangle the patient at the bedside. rationale: To prevent injury, nurses implement interventions that reduce or eliminate the eӄects of orthostatic hypotension. Mobilize the patient as soon as the physical condition allows, even if this only involves dangling at the bedside or moving to a chair. A wide base of support increases balance. Isometric exercises (i.e., activities that involve muscle tension without muscle shortening) have no beneӄcial eӄect on preventing orthostatic hypotension, but they improve activity tolerance. A high-calcium diet can help with osteoporosis but can be detrimental in an immobile patient

The nurse is caring for a patient with a pressure ulcer on the left hip. The ulcer is black. Which next step will the nurse anticipate? a. Monitor the wound. b. Document the wound. c. Debride the wound. d. Manage drainage from wound.

Debride the wound rationale: Debridement is the removal of nonviable necrotic (black) tissue. Removal of necrotic tissue is necessary to rid the ulcer of a source of infection, to enable visualization of the wound bed, and to provide a clean base for healing. A wound will not move through the phases of healing if the wound is infected. Documentation occurs after completion of skill. When treating a pressure ulcer, it is important to monitor and reassess the wound at least every 8 hours. Management of drainage will help keep the wound clean, but that is not the next step.

A nurse is observing skin integrity of an older adult. Which finding will the nurse document as a normal finding? a. Oily skin b. Faster nail growth c. Decreased elasticity d. Increased facial hair in men

Decreased elasticity rationale: Loss of skin elasticity is a common finding in the older adult. Other common findings include pigmentation changes, glandular atrophy (oil, moisture, and sweat glands), thinning hair (facial hair: decreased in men, increased in women), slower nail growth, and atrophy of epidermal arterioles.

The nurse is working on a medical-surgical unit that has been participating in a research project associated with pressure ulcers. Which risk factor will the nurse assess for that predisposes a patient to pressure ulcer development? a. Decreased level of consciousness b. Adequate dietary intake c. Shortness of breath d. Muscular pain

Decreased level of consciousness rationale: Patients who are confused or disoriented or who have changing levels of consciousness are unable to protect themselves. The patient may feel the pressure but may not understand what to do to relieve the discomfort or to communicate that he or she is feeling discomfort. Impaired sensory perception, impaired mobility, shear, friction, and moisture are other predisposing factors. Shortness of breath, muscular pain, and an adequate dietary intake are not included among the predisposing factors.

The patient applies sequential compression devices after going to the bathroom. The nurse checks the patient's application of the devices and finds that they have been put on upside down. Which of the following nursing diagnoses will the nurse add to the patient's plan of care? a. Risk for poisoning b. Deficient knowledge c. Risk for imbalanced body temperature d. Risk for suffocation

Deficient knowledge rationale: The patient needs to understand the purpose of the compression devices and that proper application is needed for them to be effective. The patient has a knowledge need and requires instruction regarding the device and its purpose and procedure. The nurse will intervene by teaching the patient about the sequential compression device and instructing the patient to call for assistance when getting up to go to the bathroom in the future, so that the nurse may assist with removal and proper reapplication. No data support a risk for poisoning, imbalanced body temperature, or suffocation.

The nurse is completing an assessment on an older-adult patient who is having difficulty falling asleep. Which condition will the nurse further assess for in this patient? a. Depression b. Mild fatigue c. Hypertension d. Hypothyroidism

Depression rationale: Older adults and other individuals who experience depressive mood problems experience delays in falling asleep, earlier appearance of REM sleep, frequent awakening, feelings of sleeping poorly, and daytime sleepiness. A person who is moderately fatigued usually achieves restful sleep, especially if the fatigue is the result of enjoyable work or exercise. Hypertension often causes early-morning awakening and fatigue. Alcohol speeds the onset of sleep. Hypothyroidism decreases stage 4 sleep.

The emergency department has been notified of a potential bioterrorist attack. The nurse assigned to the department realizes that the most important task for safety in this situation is to a. Carry out the role and responsibilities of the nurse quickly and efficiently. b. Cluster all patients with the same symptoms to a specific part of the department. c. Determine the biologic agent and manage all patients using Standard Precautions. d. Prepare for post-traumatic stress associated with this bioterrorist attack.

Determine the biologic agent and manage all patients using Standard Precautions. rationale: It is essential to determine the agent and manage all patients who are symptomatic with the suspected or confirmed bioterrorism-related illness using Standard Precautions. For certain diseases, additional precautions may be necessary. Clustering patients may be helpful with staffing and, depending on the illness, may decrease the spread. All nurses every day should carry out their roles quickly and efficiently. Psychosocial concerns are important but are not the first priority at this moment.

The nurse is caring for a patient who is immobile. The nurse wants to decrease the formation of pressure ulcers. Which action will the nurse take first? a. Offer favorite fluids. b. Turn the patient every 2 hours. c. Determine the patient's risk factors. d. Encourage increased quantities of carbohydrates and fats.

Determine the patient's risk factors. rationale: The first step in prevention is to assess the patient's risk factors for pressure ulcer development. When a patient is immobile, the major risk to the skin is the formation of pressure ulcers. Nursing interventions focus on prevention. Offering favorite fluids, turning, and increasing carbohydrates and fats are not the first steps. Determining risk factors is first so interventions can be implemented to reduce or eliminate those risk factors.

Spiritual distress has been identified in a patient who has been diagnosed with AIDS. Upon evaluating the following interventions, which are appropriate for the diagnosis of Spiritual distress? (Select all that apply.) a. Develop activities to heal body, mind, and spirit. b. Assess for potential suicide. c. Offer to pray with the patient. d. Teach relaxation, guided imagery, and meditation. e. Have patient avoid church attendance.

Develop activities to heal body, mind, and spirit. Offer to pray with the patient. Teach relaxation, guided imagery, and meditation. rationale: Interventions that are appropriate for the nursing diagnosis of Spiritual distress include (1) helping the patient develop/identify activities to heal body, mind, and spirit; (2) offering to pray with the patient; and (3) teaching relaxation, guided imagery, and medication. Assessing for potential suicide would be appropriate for the nursing diagnosis of Hopelessness. Attendance at church should be encouraged.

A nurse is helping an older-adult patient with instrumental activities of daily living. The nurse will be assisting the patient with which activity? a. Taking a bath b. Getting dressed c. Making a phone call d. Going to the bathroom

Making a phone call rationale: Instrumental activities of daily living or IADLs (such as the ability to write a check, shop, prepare meals, or make phone calls) and activities of daily living or ADLs (such as bathing, dressing, and toileting) are essential to independent living.

The nurse is caring for a patient who has had a recent stroke and is paralyzed on the left side. The patient has no respiratory or cardiac issues but cannot walk. The patient cannot button a shirt and cannot feed self due to being left-handed and becomes frustrated very easily. The patient has been eating very little and has lost 2 lbs. The patient asks the nurse, "How can I go home like this? I'm not getting better." Which health care team members will the nurse need to consult? (Select all that apply.) a. Dietitian b. Physical therapist c. Respiratory therapist d. Cardiac rehabilitation therapist e. Occupational therapist f. Psychologist

Dietitian Physical therapist Occupational therapist Psychologist rationale: Physical therapists are a resource for planning ROM or strengthening exercises, and occupational therapists are a resource for planning ADLs that patients need to modify or relearn. Because of the loss of 2 lbs and eating very little, a dietitian will also be helpful. Referral to a mental health advanced practice nurse, a licensed social worker, or a physiologist to assist with coping or other psychosocial issues is also wise. Because the patient exhibits good cardiac and respiratory function, respiratory therapy and cardiac rehabilitation probably are not needed at this time.

The nurse is admitting a patient who is a member of the Seventh Day Adventist religion. The physician has written an order for specific tests to be done the next day, which is Saturday. The nurse should a. Discuss the patient's beliefs about the Sabbath. b. Order the tests without questioning. c. Inform the physician that the tests cannot be performed. d. Reorder the tests for Sunday.

Discuss the patient's beliefs about the Sabbath. rationale: It is essential to consider cultural differences and explore personal preferences when determining nursing interventions to enhance spiritual well-being. Some Seventh Day Adventists may not mind having tests on the Sabbath. Others might. Ordering the tests without questioning may lead to patient refusal later and to wasted resources as well as spiritual distress for the patient. Informing the physician that the tests cannot be performed is premature without speaking with the patient first. It is not in the realm of the nurse to reorder tests. Some tests may be critical and may need to be done on the Sabbath.

A patient who had a colostomy placed 1 month ago is feeling depressed and does not want to participate in sexual activities anymore. The patient is afraid that the partner does not want sex. The patient is afraid the ostomy is physically unattractive. Which initial nursing intervention will be most effective in helping this patient resume sexual activity? a. Inform the patient about a support group for people with colostomies. b. Reassure the patient that lots of people resume sex the same week the colostomy is placed. c. Teach the patient about intimate activities that can be done to incorporate the ostomy. d. Discuss ways to adapt to new body image so the patient will be comfortable in resuming intimacy.

Discuss ways to adapt to new body image so the patient will be comfortable in resuming intimacy rational: The nurse should first address the patient's need to be comfortable with his or her own body image; once the patient's issues related to body image are resolved, intimacy may follow. Reassuring the patient that others manage to have sexual intercourse with an ostomy may help to decrease anxiety but may have the unintended effect of making the patient feel abnormal because he or she has not yet resumed sexual activity. Support groups may be helpful for the patient, but this is not the most effective initial intervention a nurse can provide; this may be helpful later. The patient is worried about the ostomy; incorporating it into intimate activities is insensitive and can even be damaging to the stoma.

The nurse is having a conversation with an adolescent regarding the need for sleep. The adolescent states that it is common to stay up with friends several nights a week. Which action should the nurse take next? a. Talk with the adolescent's parent about staying up with friends and the need for sleep. b. Discuss with the adolescent sleep needs and the effects of excessive daytime sleepiness. c. Refer the adolescent for counseling about alcohol abuse problems. d. Take no action for this normal occurrence

Discuss with the adolescent sleep needs and the effects of excessive daytime sleepiness rationale: Discussion regarding adolescent sleep needs should first occur with the adolescent. Although it may be common for this adolescent to want to visit with friends and experience activities that go late into the night, these activities can and do impact the hours of sleep and the physical needs of the adolescent, no matter the reason for the late nights, and they do need to be addressed. The nurse will address the adolescent, not the parents. Addressing alcohol abuse problems is not the next step but may be required later. While staying up late may be a normal occurrence for this adolescent, action is required

The nurse preceptor recognizes the new nurse's ability to determine patient safety risks when which behavior is observed? a. Checking patient identification once every shift b. Multitasking by gathering two patients' medications c. Disposing of used needles in a red needle container d. Raising all four side rails per family request

Disposing of used needles in a red needle container rationale: Needles, syringes, and other single-use injection devices should be used once and disposed of in safety red needle containers that will be disposed of properly. Patient identification should be checked multiple times a day, including before each medication, treatment, procedure, blood administration, and transfer, and at the beginning of each shift. Gathering more than one patient's medication increases the likelihood of error. Raising all four side rails is considered a restraint and requires special orders, assessment, and monitoring of the patient.

The nurse is preparing a patient for surgery. The nurse explains that the reason for writing in indelible ink on the surgical site the word "correct" is to a. Distinguish the correct surgical site. b. Label the correct patient. c. Comply with the surgeon's preference. d. Adhere to the correct regulatory standard.

Distinguish the correct surgical site. rationale: The purpose of writing on the surgical site as part of the Universal Protocol from the Joint Commission is to distinguish the correct site on the correct patient and match with the correct surgeon for patient safety and prevention of wrong site surgery. All patients who are having an invasive procedure should receive labeling in many different ways, including the record and patient armbands. Writing in indelible ink may comply with the surgeon's preference, but safety is the driving factor. Although labeling of the site helps to meet regulatory standards, this is not the reason to do this activity—the reason is to keep the patient safe.

The family is a central institution in American society; however, the concept, structure, and functioning of the family unit continue to change over time. The uniqueness of each family is referred to as family a. Durability. b. Resiliency. c. Diversity. d. Forms.

Diversity. rational: Family diversity is the uniqueness of each family unit. Family durability is the term for the intrafamilial system of support and structure that extends beyond the walls of the household. Family resiliency is the ability of the family to cope with expected and unexpected stressors. Family forms are patterns of people considered by family members to be included in the family.

Aggravating factors

Do certain activities worsen the pain?

Sleepwalking

Do not startle.

A nurse is caring for a patient with a right hemisphere stroke and partial paralysis. Which action by the nursing assistive personnel (NAP) will cause the nurse to praise the NAP? a. Dressing the left side first b. Dressing the right side first c. Dressing the lower extremities first d. Dressing the upper extremities first

Dressing the left side first rational: Dressing the left side first will be praised by the nurse. If a patient has partial paralysis and reduced sensation, the patient dresses the affected side first; in this case, the left. A stroke on the right hemisphere affects the left side of the body. The right side or upper and lower extremities are not as effective.

A nurse is caring for a dying patient. When is the best time for the nurse to discuss end-of-life care? a. During assessment b. During planning c. During implementation d. During evaluation

During assessment rationale: Because most deaths are now "negotiated" among patients, family members, and the health care team, discuss end-of-life care preferences early in the assessment phase of the nursing process. Doing so during the planning, implementation, and evaluation phases is too late.

A nurse is providing passive range of motion (ROM) for a patient with impaired mobility. Which technique will the nurse use for each movement? a. Each movement is repeated 5 times by the patient. b. Each movement is performed until the patient experiences pain. c. Each movement is completed quickly and smoothly by the nurse. d. Each movement is moved just to the point of resistance by the nurse.

Each movement is moved just to the point of resistance by the nurse. rationale: Passive ROM exercises are performed by the nurse. Carry out movements slowly and smoothly, just to the point of resistance; ROM should not cause pain. Never force a joint beyond its capacity. Each movement needs to be repeated 5 times during the session. The patient moves all joints through ROM unassisted in active ROM.

A nurse observes a patient rising from a chair slowly by pushing on the chair arms. Which type of tension and contraction did the nurse observe? a. Eccentric tension and isotonic contraction b. Eccentric tension and isometric contraction c. Concentric tension and isotonic contraction d. Concentric tension and isometric contraction

Eccentric tension and isotonic contraction rationale: This movement causes eccentric tension and isotonic contraction. Eccentric tension helps control the speed and direction of movement. For example, when using an overhead trapeze, the patient slowly lowers himself to the bed. The lowering is controlled when the antagonistic muscles lengthen. By pushing on the chair arms and rising eccentric tension and isotonic contraction occurred. In concentric tension, increased muscle contraction causes muscle shortening, resulting in movement such as when a patient uses an overhead trapeze to pull up in bed. Concentric and eccentric muscle actions are necessary for active movement and therefore are referred to as dynamic or isotonic contraction. Isometric contraction (static contraction) causes an increase in muscle tension or muscle work but no shortening or active movement of the muscle (e.g., instructing the patient to tighten and relax a muscle group, as in quadriceps set exercises or pelvic floor exercises).

A recently widowed older-adult patient is dehydrated and is admitted to the hospital for intravenous fluid replacement. During the evening shift, the patient becomes acutely confused. Which possible reversible causes will the nurse consider when assessing this patient? (Select all that apply.) a. Electrolyte imbalance b. Sensory deprivation c. Hypoglycemia d. Drug effects e. Dementia

Electrolyte imbalance Sensory deprivation Hypoglycemia Drug effects rationale: Delirium, or acute confusional state, is a potentially reversible cognitive impairment that is often due to a physiological event. Physiological causes include electrolyte imbalances, untreated pain, infection, cerebral anoxia, hypoglycemia, medication effects, tumors, subdural hematomas, and cerebrovascular infarction or hemorrhage. Sometimes it is also caused by environmental factors such as sensory deprivation or overstimulation, unfamiliar surroundings, or sleep deprivation or psychosocial factors such as emotional distress. Dementia is a gradual, progressive, and irreversible cerebral dysfunction.

The nurse is caring for a patient who is having trouble sleeping. Which action will the nurse take? a. Suggest snug-fitting nightwear. b. Provide a favorite beverage. c. Encourage deep breathing. d. Walk with the patient.

Encourage deep breathing rationale: Relaxation exercises such as slow, deep breathing for 1 or 2 minutes relieve tension and prepare the body for rest. Instruct patients to wear loose-fitting nightwear. Walking and drinking a favorite beverage would not necessarily encourage sleep

The nurse is teaching the parents of a 3-year-old child who is at risk for developmental delays. Which instruction will the nurse include in the teaching plan? a. Insist that your child discuss various points of view. b. Encourage play as your child is exploring the surroundings. c. Discuss world events with your child to foster language development. d. Actively encourage your child to read lengthy books to foster reading abilities.

Encourage play as your child is exploring the surroundings rationale: A 3-year-old child is going to use play to learn and discover the surrounding environment. Children at this age are egocentric and often are unable to see the world from any perspective other than their own. Very young children are not able to understand and comment on world events because their thinking has not advanced to abstract reasoning yet. A 3-year-old child is likely unable to read; lengthy is not appropriate.

The nurse is caring for an adolescent with an appendectomy who is reporting difficulty falling asleep. Which intervention will be most appropriate? a. Close the door to decrease noise from unit activities. b. Adjust temperature in the patient's room to 21° C (70° F). c. Ensure that the night-light in the patient's room is working. d. Encourage the discontinuation of a soda and chocolate nightly snack

Encourage the discontinuation of a soda and chocolate nightly snack. rationale: Discontinuing the soda and chocolate nightly snack will be most beneficial for this patient since it has two factors that will cause difficulty falling asleep. Coffee, tea, colas, and chocolate act as stimulants, causing a person to stay awake or to awaken throughout the night. Personal preference influences the temperature of the room, as well as the lighting of the room. Noise can be a factor in the unit and can awaken the patient, but caffeine can make it difficult to fall asleep

The nurse is caring for an older-adult patient who has been diagnosed with a stroke. Which intervention will the nurse add to the care plan? a. Encourage the patient to perform as many self-care activities as possible. b. Provide a complete bed bath to promote patient comfort. c. Coordinate with occupational therapy for gait training. d. Place the patient on bed rest to prevent fatigue.

Encourage the patient to perform as many self-care activities as possible. rationale: Nurses should encourage the older-adult patient to perform as many self-care activities as possible, thereby maintaining the highest level of mobility. Sometimes nurses inadvertently contribute to a patient's immobility by providing unnecessary help with activities such as bathing and transferring. Placing the patient on bed rest without suӄcient ambulation leads to loss of mobility and functional decline, resulting in weakness, fatigue, and increased risk for falls. After a stroke or brain attack, a patient likely receives gait training from a physical therapist; speech rehabilitation from a speech therapist; and help from an occupational therapist for ADLs such as dressing, bathing and toileting, or household chores.

A palliative team is caring for a dying patient in severe pain. Which action is the priority? a. Provide postmortem care for the patient. b. Support the patient's nurse in grieving. c. Teach the patient the stages of grief. d. Enhance the patient's quality of life.

Enhance the patient's quality of life. rationale: The primary goal of palliative care is to help patients and families achieve the best quality of life. Providing support for the patient's nurse is not the primary obligation when the patient is experiencing severe pain. Not all collaborative team members on the palliative team would be able to provide postmortem care, as is the case for nutritionists, social workers, and pharmacists. Teaching about stages of grief should not be the focus when severe pain is present.

The nurse is caring for a patient who is in the final stages of his terminal disease. The patient is very weak but refuses to use a bedpan, and wants to get up to use the bedside commode. What should the nurse do? a. Explain to the patient that he is too weak and needs to use the bedpan. b. Insert a rectal tube so that the patient no longer needs to actively defecate. c. Enlist assistance from family members if possible and assist the patient to get up. d. Put the patient on a bedpan and stay with him until he is finished.

Enlist assistance from family members if possible and assist the patient to get up. rationale: Establishing presence is part of the art of nursing. Presence involves "being with" a patient versus "doing for" a patient. Demonstrate a caring presence by listening to the patient's concerns and willingly involving family in discussions about the patient's health. The nurse should support patients as they make decisions about their health. If at all possible, the nurse should encourage the patient to maintain as much independence as possible. Inserting a rectal tube involves "doing for" instead of "being with." Placing the patient on the bedpan is against the patient's wishes and is another form of "doing for."

A nurse is developing an individualized plan of care for a patient. Which action is important for the nurse to take? a. Establish goals that are measurable and realistic. b. Set goals that are a little beyond the capabilities of the patient. c. Use the nurse's own judgment and not be swayed by family desires. d. Explain that without taking alignment risks, there can be no progress.

Establish goals that are measurable and realistic. rationale: The nurse must develop an individualized plan of care for each nursing diagnosis and must set goals that are individualized, realistic, and measurable. The nurse should set realistic expectations for care and should include the patient and family when possible. The goals focus on preventing problems or risks to body alignment and mobility.

In caring for the patient's spiritual needs, the nurse understands that a. Establishing presence is part of the art of nursing. b. Presence involves "doing for" the patient. c. A caring presence involves listening to the patient's wishes only. d. The nurse must use her expertise to make decisions for the patient.

Establishing presence is part of the art of nursing. rationale: Establishing presence is part of the art of nursing. Presence involves "being with" a patient versus "doing for" a patient. Demonstrate a caring presence by listening to the patient's concerns and willingly involving family in discussions about the patient's health. Show self-confidence when providing health instruction, and support patients as they make decisions about their health.

The nurse is caring for a patient who has been in holding in the emergency department for 24 hours. The nurse is concerned about the patient's experiencing sleep deprivation. Which action will be best for the nurse to take? a. Expedite the process of obtaining a medical-surgical room for the patient. b. Pull the curtains shut, dim the lights, and decrease the number of visitors. c. Obtain an order for a hypnotic medication to help the patient sleep. d. Ask everyone in the unit to try to be quiet so the patient can sleep

Expedite the process of obtaining a medical-surgical room for the patient rationale: The most effective treatment for sleep deprivation is elimination or correction of factors that disrupt the sleep pattern. Obtaining a private room in the medical-surgical unit for the patient will help with decreasing stimuli and promoting more rest than an individual can obtain in an emergency department even with the interventions mentioned

What is the best suggestion a nurse could make to a family requesting help in selecting a local nursing center? a. Have the family members evaluate nursing home staff according to their ability to get tasks done efficiently and safely. b. Make sure that nursing home staff members get patients out of bed and dressed according to staff's preferences. c. Explain that it is important for the family to visit the center and inspect it personally. d. Suggest a nursing center that has standards as close to hospital standards as possible.

Explain that it is important for the family to visit the center and inspect it personally. rationale: An important step in the process of selecting a nursing home is to visit the nursing home. The nursing home should not feel like a hospital. It is a home, a place where people live. Members of the nursing home staff should focus on the person, not the task. Residents should be out of bed and dressed according to their preferences, not staff preferences.

The nurse is caring for a patient with a wound. The patient appears anxious as the nurse is preparing to change the dressing. Which action should the nurse take? a. Turn on the television. b. Explain the procedure. c. Tell the patient "Close your eyes." d. Ask the family to leave the room.

Explain the procedure. rationale: Explaining the procedure educates the patient regarding the dressing change and involves him in the care, thereby allowing the patient some control in decreasing anxiety. Telling the patient to close the eyes and turning on the television are distractions that do not usually decrease a patient's anxiety. If the family is a support system, asking support systems to leave the room can actually increase a patient's anxiety.

An older-adult patient in no acute distress reports being less able to taste and smell. What is the nurse's best response to this information? a. Notify the health care provider immediately to rule out cranial nerve damage. b. Schedule the patient for an appointment at a smell and taste disorders clinic. c. Perform testing on the vestibulocochlear nerve and a hearing test. d. Explain to the patient that diminished senses are normal findings.

Explain to the patient that diminished senses are normal findings. rationale: Diminished taste and smell senses are common findings in older adults. Scheduling an appointment at a smell and taste disorders clinic, testing the vestibulocochlear nerve, or an attempt to rule out cranial nerve damage is unnecessary at this time as per the information provided.

Which nursing observation of the patient in intensive care indicates the patient is sleeping comfortably during NREM sleep? a. Eyes closed, lying quietly, respirations 12, heart rate 60 b. Eyes closed, tossing in bed, respirations 18, heart rate 80 c. Eyes closed, mumbling to self, respirations 16, heart rate 68 d. Eyes closed, lying supine in bed, respirations 22, heart rate 66

Eyes closed, lying quietly, respirations 12, heart rate 60 rationale: During NREM sleep, biological functions slow. During sleep, the heart rate decreases to 60 beats/min or less. The patient experiences decreased respirations, blood pressure, and muscle tone. Heart rates above 60 are too high and respirations of 22 are too high to indicate comfortable NREM sleep

A nurse is preparing to move a patient who is able to assist. Which principles will the nurse consider when planning for safe patient handling? (Select all that apply.) a. Keep the body's center of gravity high. b. Face the direction of the movement. c. Keep the base of support narrow. d. Use the under-axilla technique. e. Use proper body mechanics. f. Use arms and legs.

Face the direction of the movement. Use proper body mechanics. Use arms and legs. rationale: When a patient is able to assist, remember the following principles: The wider the base of support, the greater the stability of the nurse; the lower the center of gravity, the greater the stability of the nurse; facing the direction of movement prevents abnormal twisting of the spine. The use of assistive equipment and continued use of proper body mechanics significantly reduces the risk of musculoskeletal injuries. Use arms and legs (not back) because the leg muscles are stronger, larger muscles capable of greater work without injury. The under-axilla technique is physically stressful for nurses and uncomfortable for patients.

"I know it seems strange, but I feel guilty being pregnant after the death of my son last year," said a woman during her routine obstetrical examination. The nurse spends extra time with this woman, helping her realize bonding with this unborn child will not mean she is replacing the one who died. Which nursing technique does this demonstrate? a. Providing curative therapy b. Promoting spirituality c. Facilitating mourning d. Eradicating grief

Facilitating mourning rationale: The nurse facilitates mourning in family members who are still surviving. By acknowledging the pregnant woman's emotions, the nurse helps the mother bond with her fetus and recognizes the emotions that still exist for the deceased child. The nurse is not attempting to help the patient eradicate grief, which would be unrealistic. Curative therapy (curing a disease) and spiritual promotion (belief in a higher power or in the meaning of life) are not addressed by the nurse's statement.

A nurse encounters a family who experienced the death of their adult child last year. The parents are talking about the upcoming anniversary of their child's death. The nurse spends time with them discussing their child's life and death. Which nursing principle does the nurse's action best demonstrate? a. Facilitation of normal mourning b. Pain-management technique c. Grief evaluation d. Palliative care

Facilitation of normal mourning rationale: Anniversary reactions can reopen grief processes. A nurse should openly acknowledge the loss and talk about the common renewal of grief feeling around the anniversary of the individual's death; this facilitates normal mourning. The nurse is not attempting to alleviate a physical pain. The actions are of open communication, not evaluation. Palliative care refers to comfort measures for symptom relief.

Upon assessment of a middle-aged adult, the nurse observes uneven weight bearing and decreased range of joint motion. Which area is priority? a. Abuse potential b. Fall precautions c. Stroke prevention d. Self-esteem issues

Fall precautions rationale: With uneven weight bearing and decreased range of joint motion, falling is a priority. Abuse potential would indicate other findings such as bruising or unkept appearance. While stroke prevention is important in a middle-aged adult, these are not the signs of stroke. While self-esteem issues may arise from physical changes, safety is a priority over self-esteem issues.

The nurse is caring for an elderly patient who has no apparent family. When questioned about his family and his definition of family, the patient states, "I have no family. They're all gone." When asked, "Who prepares your meals?" he states, "I do, or I go out." Given the three different approaches to family nursing practice, which would be most appropriate for this patient? a. Family as context b. Family as patient c. Family as system d. Combination of "family as context" and "family as patient"

Family as context rational: If only one family member receives nursing care, it is realistic and practical to view the "family as context." Although family nursing is based on the assumption that all people regardless of age are a member of some type of family form, the patient insists that he has no family. The nurse should investigate further. However, at this time, family as patient, as system, or in a combination is not appropriate.

The nurse is caring for a patient in hospice. As she observes the family dynamics, she notes that the patient is getting adequate care, but the wife is not sleeping well and needs rest. The nurse also assesses the need for better family nutrition and meals assistance. The nurse discusses these assessments with the patient and his family and formulates a plan of care with them to address these issues. The nurse is utilizing which approach to family nursing practice? a. Family as context b. Family as patient c. Family as system d. Autocratic determination

Family as system rational: In "family as context," the primary focus is on the health of an individual member. In "family as patient," family processes and relationships are the primary focus. Often, the nurse will use the two simultaneously, as with the perspective of "family as system." Because the plan of care was developed with family input, autocratic determination was not used.

Different approaches may be taken to family nursing practice. When the nurse is caring for a patient who needs constant care in the home setting and for whom most of the care is provided by the patient's family, what is the best approach for the nurse to take? a. Family as context b. Family as patient c. Family as system d. Patient as individual

Family as system rational: The newest model, called family as system includes both relational and transactional concepts. If only one family member receives nursing care, it is realistic and practical to view the family as context. When all family members are involved in the daily care of one another, the family as patient is the best approach. Although theoretical and practical distinctions have been noted between family as context and family as patient, they are not necessarily mutually exclusive. Often, the nurse will use the two simultaneously, such as with the perspective of the family as system. Family nursing is based on the assumption that all people are members of some type of family form.

It is essential for family members to realize that a family's beliefs, values, and practices strongly influence the health-promoting behaviors of its members, and to understand that a. American families are part of the same culture with the same values and beliefs. b. Economic status has little effect on a family's ability to access adequate health care. c. Family environment in early life has a strong influence on later health practices. d. All families place a high value on good health and health practices.

Family environment in early life has a strong influence on later health practices. rational: Family environment is crucial because health behavior reinforced early in life has a strong influence on later health practices. Although American families exist within the same culture, they live in very different ways as a result of race, values, social class, and ethnicity. Economic stability increases a family's access to adequate health care. Some families do not place a high value on good health. In fact, some families accept harmful practices.

Although the family as a whole differs from individual members, the measure of family health is more than a summary of the health of all members. Of the following, what areas are unique to family assessment? (Select all that apply.) a. Family form b. Family structure c. Family function d. Family health e. Individual health

Family form Family structure Family function Family health rational: Although the family as a whole differs from individual members, the measure of family health is more than a summary of the health of all members. The form, structure, function, and health of the family are areas unique to family assessment. Although individual health is important, it varies from the family focus.

A nurse is assessing young and middle-aged adults for work-related conditions. Match the job to the work-related conditions that the nurse is assessing. Agricultural workers

Farmer's lung

Balancing employment and family life creates a variety of challenges in terms of child care and household work for both parents. This has major implications in health care because a. Maternal employment has been demonstrated to be harmful for children. b. Maternal employment has shifted the majority of household tasks to the male. c. Fathers now participate more fully in day-to-day parenting responsibilities. d. The number of single-parent families has decreased since 1970.

Fathers now participate more fully in day-to-day parenting responsibilities. rational: There is no proof that maternal employment is damaging for children. However, finding quality child care is a major issue. Managing household tasks is another challenge. Although equal division of labor receives verbal approval, most household tasks remain "women's work." Evidence suggests that the fathering role is changing. Fathers now participate more fully in day-to-day parenting responsibilities. The number of single-parent families, which doubled from the 1940s to the 1990s, seems to be stabilizing.

A nurse suspects an older-adult patient is experiencing caregiver neglect. Which assessment findings are consistent with the nurse's suspicions? a. Flea bites and lice infestation b. Left at a grocery store c. Refuses to take a bath d. Cuts and bruises

Flea bites and lice infestation rationale: Caregiver neglect includes unsafe and unclean living conditions, soiled bedding, and animal or insect infestation. Abandonment includes desertion at a hospital, nursing facility, or public location such as a shopping center. Self-neglect includes refusal or failure to provide oneself with basic necessities such as food, water, clothing, shelter, personal hygiene, medication, and safety. Physical abuse includes hitting, beating, pushing, slapping, kicking, physical restraint, inappropriate use of drugs, fractures, lacerations, rope burns, and untreated injuries.

The nurse is caring for a patient with impaired physical mobility. Which potential complications will the nurse monitor for in this patient? (Select all that apply.) a. Footdrop b. Somnolence c. Hypostatic pneumonia d. Impaired skin integrity e. Increased socialization

Footdrop Hypostatic pneumonia Impaired skin integrity rationale: Immobility leads to complications such as hypostatic pneumonia. Other possible complications include footdrop and impaired skin integrity. Interruptions in the sleep-wake cycle and social isolation are more common complications than somnolence or increased socialization.

The nurse is preparing to transfer an uncooperative patient who does not have upper body strength. Which piece of equipment will be best for the nurses to obtain? a. Drawsheet b. Full body sling c. Overhead trapeze d. Friction-reducing slide sheet

Full body sling rationale: Using a mechanical lift and full body sling to transfer an uncooperative patient who can bear partial weight or a patient who cannot bear weight and is either uncooperative or does not have upper body strength to move from bed to chair prevents musculoskeletal injuries to health care workers. The nurse should not attempt to move the patient with a drawsheet. The patient does not have upper body strength so an overhead trapeze is not appropriate. A friction-reducing slide sheet that minimizes shearing forces is not as effective as a full body sling.

The nurse is caring for a patient with a Stage IV pressure ulcer. Which type of healing will the nurse consider when planning care for this patient? a. Partial-thickness wound repair b. Full-thickness wound repair c. Primary intention d. Tertiary intention

Full-thickness wound repair rationale: Stage IV pressure ulcers are full-thickness wounds that extend into the dermis and heal by scar formation because the deeper structures do not regenerate, hence the need for full-thickness repair. The full-thickness repair has four phases: hemostasis, inflammatory, proliferative, and maturation. A wound heals by primary intention when wounds such as surgical wounds have little tissue loss; the skin edges are approximated or closed, and the risk for infection is low. Partial-thickness repairs are done on partial-thickness wounds that are shallow, involving loss of the epidermis and maybe partial loss of the dermis. These wounds heal by regeneration because the epidermis regenerates. Tertiary intention is seen when a wound is left open for several days, and then the wound edges are approximated. Wound closure is delayed until risk of infection is resolved.

The nurse is admitting a patient who has been diagnosed as having had a stroke. The health care provider writes orders for "ROM as needed." What should the nurse do next? a. Restrict patient's mobility as much as possible. b. Realize the patient is unable to move extremities. c. Move all the patient's extremities. d. Further assess the patient.

Further assess the patient. rationale: Further assessment of the patient is needed to determine what the patient is able to perform. Some patients are able to move some joints actively, whereas the nurse passively moves others. With a weak patient, the nurse may have to support an extremity while the patient performs the movement. In general, exercises need to be as active as health and mobility allow.

A nurse is caring for a patient with chronic pain from arthritis. Which action is best for the nurse to take? a. Give pain medications around the clock. b. Administer pain medication before any activity. c. Give pain medication after the pain is a 7/10 on the pain scale. d. Administer pain medication only when nonpharmacological measures have failed.

Give pain medications around the clock. rational: When a patient with arthritis has chronic pain, the best way to manage pain is to take medication regularly throughout the day to maintain constant pain relief. "Before any activity" is nonspecific, and the medication may not have time to work before activity. If the patient waits until having pain (7/10) to take the medication, pain relief takes longer. Nonpharmacological measures are used in conjunction with medications unless requested otherwise by the patient.

The nurse is caring for a patient who is experiencing a full-thickness repair. Which type of tissue will the nurse expect to observe when the wound is healing? a. Eschar b. Slough c. Granulation d. Purulent drainage

Granulation rationale: Granulation tissue is red, moist tissue composed of new blood vessels, the presence of which indicates progression toward healing. Soft yellow or white tissue is characteristic of slough—a substance that needs to be removed for the wound to heal. Black or brown necrotic tissue is called eschar, which also needs to be removed for a wound to heal. Purulent drainage is indicative of an infection and will need to be resolved for the wound to heal.

The nurse knows that children in late infancy and toddlerhood are at risk for injury owing to a. Learning to walk. b. Trying to pull up on furniture. c. Being dropped by a caregiver. d. Growing ability to explore and oral activity.

Growing ability to explore and oral activity. rationale: Injury is a leading cause of death in children over age 1, which is closely related to normal growth and development because of the child's increased oral activity and growing ability to explore the environment.

The nurse is completing a skin assessment on a patient with darkly pigmented skin. Which item should the nurse use first to assist in staging an ulcer on this patient? a. Disposable measuring tape b. Cotton-tipped applicator c. Sterile gloves d. Halogen light

Halogen light rationale: When assessing a patient with darkly pigmented skin, proper lighting is essential to accurately complete the first step in assessment—inspection—and the entire assessment process. Natural light or a halogen light is recommended. Fluorescent light sources can produce blue tones on darkly pigmented skin and can interfere with an accurate assessment. Other items that could possibly be used during the assessment include gloves for infection control, a disposable measuring device to measure the size of the wound, and a cotton-tipped applicator to measure the depth of the wound, but these items are not the first items used.

A nurse is conducting a sexual assessment. Which question is appropriate for the nurse to ask? a. Have you noticed any changes in the way you feel about yourself? b. What is your favorite sex position with men and with women? c. Do you think your partner is attractive? d. Why do you like men over women?

Have you noticed any changes in the way you feel about yourself? rational: Asking about any changes in the way you feel about yourself is an appropriate question to ask during a sexual assessment. Asking about favorite sex position with men and/or women is inappropriate and invasive. The assessment needs to focus on the patient, not the partner. Asking "why" questions is nontherapeutic and is judgmental in this scenario.

The wound care nurse visits a patient in the long-term care unit. The nurse is monitoring a patient with a Stage III pressure ulcer. The wound seems to be healing, and healthy tissue is observed. How should the nurse document this ulcer in the patient's medical record? a. Stage I pressure ulcer b. Healing Stage II pressure ulcer c. Healing Stage III pressure ulcer d. Stage III pressure ulcer

Healing Stage III pressure ulcer rationale: When a pressure ulcer has been staged and is beginning to heal, the ulcer keeps the same stage and is labeled with the words "healing stage" or healing Stage III pressure ulcer. Once an ulcer has been staged, the stage endures even as the ulcer heals. This ulcer was labeled a Stage III, and it cannot return to a previous stage such as Stage I or II. This ulcer is healing, so it is no longer labeled a Stage III.

The nurse is caring for a patient who is taking gentamicin for an infection. Which assessment is a priority? a. Hearing b. Vision c. Smell d. Taste

Hearing rational: Some antibiotics (e.g., streptomycin, gentamicin, and tobramycin) are ototoxic and permanently damage the auditory nerve, whereas chloramphenicol sometimes irritates the optic nerve. Smell and taste are not as affected.

The nurse is caring for a patient with a wound healing by full-thickness repair. Which phases will the nurse monitor for in this patient? (Select all that apply.) a. Hemostasis b. Maturation c. Inflammatory d. Proliferative e. Reproduction f. Reestablishment of epidermal layers

Hemostasis Maturation Inflammatory Proliferative rationale: The four phases involved in the healing process of a full-thickness wound are hemostasis, inflammatory, proliferative, and maturation. Three components are involved in the healing process of a partial-thickness wound: inflammatory response, epithelial proliferation (reproduction) and migration, and reestablishment of the epidermal layers.

A nurse is assessing a patient for possible altered self-concept. Which assessment finding is consistent with altered self-concept? a. Appropriately dressed with clean clothes b. Hesitant to express opinions c. Independent attitude d. Holds eye contact

Hesitant to express opinions rational: Hesitant to express views or opinions is a behavior suggestive of altered self-concept. Holds eye contact, independent attitude, and appropriate appearance are all signs of normal self-concept.

The nurse is providing discharge teaching for an older adult woman who will need dressing changes at home. Her husband, who is also elderly, is her only source of care. The husband states that he will not be able to perform the dressing changes. What does the nurse need to arrange for? a. Home care service referrals b. Extra dressing supplies c. Cancellation of the discharge d. An order for antibiotics

Home care service referrals rational: Discharge planning with a family involves an accurate assessment of what will be needed for care at the time of discharge, along with any shortcomings in the home setting. If no one can do the dressings properly, the nurse will need to arrange for a home care service referral. Extra dressing supplies will not help the situation if the husband is afraid to use them. Only under extreme situations or in an unsafe situation will the discharge be canceled. An order for antibiotics is not a replacement for good dressing change technique.

The nurse is caring for a patient who cannot bear weight but needs to be transferred from the bed to a chair. The nurse decides to use a transportable hydraulic lift. What will the nurse do? a. Place a horseshoe-shaped base on the opposite side from the chair. b. Remove straps before lowering the patient to the chair. c. Hook longer straps to the bottom of the sling. d. Attach short straps to the bottom of the sling.

Hook longer straps to the bottom of the sling. rationale: The nurse should attach the hooks on the strap to the holes in the sling. Short straps hook to top holes of the sling; longer straps hook to the bottom of the sling. The horseshoe-shaped base goes under the side of the bed on the side with the chair. Position the patient and lower slowly into the chair in accordance with manufacturer guidelines to safely guide the patient into the back of the chair as the seat descends; then remove the straps and the mechanical/hydraulic lift.

A severely depressed patient cannot state any positive attributes to life. The nurse patiently sits with this patient and assists the patient to identify several activities the patient is actually looking forward to in life. Which spiritual concept is the nurse trying to promote? a. Time management b. Reminiscence c. Hope d. Faith

Hope rationale: Hope gives a person the ability to see life as enduring or having meaning or purpose. The nurse's actions do not address time management, reminiscence, or faith. Time management is organizing and prioritizing activities to be completed in a timely manner. Reminiscence is the relationship by mentally or verbally anecdotally relieving and remembering the person and past experiences. Faith is belief in a higher power.

While gathering an adolescent's health history, the nurse recognizes that the patient began to act out behaviorally and engaged in risky behavior when the patient's parents divorced. Which information will the nurse gather to determine situational low self-esteem? a. How long the parents were married b. How the patient views behaviors c. Why the parents are divorcing d. Why the patient is acting out of control

How the patient views behaviors rational: A nurse can identify situational life stressors that can impact a person's self-concept. By asking about a patient's thoughts and feelings, the nurse will be able to use communication skills in a therapeutic manner. This will facilitate the patient's insight into behaviors and will enable the nurse to make referrals or provide needed health teaching. The length of time married and the reason for the parents' divorce do not explain the patient's behaviors. Why the patient is acting out of control is not as important as how the patient views actions when out of control.

The nurse is caring for a patient with potential skin breakdown. Which components will the nurse include in the skin assessment? (Select all that apply.) a. Vision b. Hyperemia c. Induration d. Blanching e. Temperature of skin

Hyperemia Induration Blanching Temperature of skin rationale: Assessment of the skin includes both visual and tactile inspection. Assess for hyperemia and palpate for blanching or nonblaching. Early signs of skin damage include induration, bogginess (less-than-normal stiffness), and increased warmth at the injury site compared to nearby areas. Changes in temperature can indicate changes in blood flow to that area of the skin. Vision is not included in the skin assessment.

Identity confusion

I am one with the universe.

Disturbed body image

I am ugly with all these burn scars.

A nurse is working to prevent blindness. Which preventive action is a priority? a. Screen young adults early for visual impairments. b. Include rubella and syphilis screening in the preconception care plan. c. Instruct parents to report reduced eye contact from their child immediately. d. Administer eye prophylactic antibiotics to newborns within 24 hours after birth.

Include rubella and syphilis screening in the preconception care plan. rational: Actions to prevent blindness must occur before vision impairment takes place. Screening for diseases such as rubella, syphilis, chlamydia, and gonorrhea that affect development of vision in the fetus is a preventative measure. Vision testing after birth is important to begin steps to correct or identify the problem early on so the child can develop as normally as possible; waiting until children are young adults is too late. Another technique is administering eye prophylaxis in the form of erythromycin ointment approximately 1 hour after an infant's birth. Reporting reduced eye contact is recommended but is not a preventative measure.

.A nurse is teaching a patient about patient-controlled analgesia (PCA). Which statement made by the patient indicates to the nurse that teaching is effective? a. "I will only need to be on this pain medication." b. "I feel less anxiety about the possibility of overdosing." c. "I can receive the pain medication as frequently as I need to." d. "I need the nurse to notify me when it is time for another dose."

I feel less anxiety about the possibility of overdosing. rationale: A PCA is a device that allows the patient to determine the level of pain relief delivered, reducing the risk of overdose. The PCA infusion pumps are designed to deliver a specific dose that is programmed to be available at specific time intervals (usually in the range of 8 to 15 minutes) when the patient activates the delivery button. A limit on the number of doses per hour or 4-hour interval may also be set. This can help decrease a patient's anxiety related to possible overdose. Its use also often eases anxiety because the patient is not reliant on the nurse for pain relief. Other medications, such as oral analgesics, can be given in addition to the PCA machine. One benefit of PCA is that the patient does not need to rely on the nurse to administer pain medication; the patient determines when to take the medication.

A nurse is providing medication education to a patient who just started taking ibuprofen. Which information will the nurse include in the teaching session? a. Ibuprofen helps to depress the central nervous system to decrease pain perception. b. Ibuprofen reduces anxiety, which will help you cope with your pain. c. Ibuprofen binds with opiate receptors to reduce your pain. d. Ibuprofen inhibits the production of prostaglandins.

Ibuprofen inhibits the production of prostaglandins. rational: NSAIDs like ibuprofen likely work by inhibiting the synthesis of prostaglandins to inhibit cellular responses to inflammation. Ibuprofen does not depress the central nervous system, nor does it enhance coping with pain. Opioids bind with opiate receptors to modify perceptions of pain.

The nurse in an addictions clinic is working with a patient on priority setting before the patient's discharge from residential treatment. Which goal is a priority at this time? a. Identifying local self-help groups before being discharged from the program b. Stating a plan to never be tempted by illicit substances after discharge c. Staying away from all triggers that cause substance abuse d. Recognizing personal areas of weakness to grow stronger

Identifying local self-help groups before being discharged from the program rational: Look for strengths in both the individual and the family, and provide resources and education to turn limitations into strengths, such as local self-help groups. It is not realistic to avoid ALL triggers that can result in addictive behaviors. It is unrealistic to believe that the patient will never be tempted because temptation can arise from multiple sources. On the other hand, an appropriate priority would be to recognize that triggers will arise and that the patient should learn how to handle being confronted in the post-discharge setting. Having a person talk about weaknesses without recognizing a person's strengths could be a trigger to return to an addictive lifestyle, so this would not be the most appropriate priority.

The nurse is caring for a patient with a Stage IV pressure ulcer. Which nursing diagnosis does the nurse add to the care plan? a. Readiness for enhanced nutrition b. Impaired physical mobility c. Impaired skin integrity d. Chronic pain

Impaired skin integrity rationale: After the assessment is completed and the information that the patient has a Stage IV pressure ulcer is gathered, a diagnosis of Impaired skin integrity is selected. Readiness for enhanced nutrition would be selected for an individual with an adequate diet that could be improved. Impaired physical mobility and Chronic pain do not support the current data in the question.

A 9-year-old is proudly telling the nurse about mastering the yellow belt in a martial arts class. Which developmental stage is the child exhibiting? a. Initiative versus guilt b. Industry versus inferiority c. Identity versus role confusion d. Autonomy versus shame and doubt

Industry versus inferiority rational: Industry versus inferiority occurs between the ages of 6 and 12 years. It is during this developmental task that a person gains self-esteem through new skill mastery. Initiative versus guilt is for 3 to 6 years, focusing on increasing language skills with identification of feelings. Identity versus role confusion is 12 to 20 years, focusing on finding a sense of self. Autonomy versus shame and doubt is 1 to 3 years, focusing on becoming more independent.

The nurse collects the following assessment data: right heel with reddened area that does not blanch. Which nursing diagnosis will the nurse assign to this patient? a. Imbalanced nutrition: less than body requirements b. Ineffective peripheral tissue perfusion c. Risk for infection d. Acute pain

Ineffective peripheral tissue perfusion rationale: The area on the heel has experienced a decreased supply of blood and oxygen (tissue perfusion), which has resulted in tissue damage. The most appropriate nursing diagnosis with this information is Ineffective peripheral tissue perfusion. Risk for infection, Acute pain, and Imbalanced nutrition do not support the data in the question.

The nurse is caring for an infant in the intensive care unit. Which information should the nurse consider when planning care for this patient? a. Infants cannot be assessed for pain. b. Infants respond behaviorally and physiologically to painful stimuli. c. Infants cannot tolerate analgesics owing to an underdeveloped metabolism. d. Infants have a decreased sensitivity to pain when compared with older children.

Infants respond behaviorally and physiologically to painful stimuli. rational: Infants cannot verbally express their pain, but they do express pain with behavioral cues (facial expressions, crying) and physiological indicators (changes in vital signs). Infants can tolerate analgesics, but proper dosing and close monitoring are essential. Infants and older children have the same sensitivity to pain. Pain can be assessed even though the neonate cannot verbalize; the nurse can observe behavioral clues. Nurses use behavioral cues and physiological responses to assess pain in infants.

A woman who has been in a monogamous relationship for the past 6 months presents to clinic with herpes on her labia. The patient is distraught because her partner must have cheated on her. Which response by the nurse is most effective in establishing an open rapport with a patient? a. Share a story. b. Inform the patient that all encounters are confidential. c. Encourage the patient to break up with her partner for cheating. d. Tell the patient that she must be honest about every sexual experience she has had.

Inform the patient that all encounters are confidential. rational: If open communication is to be established with the patient, the patient must know that she can trust health care team members. By telling the patient that all encounters are confidential, the nurse establishes trust. Sharing a story brings the focus to the nurse, inhibiting open rapport. The nurse does not tell the patient what to do, because that should be the patient's decision. Forcing the patient to confide by sharing every sexual encounter may hinder a trusting relationship.

The patient presents to the clinic with reports of irritability, being sleepy during the day, chronically not being able to fall asleep, and being tired. Which nursing diagnosis will the nurse document in the plan of care? a. Anxiety b. Fatigue c. Insomnia d. Sleep deprivation

Insomnia Rationale: Insomnia is experienced when the patient has chronic difficulty falling asleep, frequent awakenings from sleep, and/or short sleep or nonrestorative sleep. It is the most common sleep-related complaint and includes symptoms such as irritability, excessive daytime sleepiness, not being able to fall asleep, and fatigue. Anxiety is a vague, uneasy feeling of discomfort or dread accompanied by an autonomic response. Fatigue is an overwhelming sustained sense of exhaustion with decreased capacity for physical and mental work at a usual level. Sleep deprivation is a condition caused by dyssomnia and includes symptoms caused by illness, emotional distress, or medications

A patient presents to the emergency department with a laceration of the right forearm caused by a fall. After determining that the patient is stable, what is the next best step for the nurse to take? a. Inspect the wound for foreign bodies. b. Inspect the wound for bleeding. c. Determine the size of the wound. d. Determine the need for a tetanus antitoxin injection.

Inspect the wound for bleeding rationale: After determining that a patient's condition is stable, inspect the wound for bleeding. An abrasion will have limited bleeding, a laceration can bleed more profusely, and a puncture wound bleeds in relation to the size and depth of the wound. Address any bleeding issues. Inspect the wound for foreign bodies; traumatic wounds are dirty and may need to be addressed. Determine the size of the wound. A large open wound may expose bone or tissue and be protected, or the wound may need suturing. When the wound is caused by a dirty penetrating object, determine the need for a tetanus vaccination.

The nurse is developing a plan of care for a patient diagnosed with activity intolerance. Which strategy will the nurse use to provide the best chance of maintaining patient compliance? a. Performing 20 minutes of aerobic exercise 7 days a week with 10-minute warm-up and cool-down periods b. Instructing the patient to use an exercise log to record day, time, duration, and responses to exercise activity c. Stressing the harm of not exercising by getting the patient to take responsibility for current health status d. Arranging for the patient to join a gym that takes self-pay rather than insurance

Instructing the patient to use an exercise log to record day, time, duration, and responses to exercise activity rationale: Keeping a log may increase adherence to an exercise prescription. Recommended frequency of aerobic exercise is 3 to 5 times per week or every other day for approximately 30 minutes. Focusing on the harm of not exercising is usually counterproductive. Instead, the nurse should instruct the patient about the physiological benefits of a regular exercise program. Developing a plan of exercise that the patient may perform at home may improve compliance.

The patient presents to the clinic with a family member. The family member states that the patient has been wandering around the house and mumbling. What is the first assessment the nurse should do? a. Ask the patient why she has been wandering around the house. b. Introduce self and ask the patient her name. c. Take the patient's blood pressure, pulse, temperature, and respiratory rate. d. Immediately do a complete head-to-toe neurologic assessment. a. Ask the patient why she has been wandering around the house. b. Introduce self and ask the patient her name. c. Take the patient's blood pressure, pulse, temperature, and respiratory rate. d. Immediately do a complete head-to-toe neurologic assessment

Introduce self and ask the patient her name. rationale: Introduce self and engage the patient by asking her name to assess orientation; ask the patient why she is visiting the clinic today. Continue the assessment with vital signs and a complete workup, including a neurologic assessment.

The nurse is caring for a patient with a healing Stage III pressure ulcer. The wound is clean and granulating. Which health care provider's order will the nurse question? a. Use a low-air-loss therapy unit. b. Irrigate with Dakin's solution. c. Apply a hydrogel dressing. d. Consult a dietitian.

Irrigate with Dakin's solution rationale: Clean pressure ulcers with noncytotoxic cleansers such as normal saline, which will not kill fibroblasts and healing tissue. Cytotoxic cleansers such as Dakin's solution, acetic acid, povidone-iodine, and hydrogen peroxide can hinder the healing process and should not be utilized on clean granulating wounds. Consulting a dietitian for the nutritional needs of the patient, utilizing a low-air-loss therapy unit to decrease pressure, and applying hydrogel dressings to provide a moist environment for healing are all orders that would be appropriate

The nurse is caring for a patient who claims that he does not believe in God, nor does he believe in an "ultimate reality." The nurse realizes that this patient a. Is devoid of spirituality. b. Is an atheist/agnostic. c. Finds no meaning through relationships with others. d. Believes that what he does is meaningless.

Is an atheist/agnostic rational: Some individuals do not believe in the existence of God (atheist) or believe that there is no known ultimate reality (agnostic). This does not mean that spirituality is not an important concept for the atheist or the agnostic. Atheists search for meaning in life through their work and their relationships with others. Agnostics discover meaning in what they do or how they live because they find no ultimate meaning for the way things are. They believe that people bring meaning to what they do.

Which information from the nurse indicates a correct understanding of emerging adulthood? a. It is a type of young adulthood. b. It is a type of extended adolescence. c. It is a type of independent exploration. d. It is a type of marriage and parenthood.

It is a type of independent exploration. rationale: This newly identified stage of development from age 18 to 25 (emerging adulthood) has been described as neither an extended adolescence, as it is much freer from parental control and is much more a period of independent exploration, nor young adulthood, as most young people in their twenties have not made the transitions historically associated with adult status, especially marriage and parenthood.

A cancer patient asks the nurse what the criteria are for hospice care. Which information should the nurse share with the patient? a. It is for those needing assistance with pain management. b. It is for those having a terminal illness, such as cancer. c. It is for those with completion of an advance directive. d. It is for those expected to live less than 6 months.

It is for those expected to live less than 6 months. rationale: Patients accepted into a hospice program usually have less than 6 months to live. Patients with a terminal illness are not eligible until that point. Palliative care provides assistance with pain management when a patient is not eligible for hospice care. An advance directive can be completed by any person, even those who are healthy.

A patient is receiving opioid medication through an epidural infusion. Which action will the nurse take? a. Restrict fluid intake. b. Label the tubing that leads to the epidural catheter. c. Apply a gauze dressing to the epidural catheter insertion site. d. Ask the nursing assistive personnel to check on the patient at least once every 2 hours.

Label the tubing that leads to the epidural catheter. rationale: To reduce the accidental administration of IV medications into the epidural catheter, the tubing that leads to the epidural catheter needs to be labeled clearly. The epidural insertion site needs to be covered by a transparent dressing to prevent infection and allow the nurse to assess the site. Patients receiving epidural anesthesia need to be monitored every 15 minutes until stabilized and then at least hourly for 12 to 24 hours.

A nurse is assessing a middle-aged patient's barriers to change in eating habits. Which areas will the nurse assess that are external barriers? (Select all that apply.) a. Lack of facilities b. Lack of materials c. Lack of knowledge d. Lack of social supports e. Lack of short- and long-term goals

Lack of facilities Lack of materials Lack of social supports rationale: External barriers to change include lack of facilities, materials, and social supports. Internal barriers are lack of knowledge, insufficient skills, and undefined short- and long-term goals.

A patient states that she is pregnant and concerned because she does not know what to expect, and she wants her husband to play an active part in the birthing process. Which information should the nurse share with the patient? a. Lamaze classes can prepare pregnant women and their partners for what is coming. b. The frequency of sexual intercourse is key to helping the husband feel valued. c. After the birth, the stress of pregnancy will disappear and will be replaced by relief. d. After the baby is born, the wife should accept the extra responsibilities of motherhood.

Lamaze classes can prepare pregnant women and their partners for what is coming. rationale: Childbirth education classes (like Lamaze) can prepare pregnant women, their partners, and other support persons to participate in the birthing process. The psychodynamic aspect of sexual activity is as important as the type or frequency of sexual intercourse to young adults; however, this does not relate to the issue the patient reports (lack of knowledge and participation). The stress that many women experience after childbirth has a significant impact on the health of postpartum women. Ideally partners should share all responsibilities; however, this does not relate to the patient's concerns.

Which goal is priority when the nurse is caring for a middle-aged adult? a. Maintain immediate family relationships. b. Maintain future generation relationships. c. Maintain personal career relationships. d. Maintain work relationships.

Maintain future generation relationships rationale: Many middle-aged adults find particular joy in helping their children and other young people become productive and responsible adults. While immediate family is important, this goal is priority in young adults, not as important in middle-aged adults. During this period, personal and career achievements have often already been experienced; therefore, these goals are not priority.

The nurse is teaching a class to pregnant women about common physiological changes during pregnancy. Which information should the nurse include in the teaching session? a. Pregnancy is not a time to be having sexual activity. b. Urinary frequency will occur early in the pregnancy. c. Breast tenderness should be reported as soon as possible. d. Late in the pregnancy Braxton Hicks contraction may occur.

Late in the pregnancy Braxton Hicks contraction may occur. rationale: During the third trimester (late pregnancy), increases in Braxton Hicks contractions (irregular, short contractions), fatigue, and urinary frequency (not early) occur. Normally, women commonly have morning sickness, breast enlargement and tenderness, and fatigue. Women need to be reassured that sexual activity will not harm the fetus.

The nurse is assessing body alignment for a patient who is immobilized. Which patient position will the nurse use? a. Supine position b. Lateral position c. Lateral position with positioning supports d. Supine position with no pillow under the patient's head

Lateral position rationale: Assess body alignment for a patient who is immobilized or bedridden with the patient in the lateral position, not supine. Remove all positioning supports from the bed except for the pillow under the head, and support the body with an adequate mattress.

A nurse is assessing a group of adolescents. Which person is most likely to have the highest self-esteem? a. Latino adolescent female who has strong ethnic pride b. Caucasian boy who lives below federal poverty level c. African-American adolescent male who has severe acne d. Adolescent who was suspended twice from high school

Latino adolescent female who has strong ethnic pride rational: When cultural identity is central to self-concept and is positive, cultural pride and self-esteem tend to be strong. Environmental stressors such as low-income, body image stressors such as acne, and role performance failure often influence self-esteem negatively.

A nurse is providing postmortem care. Which action will the nurse take? a. Leave dentures in the mouth. b. Lower the head of the bed. c. Cover the body with a sterile sheet. d. Remove all tubes for an autopsy.

Leave dentures in the mouth. rationale: Leave dentures in the mouth to maintain facial shape. Raise the head of the bed as soon as possible after death to prevent discoloration of the face. Cover the body with a clean sheet. Autopsy often does not allow removal of tubes, equipment, and indwelling lines.

Genu valgum

Legs curved inward so knees come together as person walks

The nurse is caring for a patient who is at risk for skin impairment. The patient is able to sit up in a chair. The nurse includes this intervention in the plan of care. How long should the nurse schedule the patient to sit in the chair? a. At least 3 hours b. Less than 2 hours c. No longer than 30 minutes d. As long as the patient remains comfortable

Less than 2 hours rationale: When patients are able to sit up in a chair, make sure to limit the amount of time to 2 hours or less. The chair sitting time should be individualized. In the sitting position, pressure on the ischial tuberosities is greater than in a supine position. Utilize foam, gel, or an air cushion to distribute weight. Sitting for longer than 2 hours can increase the chance of ischemia.

A nurse is using the proximodistal pattern to assess an infant's growth and development as normal. Which assessment finding will the nurse determine as normal? a. Bangs objects before turns b. Lifts head before grasps c. Walks before crawls d. Laughs before coos

Lifts head before grasps rationale: Lifting the head before grasping is a normal finding according to proximodistal growth. The proximodistal growth pattern starts at the center of the body and moves toward the extremities (e.g., organ systems in the trunk of the body develop before arms and legs). The infant should turn before banging objects and crawl before walking according to the proximodistal pattern of growth. The infant should coo before laughing, but this is not an example of proximodistal; this is an example of language development.

An adolescent tells the nurse that a health professional said the fibrous tissue that connects bone and cartilage was strained in a sporting accident. On which structure will the nurse focus an assessment? a. Tendon b. Ligament c. Synergistic muscle d. Antagonistic muscle

Ligament rationale: Ligaments are white, shiny, and flexible bands of fibrous tissue that bind joints and connect bones and cartilage. Tendons are strong, flexible, and inelastic as they serve to connect muscle to bone. Muscles attach bone to bone. Synergistic muscles contract to accomplish the same movement. Antagonistic muscles cause movement at the joint.

A nurse is caring for a dying patient. One of the nurse's goals is to promote dignity and validation of the dying person's life. Which action will the nurse take to best achieve this goal? a. Take pictures of visitors. b. Provide quiet visiting time. c. Call the organ donation coordinator. d. Listen to family stories about the person.

Listen to family stories about the person. rationale: Listening to family members' stories validates the importance of the dying individual's life and reinforces the dignity of the person's life. Taking pictures of visitors does not address the value of a person's life. Calling organ donation and providing private visiting time are components of the dying process, but they do not validate a dying person's life.

A nurse is assessing young and middle-aged adults for work-related conditions. Match the job to the work-related conditions that the nurse is assessing. Dry cleaners

Liver disease

A nurse is obtaining a history on an older adult. Which finding will the nurse most typically find? a. Lives in a nursing home b. Lives with a spouse c. Lives divorced d. Lives alone

Lives with a spouse rationale: In 2012, 57% of older adults in non-institutional settings lived with a spouse (45% of older women, 71% of older men); 28% lived alone (35% of older women, 19% of older men); and only 3.5% of all older adults resided in institutions such as nursing homes or centers. Most older adults have lost a spouse due to death rather than divorce.

The nurse is caring for a 14-year-old patient in the hospital. Which goal will be priority? a. Maintain industry b. Maintain identity c. Maintain intimacy d. Maintain initiative

Maintain identity rationale: According to Erikson, a 14-year-old adolescent is developing an identity versus role confusion. Maintaining initiative is for 3 to 6 year olds. Maintaining industry is for 6 to 11 year olds. Maintaining intimacy is for young adults.

An elderly patient presents to the hospital with a history of falls, confusion, and stroke. The nurse determines that the patient is at high risk for falls. Which of the following interventions is most appropriate for the nurse to take? a. Place the patient in restraints. b. Lock beds and wheelchairs when transferring. c. Place a bath mat outside the tub. d. Silence fall alert alarm upon request of family.

Lock beds and wheelchairs when transferring. rationale: Locking the bed and wheelchairs when transferring will help to prevent these pieces of equipment from moving during transfer and will assist in the prevention of falls. Patients are not automatically placed in restraints. The restraint process consists of many steps, including thorough assessment and exhausting of alternatives. All mats and rugs should be secured to help prevent falls. Silencing alarms upon the request of family is not appropriate and could contribute to an unsafe environment.

The patient is admitted with chronic back pain. The nurse who is caring for this patient should a. Focus on finding quick remedies for the back pain. b. Look at how pain influences the patient's ability to function. c. Realize that the patient's only goal is relief of the back pain. d. Help the patient realize that there is little hope of relief from chronic pain.

Look at how pain influences the patient's ability to function. rationale: Do not just look at the patient's back pain as a problem to solve with quick remedies, but rather look at how the pain influences the patient's ability to function and achieve goals established in life (not just pain relief). Mobilizing the patient's hope is central to a healing relationship.

A nurse is observing for the universal loss in an older-adult patient. What is the nurse assessing? a. Loss of finances through changes in income b. Loss of relationships through death c. Loss of career through retirement d. Loss of home through relocation

Loss of relationships through death rationale: The universal loss for older adults usually revolves around the loss of relationships through death. Life transitions, of which loss is a major component, include retirement and the associated financial changes, changes in roles and relationships, alterations in health and functional ability, changes in one's social network, and relocation. However, these are not the universal loss.

A patient with diabetes mellitus is starting an exercise program. Which types of exercises will the nurse suggest? a. Low intensity b. Low to moderate intensity c. Moderate to high intensity d. High intensity

Low to moderate intensity rationale: Instruct patients diagnosed with diabetes mellitus to perform low- to moderate-intensity exercises, carry a concentrated form of carbohydrates (sugar packets or hard candy), and wear a medical alert bracelet. Low intensity is not beneficial. Moderate to high and high intensity are not recommended for a beginner exercise program.

The patient is immobilized after undergoing hip replacement surgery. Which finding will alert the nurse to monitor for hemorrhage in this patient? a. Thick, tenacious pulmonary secretions b. Low-molecular-weight heparin doses c. SCDs wrapped around the legs d. Elastic stockings (TED hose)

Low-molecular-weight heparin doses rationale: Heparin and low-molecular-weight heparin are the most widely used drugs in the prophylaxis of deep vein thrombosis. Because bleeding is a potential side eӄect of these medications, continually assess the patient for signs of bleeding. Pulmonary secretions that become thick and tenacious are diӄcult to remove and are a sign of inadequate hydration or developing pneumonia but not of bleeding. SCDs consist of sleeves or stockings made of fabric or plastic that are wrapped around the leg and are secured with Velcro. They decrease venous stasis by increasing venous return through the deep veins of the legs. They do not usually cause bleeding. Elastic stockings also aid in maintaining external pressure on the muscles of the lower extremities and in promoting venous return. They do not usually cause bleeding.

A nurse is following the no-lift policy when working to prevent personal injury. Which type of personal back injury is the nurse most likely trying to prevent? a. Thoracic b. Cervical c. Lumbar d. Sacral

Lumbar rationale: The most common back injury for nurses is strain on the lumbar muscle group, which includes the muscles around the lumbar vertebrae. While cervical, thoracic, and sacral can occur, lumbar is the most common.

The nurse is caring for a patient with inner ear problems. Which goal is the priority? a. Maintain balance. b. Maintain proprioception. c. Maintain muscle strength. d. Maintain body alignment.

Maintain balance rationale: Within the inner ear are the semicircular canals, three fluid-filled structures that help maintain balance. Proprioception is the awareness of the position of the body and its parts, and proprioceptors are located on nerve endings, not the inner ear. Muscle strength is maintained with activity and exercise. Although body alignment is important, it is not maintained by the inner ear.

A nurse is caring for a young adult. Which goal is priority? a. Maintain peer relationships. b. Maintain family relationships. c. Maintain parenteral relationships. d. Maintain recreational relationships.

Maintain family relationships. rationale: Family is important during young adulthood. Challenges may include the demands of working and raising families. Peer is more important in the adolescent years. Young adults are much freer from parental control. While recreation is important, the family and work are the priorities in young adults.

A nursing assistive personnel (NAP) is caring for a dying patient. Which action by the NAP will cause the nurse to intervene? a. Elevating head of bed b. Making the patient eat c. Giving mouth care every 2 to 4 hours d. Keeping skin clean, dry, and moisturized

Making the patient eat rationale: Patients should never be forced to eat so the nurse will intervene to correct this inappropriate behavior. Eating in the last days of life often causes the patient pain and discomfort. Equally, as the body is shutting down the nutrients in food are not able to be absorbed. Therefore, forcing patients to eat serves no beneficial purpose for the patient. All the rest are correct and do not need the nurse to intervene. Elevating the head of the bed is appropriate and will promote ease of breathing and lung expansion and facilitate postural drainage. Giving mouth care will protect membranes if dehydrated, nauseated, or vomiting. Keeping skin clean, dry, and moisturized will decrease skin discomfort and prevent skin breakdown.

Calcium alginate

Manufactured from seaweed and comes in sheet and rope form

A nurse is planning care for a 30 year old. Which goal is priority? a. Refine self-perception. b. Master career plans. c. Examine life goals. d. Achieve intimacy.

Master career plans rationale: From 29 to 34, the person directs enormous energy toward achievement and mastery of the surrounding world. The years from 35 to 43 are a time of vigorous examination of life goals and relationships. Between the ages of 23 and 28, the person refines self-perception and ability for intimacy.

The nurse is caring for two patients; both are having a hysterectomy. The first patient is having the hysterectomy after a complicated birth. The second patient has uterine cancer. What will most likely influence the experience of pain for these two patients? a. Meaning of pain b. Neurological factors c. Competency of the surgeon d. Postoperative support personnel

Meaning of pain rationale: The degree and quality of pain perceived by a patient are related to the meaning of the pain. The patient's perception of pain is influenced by psychological factors, such as anxiety and coping, which in turn influence the patient's experience of pain. Each patient's experience is different. Neurological factors can interrupt or influence pain perception, but neither of these patients is experiencing alterations in neurological function. The knowledge, attitudes, and beliefs of nurses, health care providers, the surgeon, and other health care personnel about pain affect pain management but do not necessarily influence a patient's pain perceptions.

The nurse is assessing an immobile patient for deep vein thromboses (DVTs). Which action will the nurse take? a. Remove elastic stockings every 4 hours. b. Measure the calf circumference of both legs. c. Lightly rub the lower leg for redness and tenderness. d. Dorsiflex the foot while assessing for patient discomfort.

Measure the calf circumference of both legs. rationale: Measure bilateral calf circumference and record it daily as an assessment for DVT. Unilateral increases in calf circumference are an early indication of thrombosis. Homan's sign, or calf pain on dorsiӄexion of the foot, is no longer a reliable indicator in assessing for DVT, and it is present in other conditions. Remove the patient's elastic stockings and/or sequential compression devices (SCDs) every 8 hours, and observe the calves for redness, warmth, and tenderness. Instruct the family, patient, and all health care personnel not to massage the area because of the danger of dislodging the thrombus.

A nurse is working in a facility that follows a comprehensive safe patient-handling program. Which finding will alert the nurse to intervene? a. Mechanical lifts are in a locked closet. b. Algorithms for patient handling are available. c. Ergonomic assessment protocols are being followed. d. A no-lift policy is in place with adherence by all staff.

Mechanical lifts are in a locked closet. rationale: The nurse will follow up when lifts are not kept in convenient locations. Comprehensive safe patient-handling programs include the following elements: an ergonomics assessment protocol for health care environments, patient assessment criteria, algorithms for patient handling and movement, special equipment kept in convenient locations to help transfer patients, back injury resource nurses, an "after-action review" that allows the health care team to apply knowledge about moving patients safely in different settings, and a no-lift policy.

A nurse is preparing a community class about sexually transmitted infections. Which primary group will the nurse focus on for this class? a. Bisexual women b. Men who have sex with men c. Youths between the ages of 24 and 27 d. Pregnant women and their partners

Men who have sex with men rational: About 20 million people in the United States are diagnosed with an STI each year, with the highest incidence occurring in men who have sex with men, bisexual men, and youths between the ages of 15 and 24. While bisexual women, youths between the ages of 24 and 27, and pregnant women and their partners are important, they are not the primary groups affected by STIs.

The nurse is preparing to move a patient to a wheelchair. Which action indicates the nurse is following recommendations for safe patient handling? a. Mentally reviews the transfer steps before beginning b. Uses own strength to transfer the patient c. Focuses solely on body mechanics d. Bases decisions on intuition

Mentally reviews the transfer steps before beginning rationale: Safe patient handling includes mentally reviewing the transfer steps before beginning the procedure to ensure both the patient's and your safety. Use the patient's strength when lifting, transferring, or moving when possible. Body mechanics alone do not protect the nurse from injury to the musculoskeletal system when moving, lifting, or transferring patients. After completing the assessment, nurses use an algorithm to guide decisions about safe patient handling.

A nurse is caring for a patient with osteoporosis and lactose intolerance. What will the nurse do? a. Encourage dairy products. b. Monitor intake of vitamin D. c. Increase intake of caӄeinated drinks. d. Try to do as much as possible for the patient.

Monitor intake of vitamin D. rationale: Encourage patients at risk to be screened for osteoporosis and assess their diets for calcium and vitamin D intake. Patients who have lactose intolerance need dietary teaching about alternative sources of calcium. Caӄeine should be decreased. The goal of the patient with osteoporosis is to maintain independence with ADLs. Assistive ambulatory devices, adaptive clothing, and safety bars help the patient maintain independence.

Which information from a co-worker on a gerontological unit will cause the nurse to intervene? a. Most older people have dependent functioning. b. Most older people have strengths we should focus on. c. Most older people should be involved in care decision. d. Most older people should be encouraged to have independence.

Most older people have dependent functioning. rationale: Most older people remain functionally independent despite the increasing prevalence of chronic disease; therefore, this misconception should be addressed. It is critical for you to respect older adults and actively involve them in care decisions and activities. You also need to identify an older adult's strengths and abilities during the assessment and encourage independence as an integral part of your plan of care.

A nurse is providing range of motion to the shoulder and must perform external rotation. Which action will the nurse take? a. Moves patient's arm in a full circle b. Moves patient's arm cross the body as far as possible c. Moves patient's arm behind body, keeping elbow straight d. Moves patient's arm until thumb is upward and lateral to head with elbow flexed

Moves patient's arm until thumb is upward and lateral to head with elbow flexed rationale: External rotation: With elbow ӄexed, move arm until thumb is upward and lateral to head. Circumduction: Move arm in full circle (Circumduction is combination of all movements of ball-and-socket joint.) Adduction: Lower arm sideways and across body as far as possible. Hyperextension: Move arm behind body, keeping elbow straight.

A nurse is teaching the staff about the sleep cycle. Which sequence will the nurse include in the teaching session? a. NREM Stage 1, 2, 3, 4, REM b. NREM Stage 1, 2, 3, 4, 3, 2, 1, REM c. NREM Stage 1, 2, 3, 4, REM, 4, 3, 2 REM d. NREM Stage 1, 2, 3, 4, 3, 2, REM

NREM Stage 1, 2, 3, 4, 3, 2, REM rationale: The cyclical pattern usually progresses from stage 1 through stage 4 of NREM, followed by a reversal from stages 4 to 3 to 2, ending with a period of REM sleep. The others are incorrect sequences.

The nurse is caring for a patient in the intensive care unit who is having trouble sleeping. The nurse explains the purpose of sleep and its benefits. Which information will the nurse include in the teaching session? (Select all that apply.) a. NREM sleep contributes to body tissue restoration. b. During NREM sleep, biological functions increase. c. Restful sleep preserves cardiac function. d. Sleep contributes to cognitive restoration. e. REM sleep decreases cortical activity

NREM sleep contributes to body tissue restoration. Restful sleep preserves cardiac function. Sleep contributes to cognitive restoration. rationale: Sleep contributes to physiological and psychological restoration. NREM sleep contributes to body tissue restoration. It allows the body to rest and conserve energy. This benefits the cardiac system by allowing the heart to beat fewer times each minute. During stage 4, the body releases growth hormone for renewal and repair of specialized cells such as the brain. During NREM sleep, biological functions slow. REM sleep is necessary for brain tissue restoration and cognitive restoration and is associated with a change in cerebral blood flow and increased cortical activity

A nurse is developing a plan of care for an older adult. Which information will the nurse consider? a. Should be standardized because most geriatric patients have the same needs b. Needs to be individualized to the patient's unique needs c. Focuses on the disabilities that all aging persons face d. Must be based on chronological age alone

Needs to be individualized to the patient's unique needs rationale: Every older adult is unique, and the nurse needs to approach each one as a unique individual. The nursing care of older adults poses special challenges because of great variation in their physiological, cognitive, and psychosocial health. Aging does not automatically lead to disability and dependence. Chronological age often has little relation to the reality of aging for an older adult.

A nurse is caring for an older-adult patient on bed rest with potential sensory deprivation. Which action will the nurse take? a. Offer the patient a back rub. b. Hang a "Do not disturb" sign on patient's door. c. Ask the patient "Would you like a newspaper to read?" d. Place the patient in the room farthest from the nurses' station.

Offer the patient a back rub. rational: Comfort measures such as washing the face and hands and providing back rubs improve the quality of stimulation and lessen the chance of sensory deprivation. The patient with sensory deprivation needs meaningful stimuli, and therapeutic massage helps establish a humanistic relationship that the patient is missing. All of the other options do not promote patient-nurse interaction and promote further social isolation.

A nurse is teaching a patient about self-concept. Which information from the patient indicates the nurse needs to follow up about components of self-concept? a. One component is identity. b. One component is coping. c. One component is body image. d. One component is role performance.

One component is coping. rational: The nurse will need to follow up for the information that a component of self-concept is coping; this is a misconception and must be corrected. The components of self-concept are identity, body image, and role performance. While self-concept may affect coping, coping is not a component of self-concept.

The nurse is precepting a student nurse and is careful to check with the student all components of the medication process. The nurse explains to the student that most errors occur in a. Ordering and transcribing. b. Dispensing and administering. c. Dispensing and transcribing. d. Ordering and administering.

Ordering and administering. rationale: Most medication errors occur in the ordering and administering stages of the medication process.

A nurse is caring for a patient with rheumatoid arthritis who is now going to be taking 2 acetaminophen (Tylenol) tablets every 6 hours to control pain. Which part of the patient's social history is the nurse most concerned about? a. Patient drinks 1 to 2 glasses of wine every night. b. Patient smokes 2 packs of cigarettes a day. c. Patient occasionally uses marijuana. d. Patient takes antianxiety medications.

Patient drinks 1 to 2 glasses of wine every night. rational: The major adverse effect of acetaminophen is hepatotoxicity (liver toxicity). Because both alcohol and acetaminophen are metabolized by the liver, when taken together, they can cause liver damage. Smoking cigarettes and smoking marijuana are not healthy behaviors, but their effects on health are not affected by acetaminophen. Antianxiety medications can be taken with acetaminophen.

A nurse delegates a position change to a nursing assistive personnel. The nurse instructs the NAP to place the patient in the lateral position. Which finding by the nurse indicates a correct outcome? a. Patient is lying on side. b. Patient is lying on back. c. Patient is lying semiprone. d. Patient is lying on abdomen.

Patient is lying on side. rationale: In the side-lying (or lateral) position the patient rests on the side with the major portion of body weight on the dependent hip and shoulder. Patients in the supine position rest on their backs. Sims' position is semiprone. The patient in the prone position lies face or chest down on the abdomen.

The nurse is administering ibuprofen (Advil) to an older patient. Which assessment data causes the nurse to hold the medication? (Select all that apply.) a. Patient states allergy to aspirin. b. Patient states joint pain is 2/10 and intermittent. c. Patient reports past medical history of gastric ulcer. d. Patient reports last bowel movement was 4 days ago. e. Patient experiences respiratory depression after administration of an opioid medication.

Patient states allergy to aspirin. Patient reports past medical history of gastric ulcer. rational: NSAIDs can cause bleeding, especially in the gastrointestinal (GI) tract; therefore, NSAIDs are most likely contraindicated in this patient. Patients with an allergy to aspirin or have asthma are sometimes also allergic to other NSAIDs. The nurse needs to verify that the health care provider is aware of the history of GI bleeding and of allergy to aspirin before administering ibuprofen. NSAIDs do not interfere with bowel function and are used for the treatment of mild to moderate acute intermittent pain. NSAIDs also do not suppress the central nervous system by causing respiratory depression.

The nurse is caring for a patient with expressive aphasia from a traumatic brain injury. Which goal will the nurse include in the plan of care? a. Patient will carry a pen and a pad of paper around for communication. b. Patient will recover full use of speech vocabulary in 1 day. c. Patient will thicken drinks to prevent aspiration. d. Patient will communicate nonverbally.

Patient will communicate nonverbally. rational: Expressive aphasia, a motor type of aphasia, is the inability to name common objects or express simple ideas in words or writing. To adapt to expressive aphasia, the nurse and the patient need to work on ways to communicate nonverbally through means such as pointing and gestures. Goals and outcomes need to be realistic and measurable; recovery in 1 day is not realistic. A patient who has expressive aphasia may not be able to speak or write words with a pen and paper. Thickening drinks prevents aspiration risk and is not included in a plan of care for this patient.

A home health nurse is assembling a puzzle with an older-adult patient and notices that the patient is having difficulty connecting two puzzle pieces. Which aspect of sensory deprivation will the nurse document as being most affected? a. Perceptual b. Cognitive c. Affective d. Social

Perceptual rational: Alterations in spatial orientation and in visual/motor coordination are signs of perceptual dysfunction. Cognitive function is the ability to think and the capacity to learn; the patient is not disoriented or unable to learn. Affective problems include boredom and restlessness; the patient is participating in an activity. The patient is interacting with the home health nurse, so socialization is not a problem.

A nurse is assessing a child for sexual abuse. Which assessment findings will the nurse expect? a. Physical aggression and sleep disturbances b. Many peers and no drug usage c. Panic attacks and anorexia d. Anxiety and depression

Physical aggression and sleep disturbances rational: Behavioral signs of sexual abuse in a child include physical aggression, sleep disturbance, poor peer relationships, and substance abuse. Panic attacks, anorexia, anxiety, and depression are behavioral signs for adults.

A nurse is establishing a relationship with the patient who is severely visually impaired and is teaching the patient how to contact the nurse for assistance. Which action will the nurse take? a. Place a raised Braille sticker on the call button. b. Explain to the patient that a staff person will stop by once an hour to see if the patient needs anything. c. Instruct the patient to tell a family member to get the attention of the staff. d. Color code the call light system.

Place a raised Braille sticker on the call button. rational: The nurse should devise a plan of care that is accommodating of the patient's visual deficit. Placing a sticker on the call light allows the patient to page the nurse for assistance as needed. Using family members is not the best option. Making hourly rounds is not sufficient; the nurse needs to ensure that the patient can get in touch at any time. Color coding the call light will not help a severely visually impaired patient.

The nurse is caring for a patient on the medical-surgical unit who is experiencing an exacerbation of asthma. Which intervention will be most appropriate to help this patient sleep? a. Place bed in semi-Fowler's position. b. Offer iron-rich foods for meals. c. Provide a snack before bedtime. d. Encourage the patient to read.

Place bed in semi-Fowler's position. rationale: Placing the patient in a semi-Fowler's position eases the work of breathing. Respiratory disease often interferes with sleep. Patients with chronic lung disease such as emphysema or asthma are short of breath and frequently cannot sleep without two or three pillows to raise their heads. Iron-rich food may help a patient with restless legs syndrome. Providing a snack and encouraging the patient to read may be good interventions for patients, but the most appropriate would be raising the head of the bed

The nurse is caring for a patient with a surgical incision that eviscerates. Which actions will the nurse take? (Select all that apply.) a. Place moist sterile gauze over the site. b. Gently place the organs back. c. Contact the surgical team. d. Offer a glass of water. e. Monitor for shock.

Place moist sterile gauze over the site Contact the surgical team Monitor for shock rationale: The presence of an evisceration (protrusion of visceral organs through a wound opening) is a surgical emergency. Immediately place damp sterile gauze over the site, contact the surgical team, do not allow the patient anything by mouth (NPO), observe for signs and symptoms of shock, and prepare the patient for emergency surgery.

The nurse is caring for a patient who needs to be placed in the prone position. Which action will the nurse take? a. Place pillow under the patient's abdomen after turning. b. Turn head toward one side with large, soft pillow. c. Position legs flat against bed. d. Raise head of bed to 45 degrees.

Place pillow under the patient's abdomen after turning. rationale: Placing a pillow under the patient's abdomen after turning decreases hyperextension of lumbar vertebrae and strain on lower back; breathing may also be enhanced. Head is turned toward one side with a small pillow to reduce ӄexion or hyperextension of cervical vertebrae. Legs should be supported with pillows to elevate toes and prevent footdrop. Forty-ӄve degrees is the position for Fowler's position; prone is on the stomach.

The nurse is teaching a patient how to use a cane. Which information will the nurse include in the teaching session? a. Place the cane at the top of the hip bone. b. Place the cane on the stronger side of the body. c. Place the cane in front of the body and then move the good leg. d. Place the cane 10 to 15 inches in front of the body when walking.

Place the cane on the stronger side of the body. rationale: Have the patient keep the cane on the stronger side of the body. A person's cane length is equal to the distance between the greater trochanter and the floor. The cane should be moved first and then the weaker leg. For maximum support when walking, the patient places the cane forward 15 to 25 cm (6 to 10 inches), keeping body weight on both legs. The weaker leg is then moved forward to the cane, so body weight is divided between the cane and the stronger leg.

The home health nurse is caring for a patient with tactile and visual deficits. The nurse is concerned about injury related to inability to feel harmful stimuli and teaches the patient safety strategies to maintain independence. Which action by the patient indicates successful learning? a. Asks the nurse to test the temperature of the water before entering the bath. b. Places colored stickers on faucet handles to indicate temperature. c. Replaces all lace-up shoes with Velcro straps for ease. d. Uses a heating pad on a low setting to keep warm.

Places colored stickers on faucet handles to indicate temperature. rational: If a patient with tactile deficits also has a visual impairment, it is important to be sure that water faucets are clearly marked "hot" and "cold," or use color codes (i.e., red for hot and blue for cold). Discourage the use of heating pads in this population. Asking the nurse to test the water does not promote independence, although it does promote safety. Velcro is easier for a patient with a tactile deficit to manipulate and promotes self-care but not safety.

A patient has been admitted and placed on fall precautions. The nurse explains to the patient that interventions for the precautions include a. Encouraging visitors in the early evening. b. Placing all four side rails in the "up" position. c. Checking on the patient once a shift. d. Placing a high risk for falls armband on the patient.

Placing a high risk for falls armband on the patient. rationale: Placing a high risk for falls armband on the patient encourages communication among the whole interdisciplinary team. Anyone who interacts with the patient should see this armband, understand its meaning, and assist the patient as necessary. The timing of visitors would not affect falls. All four side rails are considered a restraint and can contribute to falling. Individuals on high risk for fall alerts should be checked frequently, at least every hour.

An older couple expresses concern because they are easily fatigued during sexual intercourse and cannot reach climax. Which strategies to increase sexual stamina will the nurse offer? (Select all that apply.) a. Plan sexual activity around a time when the couple feels rested. b. Encourage intimate touching, such as hugging and kissing. c. Use extra lubrication to decrease discomfort. d. Take pain medication before intercourse.

Plan sexual activity around a time when the couple feels rested. Avoid alcohol and tobacco Eat well-balanced meals. rational: Alcohol, tobacco, and certain medications (such as narcotics for pain) may cause drowsiness and fatigue and negatively affect sexual stamina. Eating well-balanced meals can help to increase energy levels. Planning sexual activity when the couple is well rested will help them not get fatigued as quickly. Encouraging intimate touching may help increase libido but not energy levels. Extra lubrication and taking pain medications may ease the discomfort of sexual intercourse but are not appropriate interventions for fatigue.

A nurse is performing passive range of motion (ROM) and splinting on an at-risk patient. Which finding will indicate goal achievement for the nurse's action? a. Prevention of atelectasis b. Prevention of renal calculi c. Prevention of pressure ulcers d. Prevention of joint contractures

Prevention of joint contractures rationale: Goal achievement for passive ROM is prevention of joint contractures. Contractures develop in joints not moved periodically through their full ROM. ROM exercises reduce the risk of contractures. Researchers noted that prompt use of splinting with prescribed ROM exercises reduced contractures and improved active range of joint motion in aӄected lower extremities. Deep breathing and coughing and using an incentive spirometer will help prevent atelectasis. Adequate hydration helps prevent renal calculi and urinary tract infections. Interventions aimed at prevention of pressure ulcers include positioning, skin care, and the use of therapeutic devices to relieve pressure.

The nurse is providing care to a patient who is bedridden. The nurse raises the height of the bed. What is the rationale for the nurse's action? a. Narrows the nurse's base of support. b. Allows the nurse to bring feet closer together. c. Prevents a shift in the nurse's base of support. d. Shifts the nurse's center of gravity farther away from the base of support.

Prevents a shift in the nurse's base of support. rationale: Raising the height of the bed when performing a procedure prevents bending too far at the waist and shifting the base of support. Balance is maintained by proper body alignment and posture through two simple techniques. First, widen the base of support by separating the feet to a comfortable distance. Second, increase balance by bringing the center of gravity closer to the base of support.

The nurse is caring for a patient who has experienced a laparoscopic appendectomy. For which type of healing will the nurse focus the care plan? a. Partial-thickness repair b. Secondary intention c. Tertiary intention d. Primary intention

Primary intention rationale: A clean surgical incision is an example of a wound with little loss of tissue that heals with primary intention. The skin edges are approximated or closed, and the risk for infection is low. Partial-thickness repairs are done on partial-thickness wounds that are shallow, involving loss of the epidermis and maybe partial loss of the dermis. These wounds heal by regeneration because the epidermis regenerates. Tertiary intention is seen when a wound is left open for several days, and then the wound edges are approximated. Wound closure is delayed until the risk of infection is resolved. A wound involving loss of tissue such as a burn or a pressure ulcer or laceration heals by secondary intention. The wound is left open until it becomes filled with scar tissue. It takes longer for a wound to heal by secondary intention; thus the chance of infection is greater.

A nurse grimaces while changing a patient's colostomy bag. Which effect will the nurse's behavior most likely have on the patient? a. Assist recovery by using honest communication. b. Motivate the patient to increase physical activity. c. Promote development of a negative body image. d. Develop a kind nickname for the colostomy bag.

Promote development of a negative body image. rational: Negative nonverbal reactions by a nurse to a patient's scar or surgical alterations contribute to the patient's developing a negative body image. Nurses who have shocked or disgusted facial expressions contribute to patients' developing a negative body image. Expressions of distaste by the nurse will not facilitate recovery or ongoing communication, encourage physical activity, or promote acceptance of the colostomy bag by adopting a positive nickname.

A nurse is providing prenatal care to a first-time mother. Which information will the nurse share with the patient? (Select all that apply.) a. Regular trend for postpartum depression b. Protection against urinary infection c. Strategies for empty nest syndrome d. Exercise patterns e. Proper diet

Protection against urinary infection Exercise patterns Proper diet rationale: Prenatal care includes a thorough physical assessment of the pregnant woman during regularly scheduled intervals. Information regarding STIs and other vaginal infections and urinary infections that will adversely affect the fetus and counseling about exercise patterns, diet, and child care are important for a pregnant woman. Empty nest syndrome occurs as children leave the home. Postpartum depression is rare.

A mother brings her 12-year-old daughter into a clinic and inquires about getting a human papillomavirus (HPV) vaccine that day. Which information will the nurse share with the mother and daughter about the HPV vaccine? a. Protects against human immunodeficiency virus (HIV) b. Protects against cervical cancer c. Protects against chlamydia d. Protects against pregnancy

Protects against cervical cancer rational: The HPV vaccine is effective against the four most common types of HPVs that can cause cervical cancer. It is not effective against HIV, chlamydia, or pregnancy.

The nurse is caring for a surgical patient. Which intervention is most important for the nurse to complete to decrease the risk of pressure ulcers and encourage the patient's willingness and ability to increase mobility? a. Explain the risks of immobility to the patient. b. Turn the patient every 3 hours while in bed. c. Encourage the patient to sit up in the chair. d. Provide analgesic medication as ordered.

Provide analgesic medication as ordered. rationale: Maintaining adequate pain control (providing analgesic medications) and patient comfort increases the patient's willingness and ability to increase mobility, which in turn reduces pressure ulcer risks. Although sitting in the chair is beneficial, it does not increase mobility or provide pain control. Explaining the risk of immobility is important for the patient because it may impact the patient's willingness but not his or her ability. Turning the patient is important for decreasing pressure ulcers but needs to be done every 2 hours and, again, does not influence the patient's ability to increase mobility.

The nurse is caring for a patient on the medical-surgical unit with a wound that has a drain and a dressing that needs changing. Which action should the nurse take first? a. Provide analgesic medications as ordered. b. Avoid accidentally removing the drain. c. Don sterile gloves. d. Gather supplies.

Provide analgesic medications as ordered rationale: Because removal of dressings is painful, if often helps to give an analgesic at least 30 minutes before exposing a wound and changing the dressing. The next sequence of events includes gathering supplies for the dressing change, donning gloves, and avoiding the accidental removal of the drain during the procedure.

The nurse is caring for a patient who is terminally ill with very little time left to live. The patient states, "I always believed that there was life after death. Now, I'm not so sure. Do you think there is?" The nurse states, "I believe there is." The nurse has attempted to a. Strengthen the patient's religion. b. Provide hope. c. Support the patient's agnostic beliefs. d. Support the horizontal dimension of spiritual well-being.

Provide hope. rational: When a person has the attitude of something to look forward to, hope is present. Religion refers to the system of organized beliefs and worship that a person practices to outwardly express spirituality. This is not evident here. Agnostics believe that there is no known ultimate reality. This would indicate a lack of belief in life after death. The horizontal dimension of spiritual well-being describes positive relationships and connections people have with others. In this case, the patient is more concerned with the vertical dimension, which supports the transcendent relationship with God or some other higher power.

A nurse is discussing sexuality with an older adult. Which action will the nurse take? a. Ask closed-ended questions about specific symptoms the patient may experience. b. Provide information about the prevention of sexually transmitted infections. c. Discuss the issues of sexuality in a group in a private room. d. Explain that sexuality is not necessary as one ages.

Provide information about the prevention of sexually transmitted infections. rationale: Include information about the prevention of sexually transmitted infections when appropriate. Open-ended questions inviting an older adult to explain sexual activities or concerns elicit more information than a list of closed-ended questions about specific activities or symptoms. You need to provide privacy for any discussion of sexuality and maintain a nonjudgmental attitude. Sexuality and the need to express sexual feelings remain throughout the human life span.

An older patient has fallen and suffered a hip fracture. As a consequence, the patient's family is concerned about the patient's ability to care for self, especially during this convalescence. What should the nurse do? a. Stress that older patients usually ask for help when needed. b. Inform the family that placement in a nursing center is a permanent solution. c. Tell the family to enroll the patient in a ceramics class to maintain quality of life. d. Provide information and answer questions as family members make choices among care options.

Provide information and answer questions as family members make choices among care options. rationale: Nurses help older adults and their families by providing information and answering questions as they make choices among care options. Some older adults deny functional declines and refuse to ask for assistance with tasks that place their safety at great risk. The decision to enter a nursing center is never final, and a nursing center resident sometimes is discharged to home or to another less-acute residence. What defines quality of life varies and is unique for each person.

The nurse is caring for a patient in acute respiratory distress. The patient has multiple monitoring systems on that constantly beep and make noise. The patient is becoming agitated and frustrated over the inability to sleep. Which action by the nurse is most appropriate for this patient? a. Administer an opioid medication to help the patient sleep. b. Keep the door open during the night. c. Open the shades at night. d. Provide the patient with earplugs.

Provide the patient with earplugs. rational: Control of excessive stimuli becomes an important part of a patient's care; earplugs provide relief. Quiet time means dimming the lights throughout the unit, closing the shades, and shutting the doors. Allow patients to shut their room door to decrease noise. Opioid medications (for pain relief) should not be the first option; however, antianxiety medications and sleep aids may be considered.

A nurse is caring for a patient with a wound. Which assessment data will be most important for the nurse to gather with regard to wound healing? a. Muscular strength assessment b. Pulse oximetry assessment c. Sensation assessment d. Sleep assessment

Pulse oximetry assessment rationale: Oxygen fuels the cellular functions essential to the healing process; the ability to perfuse tissues with adequate amounts of oxygenated blood is critical in wound healing. Pulse oximetry measures the oxygen saturation of blood. Assessment of muscular strength and sensation, although useful for fitness and mobility testing, does not provide any data with regard to wound healing. Sleep, although important for rest and for integration of learning and restoration of cognitive function, does not provide any data with regard to wound healing.

A young-adult patient is brought to the hospital by police after crashing the car in a high-speed chase when trying to avoid arrest for spousal abuse. Which action should the nurse take? a. Question the patient about drug use. b. Offer the patient a cup of coffee to calm nerves. c. Discretely assess the patient for sexually transmitted infections. d. Deal with the issue at hand, not asking about previous illnesses.

Question the patient about drug use. rationale: Reports of arrests because of driving while intoxicated, wife or child abuse, or disorderly conduct are reasons for the nurse to investigate the possibility of drug abuse more carefully. Caffeine is a naturally occurring legal stimulant that stimulates the central nervous system and is not the choice for calming nerves. Although sexually transmitted infections occur in the young adult, this is not an action a nurse should take in this situation. The nurse may obtain important information by making specific inquiries about past medical problems, changes in food intake or sleep patterns, and problems of emotional lability.

A husband brings his children in to visit their mother in the hospital. The nurse asks how the family is getting along at home without their mom around. The husband states, "None of her jobs are getting done, and I don't do those jobs, so the house and the kids are falling apart." The nurse suspects that this family structure is a. Very flexible. b. Quite rigid. c. Extremely open. d. Hardy.

Quite rigid rational: A rigid structure specifically dictates who is able to accomplish a task and may limit the number of people outside the immediate family who assume these tasks. An extremely open or very flexible structure also presents problems for the family in that consistent patterns of behavior do not exist, and enactment of roles is overly flexible. Hardiness is the internal strength and durability of the family unit characterized by a sense of control over the outcome of life and an active rather than passive orientation in adapting to stressful events.

The nurse is preparing an older-adult patient's evening medications. Which treatment will the nurse recognize as relatively safe for difficulty sleeping in older adults? a. Ramelteon (Rozerem) b. Benzodiazepine c. Antihistamine d. Kava

Ramelteon (Rozerem) rationale: Ramelteon (Rozerem), a melatonin receptor agonist, is well tolerated and appears to be effective in improving sleep by improving the circadian rhythm and shortening time to sleep onset. It is safe for long- and short-term use particularly in older adults. The use of benzodiazepines in older adults is potentially dangerous because of the tendency of the drugs to remain active in the body for a longer time. As a result, they also cause respiratory depression, next-day sedation, amnesia, rebound insomnia, and impaired motor functioning and coordination, which leads to increased risk of falls. Caution older adults about using over-the-counter antihistamines because their long duration of action can cause confusion, constipation, and urinary retention. Kava promotes sleep in patients with anxiety; it should be used cautiously because of its potential toxic effects on the liver

The nurse is caring for a male patient newly diagnosed with type 1 diabetes mellitus. The patient is not adjusting well to the diagnosis and is refusing to learn how to self-inject insulin. The patient's wife is critical of the patient's refusal to learn; a small argument ensues, and the wife leaves, stating, "I'll be back later when I cool off." What should the nurse do? a. Ask the patient if he would like his wife excluded at visiting time. b. Tell the wife in the hall that her behavior is unacceptable and cannot be tolerated. c. Realize that the wife will be an important part of therapy. d. Tell the patient that he needs to listen to his wife more.

Realize that the wife will be an important part of therapy. rational: Although relationships are strained when confronted with illness, research indicates that family members have the potential to be a primary force for coping. Excluding the wife would limit that potential. The "unacceptable behavior," in this situation, may be the way this individual expresses concern and frustration. The nurse may want to speak with the wife to allow her to vent those concerns and frustrations. Telling the patient that he needs to listen to his wife would set up an adversarial situation that should be avoided.

When initiating the care of families, one factor that helps organize the family approach to the nursing process is that the nurse a. Views all patients as unique individuals. b. Realizes that families have little impact on individuals. c. Understands that individuals have little impact on families. d. Realizes that individuals have an impact on families.

Realizes that individuals have an impact on families. rational: When initiating the care of families, three factors organize the family approach to the nursing process: (1) that the nurse views all individuals within their family context, (2) that families have an impact on individuals, and (3) that individuals have an impact on families.

A previously toilet trained toddler has started wetting again. A nurse is gathering a health history from the grandparent. Which health history finding will the nurse most likely consider as the cause of the wetting? a. Dietary changes b. Recent parental death c. Playmate moved away d. Sibling was sick 2 days

Recent parental death rationale: A child's stage of development and chronological age will influence grieving. Toddlers can show grief from a loss of parent(s) through changes in their eating patterns, changes in their sleeping patterns, fussiness, and changes in their bowel and bladder habits. It is common for younger children to regress when under increased stress. Siblings being sick, dietary changes, and playmates moving away are unlikely to cause wetting.

The nurse is caring for a patient who is having difficulty understanding the written and spoken word. Which type of aphasia will the nurse report to the oncoming shift? a. Expressive b. Receptive c. Global d. Motor

Receptive rational: Sensory or receptive aphasia is the inability to understand written or spoken language. A patient is able to express words but is unable to understand questions or comments of others. Expressive aphasia, a motor type of aphasia, is the inability to name common objects or express simple ideas in words or writing. Global aphasia is the inability to understand language or communicate orally.

A nurse is working in a facility that uses no-lift policies. Which benefits will the nurse observe in the facility? (Select all that apply.) a. Reduced number of work-related injuries b. Increased musculoskeletal accidents c. Reduced safety of patients d. Improved health of nurses e. Increased indirect costs

Reduced number of work-related injuries Improved health of nurses rationale: Implementing evidence-based interventions and programs (e.g., lift teams) reduces the number of work-related injuries, which improves the health of the nurse and reduces indirect costs to the health care facility (e.g., workers' compensation and replacing injured workers). Knowing the movements and functions of muscles in maintaining posture and movement and implementing evidence-based knowledge about safe patient handling are essential to protecting the safety of both the patient and the nurse.

A nurse is caring for a patient who expresses a desire to have an elective abortion. The nurse's religious and ethical values are strongly opposed. How should the nurse best handle the situation? a. Attempt to educate the patient about the consequences of abortion. b. Refer the patient to a family planning center or another health professional. c. Continue to care for the patient, and limit conversation as much as possible. d. Inform the patient that, because of immoral issues, another nurse will have to care for her.

Refer the patient to a family planning center or another health professional. rational: The nurse must be aware of personal beliefs and values and is not required to participate in counseling or procedures that compromise those values. However, the patient is entitled to nonjudgmental care and should be referred to someone who can create a trusting environment. The nurse should not care for a patient if the quality of care could be jeopardized. The nurse should not attempt to push personal values onto a patient. The nurse also should not create tension by informing the patient that he or she does not have the same morals; this could cause the patient to feel guilty or defensive when receiving care from any health care professional.

The medical-surgical acute care patient has received a nursing diagnosis of Impaired skin integrity. Which health care team member will the nurse consult? a. Respiratory therapist b. Registered dietitian c. Case manager d. Chaplain

Registered dietitian rationale: Refer patients with pressure ulcers to the dietitian for early intervention for nutritional problems. Adequate calories, protein, vitamins, and minerals promote wound healing for the impaired skin integrity. The nurse is the coordinator of care, and collaborating with the dietitian would result in planning the best meals for the patient. The respiratory therapist can be consulted when a patient has issues with the respiratory system. Case management can be consulted when the patient has a discharge need. A chaplain can be consulted when the patient has a spiritual need.

A woman is in labor and refuses to receive any sort of anesthesia medication. Which alternative treatment is best for this patient? a. Transcutaneous electrical nerve stimulation (TENS) b. Herbal supplements with analgesic effects c. Pudendal block (regional anesthesia) d. Relaxation and guided imagery

Relaxation and guided imagery rationale: In the case of a patient in labor, relaxation with guided imagery is often an effective supplement for pain management because it provides women with a sense of control over their pain. Relaxation and guided imagery can be used during any phase of health or illness. TENS units are typically used to manage postsurgical and procedural pain. Herbal supplements need to be evaluated for safety during pregnancy. Additionally, some patients consider herbal supplements to be another form of medication, and they are not typically used to control acute pain. A pudendal block is a type of regional anesthesia (injection or infusion of local anesthetics to block a group of sensory nerve fibers); use of it does not respect the patient's wishes for nonpharmacological pain control.

The nurse is caring for a patient with a urinary catheter. After the nurse empties the collection bag and disposes of the urine, the next step is to a. Use alcohol-based gel on hands. b. Wash hands with soap and water. c. Remove eye protection and dispose of in garbage. d. Remove gloves and dispose of in garbage.

Remove gloves and dispose of in garbage. rationale: After disposing of the urine, the first step in removing personal protective equipment is removing gloves and disposing of them properly. In this scenario, the next step would be to remove eye protection followed by hand hygiene. Wash hands if the hands are visibly soiled; otherwise the use of alcohol-based gel is indicated for routine decontamination of hands.

The nurse enters the patient's room and notices a small fire in the headlight above the patient's bed. Immediately, the nurse assigns a nursing diagnosis of risk for injury with a goal for the patient to be safe. Which of the following actions should the nurse take first? a. Activate the alarm. b. Extinguish the fire. c. Remove the patient. d. Confine the fire.

Remove the patient. rationale: Nurses use the mnemonic RACE to set priorities in case of fire. All of these interventions are necessary, but this patient is in immediate danger with the fire being over his head and should be rescued and removed from the situation.

A nurse reviews an immobilized patient's laboratory results and discovers hypercalcemia. Which condition will the nurse monitor for most closely in this patient? a. Hypostatic pneumonia b. Renal calculi c. Pressure ulcers d. Thrombus formation

Renal calculi rationale: Renal calculi are calcium stones that lodge in the renal pelvis or pass through the ureters. Immobilized patients are at risk for calculi because they frequently have hypercalcemia. Hypercalcemia does not lead to hypostatic pneumonia, pressure ulcers, or thrombus formation. Immobility is one cause of hypostatic pneumonia, which is inӄammation of the lung from stasis or pooling of secretions. A pressure ulcer is an impairment of the skin that results from prolonged ischemia (decreased blood supply) within tissues. A thrombus is an accumulation of platelets, ӄbrin, clotting factors, and cellular elements of the blood attached to the interior wall of a vein or artery, which sometimes occludes the lumen of the vessel.

A nurse is caring for a postoperative patient. Which finding will alert the nurse to a potential wound dehiscence? a. Protrusion of visceral organs through a wound opening b. Chronic drainage of fluid through the incision site c. Report by patient that something has given way d. Drainage that is odorous and purulent

Report by patient that something has given way rationale: Patients often report feeling as though something has given way with dehiscence. Dehiscence occurs when an incision fails to heal properly and the layers of skin and tissue separate. It involves abdominal surgical wounds and occurs after a sudden strain such as coughing, vomiting, or sitting up in bed. Evisceration is seen when vital organs protrude through a wound opening. When there is an increase in serosanguineous drainage from a wound in the first few days after surgery, be alert for the potential for dehiscence. Infection is characterized by drainage that is odorous and purulent.

The nurse gives instructions to a nursing assistive personnel (NAP) regarding exercise for a patient. Which action by the NAP indicates a correct understanding of the directions? a. Determines the patient's ability to exercise b. Teaches the patient how to do the exercises c. Reports the patient got dizzy after exercising d. Advises the patient to work through the pain

Reports the patient got dizzy after exercising rationale: The NAP notifies the nurse if a patient reports increased fatigue, dizziness, or light-headedness when obtaining preexercise and/or postexercise vital signs. The nurse first must assess the patient's ability and tolerance to exercise. The nurse also teaches patients and their families how to implement exercise programs. The NAP can prepare patients for exercise (e.g., putting on shoes and clothing, providing hygiene needs, and obtaining preexercise and postexercise vital signs). The NAP can help the patient exercise.

A patient has obstructive sleep apnea. Which assessment is the priority? a. Gastrointestinal function b. Neurological function c. Respiratory status d. Circulatory status

Respiratory status rationale: In obstructive sleep apnea, the upper airway becomes partially or completely blocked, diminishing nasal airflow or stopping it. The person still attempts to breathe because the chest and abdominal movement continue, which results in loud snoring and snorting sounds. According to the ABCs of prioritizing care, airway and respiratory status takes priority over gastrointestinal, circulatory, and neurologic functioning

A nurse writes the following outcomes for a patient who has chronic obstructive pulmonary disease to improve activity level: Diastolic blood pressure will remain below 70 mm Hg with systolic below 130 mm Hg. Resting heart rate will range between 65 and 75. The last goal is that the patient will exercise 3 times a week. Which evaluative findings indicate successful goal achievement? (Select all that apply.) a. Resting heart rate 70 b. Blood pressure 126/64 c. Blood pressure 140/90 d. Reports doing stretching and flexibility exercises 2 times this week e. Reports doing resistive training 1 time and aerobics 2 times this week

Resting heart rate 70 Blood pressure 126/64 Reports doing resistive training 1 time and aerobics 2 times this week rationale: Compare actual outcomes with expected outcomes to determine the patient's health status and progression. Heart rate of 70 is between 65 and 75. Blood pressure 126/64 meets the goal. Did resistive training 1 time and aerobics 2 times equals exercising 3 times a week. Did stretching and flexibility exercises 2 times is below the 3 times a week. Blood pressure 140/90 is too high and does not meet the goal.

A nurse's goal for an older adult is to reduce the risk of adverse medication effects. Which action will the nurse take? a. Review the patient's list of medications at each visit. b. Teach that polypharmacy is to be avoided at all cost. c. Avoid information about adverse effects. d. Focus only on prescribed medications.

Review the patient's list of medications at each visit. rationale: Strategies for reducing the risk for adverse medication effects include reviewing the medications with older adults at each visit; examining for potential interactions with food or other medications; simplifying and individualizing medication regimens; taking every opportunity to inform older adults and their families about all aspects of medication use; and encouraging older adults to question their health care providers about all prescribed and over-the-counter medications. Although polypharmacy often reflects inappropriate prescribing, the concurrent use of multiple medications is often necessary when an older adult has multiple acute and chronic conditions. Older adults are at risk for adverse drug effects because of age-related changes in the absorption, distribution, metabolism, and excretion of drugs. Work collaboratively with the older adult to ensure safe and appropriate use of all medications—both prescribed medications and over-the-counter medications and herbal options.

When evaluating a patient's risk for spiritual crises, which of the following are part of the evaluation process? (Select all that apply.) a. Review the patient's self-perception regarding spiritual health. b. Review the patient's view of his/her purpose in life. c. Discuss with family and associates the patient's connectedness. d. Ask whether the patient's expectations are being met. e. Impress on the patient that spiritual health is permanent once obtained.

Review the patient's self-perception regarding spiritual health. Review the patient's view of his/her purpose in life. Discuss with family and associates the patient's connectedness. Ask whether the patient's expectations are being met. rationale: One critical thinking model for spiritual health evaluation lists the evaluation process as including a review of the patient's self-perception regarding spiritual health, the patient's view of his/her purpose in life, discussion with the family and close associates about the patient's connectedness, and determining whether the patient's expectations are being met. Attainment of spiritual health is a lifelong goal.

The nurse is caring for an older-adult patient. Which technique will the nurse use to enhance an older-adult patient's self-concept? a. Discussing current weather b. Encouraging patients to sing c. Reviewing old photos with patients d. Allowing patients extra computer time

Reviewing old photos with patients rational: Nurses can improve self-concept by reviewing old photographs when working with older-adult patients. This form of life review is helpful to older adults in remembering positive life events and people. Discussing weather does not involve personal reflection. Singing does not improve self-concept. Giving patients extra computer time is not applicable to improving self-concept but may help with learning.

The nurse is creating a plan of care for a patient with glaucoma. Which nursing diagnosis will the nurse include in the care plan to address a safety complication of the sensory deficit? a. Body image disturbance b. Social isolation c. Risk for falls d. Fear

Risk for falls rational: A visual disturbance poses great risk for injury due to falling from impaired depth perception and inability to see obstacles. Body image disturbance, social isolation, and fear are all valid nursing diagnoses that apply to a patient with vision deficit; however, they do not address the greatest risk for injury.

A confused patient is restless and continues to try to remove his oxygen and urinary catheter. What is the priority nursing diagnosis and intervention to implement for this patient? a. Risk for injury: Prevent harm to patient, use restraints if alternatives fail. b. Deficient knowledge: Explain the purpose of oxygen therapy and the urinary catheter. c. Disturbed body image: Encourage patient to express concerns about body. d. Caregiver role strain: Identify resources to assist with care.

Risk for injury: Prevent harm to patient, use restraints if alternatives fail. rationale: The priority nursing diagnosis is risk for injury. This patient could cause harm to himself by interrupting the oxygen therapy or by damaging the urethra by pulling the urinary catheter out. Before restraining a patient, it is important to implement and exhaust alternatives to restraint. Alternatives can include distraction and providing companionship or supervision. Patients may be moved to a location closer to the nurses' station; trained sitters or family members may be involved. Nurses need to ensure that patients are provided adequate food, liquid, toileting, and relief from pain. If these and other alternatives fail, this individual may need restraints; in this case, an order would need to be obtained for the restraint. This patient may have deficient knowledge; educating the patient about treatments could be considered as an alternative to restraints; however, the nursing diagnosis of highest priority is risk for injury. This scenario does not indicate that the patient has a disturbed body image or that the patient's caregiver is strained.

An adult son is adjusting to the idea of his chronically ill parents moving into his family home. The community health nurse is assessing the adult son for potential stressors secondary to the new family living arrangement. Which stressor will the nurse assess for in this adult son? a. Role confusion b. Role ambiguity c. Role performance d. Role overload

Role overload rational: Role overload involves having more roles or responsibilities within a role than are manageable. Role overload is common in individuals who unsuccessfully attempt to meet the demands of work and family while trying to find some personal time. Role confusion is an aspect of the developmental task of adolescence and young adulthood (identity versus role confusion). Role ambiguity involves unclear role expectations. Role performance is the way in which individuals perceive their ability to carry out significant roles; it is not a stressor unless it is judged ineffective. There is no data in the question to indicate this.

A nurse is assessing a patient's self-concept. Which area should the nurse assess first? a. Role performance b. Vital signs c. Anxiety d. Morals

Role performance rational: In assessing self-concept and self-esteem, first focus on each component of self-concept (identity, body image, and role performance). Self-concept is a psychological/emotional issue, not a physical issue for vital signs. Anxiety may be a stressors or a sign of low self-concept. Self-concept is not a moral issue.

A nurse is using the PLISSIT model when caring for a patient with dyspareunia from diminished vaginal secretions. The nurse suggests using water-soluble lubricants. Which component of PLISSIT is the nurse using? a. P b. LI c. SS d. IT

SS rational: The nurse is using the specific suggestions (SS). The PLISSIT model is as follows: Permission to discuss sexuality issues Limited Information related to sexual health problems being experienced Specific Suggestions—only when the nurse is clear about the problem Intensive Therapy—referral to professional with advanced training if necessary

A nurse is assessing a patient's wound. Which nursing observation will indicate the wound healed by secondary intention? a. Minimal loss of tissue function b. Permanent dark redness at site c. Minimal scar tissue d. Scarring that may be severe

Scarring that may be severe rationale: A wound healing by secondary intention takes longer than one healing by primary intention. The wound is left open until it becomes filled with scar tissue. If the scarring is severe, permanent loss of function often occurs. Wounds that heal by primary intention heal quickly with minimal scarring. Scar tissue contains few pigmented cells and has a lighter color than normal skin.

A nurse notices that a patient has a structural curvature of the spine associated with vertebral rotation. Which condition will the nurse most likely find documented in the patient's medical record? a. Scoliosis b. Arthritis c. Osteomalacia d. Osteogenesis

Scoliosis rationale: Scoliosis is a structural curvature of the spine associated with vertebral rotation. Osteogenesis imperfecta is an inherited disorder that makes bones porous, short, bowed, and deformed. Osteomalacia is an uncommon metabolic disease characterized by inadequate and delayed mineralization, resulting in compact and spongy bone. Arthritis is an inflammatory joint disease characterized by inflammation or destruction of the synovial membrane and articular cartilage and by systemic signs of inflammation.

The nurse is caring for a patient in the burn unit. Which type of wound healing will the nurse consider when planning care for this patient? a. Partial-thickness repair b. Secondary intention c. Tertiary intention d. Primary intention

Secondary intention rationale: A wound involving loss of tissue such as a burn or a pressure ulcer or laceration heals by secondary intention. The wound is left open until it becomes filled with scar tissue. It takes longer for a wound to heal by secondary intention; thus the chance of infection is greater. A clean surgical incision is an example of a wound with little loss of tissue that heals by primary intention. The skin edges are approximated or closed, and the risk for infection is low. Partial-thickness repair is done on partial-thickness wounds that are shallow, involving loss of the epidermis and maybe partial loss of the dermis. These wounds heal by regeneration because the epidermis regenerates. Tertiary intention is seen when a wound is left open for several days, and then the wound edges are approximated. Wound closure is delayed until the risk of infection is resolved.

Two 50-year-old men are discussing their Saturday activities. The first man describes how he tutors children as a volunteer at a community center. The other man says that he would never work with children and that he prefers to work out at the gym to meet young women to date. Which developmental stage is the second man exhibiting? a. Mistrust b. Inferiority c. Generativity d. Self-absorption

Self-absorption rational: In the generativity versus self-absorption developmental task, a self-absorbed person is concerned about own personal wants and desires in a self-centered manner. Generativity is the first man's developmental stage. Trust versus mistrust occurs in the first year of life. Industry versus inferiority commonly occurs in school children. Identity versus role confusion commonly occurs at the start of adolescence into young adulthood.

A nurse is teaching a patient about self-concept. Which information from the patient indicates a correct understanding of the teaching? a. Self-concept is how a person feels about others. b. Self-concept is how a person thinks about others. c. Self-concept is how a person feels about oneself. d. Self-concept is how a person thinks about oneself.

Self-concept is how a person thinks about oneself rational: Self-concept, or how a person thinks about oneself, directly affects self-esteem, or how one feels about oneself. While others may influence self-concept, self-concept is not how one feels or thinks about others.

Equipment-related accidents are risks in the health care agency. The nurse assesses for this risk when using a. Sequential compression devices. b. A measuring device that measures urine. c. Computer-based documentation. d. A manual medication-dispensing device.

Sequential compression devices rationale: Sequential compression devices are used on a patient's extremities to assist in prevention of deep vein thrombosis and have the potential to malfunction and harm the patient. Measuring devices used by the nurse to measure urine, computer documentation, and manual dispensing devices can break or malfunction but are not used directly on a patient.

A nurse is caring for a 35-year-old female patient who recently started taking antidepressants after repeated attempts at fertility treatment. The patient tells the nurse, "I feel happier, but my sex drive is gone." Which nursing diagnosis has the highest priority? a. Sexual dysfunction b. Ineffective coping c. Risk for self-directed violence d. Deficient knowledge about contraception

Sexual dysfunction rational: Antidepressants have adverse effects on sexual desire and response. The nurse should be sure to educate the patient on the potential for these side effects and how to correct for them, for example, using lubricant to ease discomfort. The patient has taken steps toward effective coping by seeking therapy. The patient has not expressed a reason for the nurse to be concerned about contraceptives. The nurse should always assess for concerns about violence in a patient's life. Although some antidepressants have been related to self-directed violence, this patient focus is on becoming pregnant (fertility treatments) but sex drive is gone.

The patient has just been diagnosed with narcolepsy. The nurse teaches the patient about management of the condition. Which information from the patient will cause the nurse to intervene? a. Takes antidepressant medications b. Naps shorter than 20 minutes c. Sits in hot, stuffy rooms d. Chews gum

Sits in hot, stuffy rooms rationale: The nurse will intervene about sitting in a hot, stuffy room as this will make the narcolepsy worse so this needs to be corrected. Patients with narcolepsy need to avoid factors that increase drowsiness (e.g., alcohol, heavy meals, exhausting activities, long-distance driving, and long periods of sitting in hot, stuffy rooms). Patients are treated with antidepressants, and management techniques involve scheduling naps no longer than 20 minutes and chewing gum. Additional management techniques include exercise, light high-protein meals, deep breathing, and taking vitamins

A veteran is hospitalized after surgical amputation of both lower extremities owing to injuries sustained during military service. Which type of loss will the nurse focus the plan of care on for this patient? a. Perceived loss b. Situational loss c. Maturational loss d. Uncomplicated loss

Situational loss rationale: Loss of a body part from injury is a situational loss. Maturational losses occur as part of normal life transitions across the life span. A perceived loss is uniquely defined by the person experiencing the loss and is less obvious to other people. Uncomplicated loss is not a type of loss; it is a description of normal grief.

The nurse is updating the plan of care for a patient with impaired skin integrity. Which findings indicate achievement of goals and outcomes? (Select all that apply.) a. The patient's expectations are not being met. b. Skin is intact with no redness or swelling. c. Nonblanchable erythema is absent. d. No injuries to the skin and tissues are evident. e. Granulation tissue is present.

Skin is intact with no redness or swelling. Nonblanchable erythema is absent. No injuries to the skin and tissues are evident. Granulation tissue is present rationale: Optimal outcomes are to prevent injury to skin and tissues, reduce injury to skin, reduce injury to underlying tissues, and restore skin integrity. Skin intact, nonblanchable erythema absent, no injuries, and presence of granulation tissue are all findings indicating achievement of goals and outcomes. The patient's expectations not being met indicates no progression toward goals/outcomes.

The nurse is caring for a postpartum patient. The patient's labor has lasted over 28 hours within the hospital; the patient has not slept and is disoriented to date and time. Which nursing diagnosis will the nurse document in the patient's care plan? a. Insomnia b. Impaired parenting c. Ineffective coping d. Sleep deprivation

Sleep deprivation rationale: This patient has been deprived of sleep by staying awake during a 28-hour labor. Disorientation is one potential sign of sleep deprivation. In this scenario, there is a clear cause for the patient's lack of sleep, and it is a one-time episode. Insomnia, on the other hand, is a chronic disorder whereby patients have difficulty falling asleep, awaken frequently, or sleep only for a short time. This scenario does not indicate that this has been a chronic problem for this patient. Although ineffective coping can manifest as a sleep disturbance, clear evidence shows that it was labor that deprived this patient of sleep, not an inability to cope. It could be difficult to care for an infant when sleep deprived; however, this scenario gives no evidence that this mother displays impaired parenting and is not caring adequately for her child or lacks the skills to do so

A community health nurse is providing an educational session at the senior center on how to promote sleep. Which practices should the nurse recommend? (Select all that apply.) a. Take a nap in the afternoon. b. Sleep where you sleep best. c. Use sedatives as a last resort. d. Watch television right before sleep. e. Decrease fluids 2 to 4 hours before sleep. f. Get up if unable to fall asleep in 15 to 30 minutes.

Sleep where you sleep best Use sedatives as a last resort. Decrease fluids 2 to 4 hours before sleep. Get up if unable to fall asleep in 15 to 30 minutes. rationale: The nurse should instruct the patient to sleep where he or she sleeps best, to use sedatives as a last resort, to decrease fluid intake to cut down on bathroom trips, and, if unable to sleep in 15 to 30 minutes, to get up out of bed. Naps should be eliminated if they are not part of the individual's routine schedule, and if naps are taken, they should be limited to 20 minutes or less a day. Television can stimulate and disrupt sleep patterns.

A community health nurse is providing an educational session at the senior center on how to promote sleep. Which practices should the nurse recommend? (Select all that apply.) a. Take a nap in the afternoon. b. Sleep where you sleep best. c. Use sedatives as a last resort. d. Watch television right before sleep. e. Decrease fluids 2 to 4 hours before sleep. f. Get up if unable to fall asleep in 15 to 30 minutes

Sleep where you sleep best. Use sedatives as a last resort. Decrease fluids 2 to 4 hours before sleep. Get up if unable to fall asleep in 15 to 30 minutes rationale: The nurse should instruct the patient to sleep where he or she sleeps best, to use sedatives as a last resort, to decrease fluid intake to cut down on bathroom trips, and, if unable to sleep in 15 to 30 minutes, to get up out of bed. Naps should be eliminated if they are not part of the individual's routine schedule, and if naps are taken, they should be limited to 20 minutes or less a day. Television can stimulate and disrupt sleep patterns

A nurse is assessing an older adult for cognitive changes. Which symptom will the nurse report as normal? a. Disorientation b. Poor judgment c. Slower reaction time d. Loss of language skills

Slower reaction time rationale: Slower reaction time is a common change in the older adult. Symptoms of cognitive impairment, such as disorientation, loss of language skills, loss of the ability to calculate, and poor judgment are not normal aging changes and require further investigation of underlying causes.

A patient has both hearing and visual sensory impairments. Which psychological nursing diagnosis will the nurse add to the care plan? a. Risk for falls b. Self-care deficit c. Social isolation d. Impaired physical mobility

Social isolation rational: In focusing on the psychological aspect of care, the nurse is most concerned about social isolation for a patient who may have difficulty communicating owing to visual and hearing impairment. Self-care deficit, impaired physical mobility, and fall risk are physiological risks for the patient.

A 72-year-old woman was recently widowed. She worked as a teller at a bank for 40 years and has been retired for the past 5 years. She never learned how to drive. She lives in a rural area that does not have public transportation. Which psychosocial change does the nurse focus on as a priority? a. Sexuality b. Retirement c. Environment d. Social isolation

Social isolation rationale: The highest priority at this time is the potential for social isolation. This woman does not know how to drive and lives in a rural community that does not have public transportation. All of these factors contribute to her social isolation. Other possible changes she may be going through right now include sexuality related to her advanced age and recent death of her spouse; however, this is not the priority at this time. She has been retired for 5 years, so this is also not an immediate need. She may eventually experience needs related to environment, but the data do not support this as an issue at this time.

A nurse is caring for a patient who fell on the ice and has connective tissue damage in the wrist and hand. The patient describes the pain as throbbing. Which type of pain does the nurse document in this patient's medical record? a. Visceral pain b. Somatic pain c. Centrally generated pain d. Peripherally generated pain

Somatic pain rational: Somatic pain comes from bone, joint, connective tissue, or muscle. Visceral pain arises from the visceral (internal) organs such as the GI tract and pancreas. Peripherally generated pain in the peripheral nerves can be caused by polyneuropathies or mononeuropathies. Centrally generated pain results from injury to the central or peripheral nervous system, causing deafferentation or sympathetically maintained pain.

The nurse is caring for a patient with conductive hearing loss resulting from prolonged cerumen impaction. Which intervention by the nurse is most important in establishing effective communication with the patient? a. Speaking with hands, face, and expressions b. Using a loud voice, enunciating every syllable c. Having direct conversation with the patient in the affected ear d. Repeating the phrase again, if the patient does not understand what the nurse said

Speaking with hands, face, and expressions rational: Use visible expressions. Speak with your hands, your face, and your eyes. Do not shout. Speaking in loud tones can distort a patient's ability to hear; the nurse should speak in normal low tones. If the patient does not understand the first time, try rephrasing instead of repeating the message. The nurse can direct conversation toward the patient's unaffected ear.

In assessing the spiritual health of her patients, the nurse understands that a. Spiritual beliefs change as patients grow and develop. b. Spiritual health in older adults leads to peace and acceptance of others. c. Older adults often express spirituality by focusing on themselves. d. The basis of beliefs among older people is focused on one or two factors.

Spiritual beliefs change as patients grow and develop rationale: Spiritual beliefs change as patients grow and develop. Health spirituality in older adults leads to peace and acceptance of self. However, older adults often express their spirituality by turning to important relationships and giving of themselves to others. Beliefs among older people vary based on many factors, such as gender, past experience, religion, economic status, and ethnic background.

The nurse creates a referral to pastoral care when he/she realizes that the patient is in need of a. Psychiatric care. b. Return to religious affiliation. c. Spiritual care. d. Transfer to the psychiatric unit.

Spiritual care. rationale: Spiritual care helps people identify meaning and purpose in life, look beyond the present, and maintain personal relationships, as well as a relationship with a higher being or life force. The patient may need psychiatric care and may be transferred to the psychiatric unit, but referral to pastoral care will not provide that. Return to a religious affiliation may follow a return to spiritual health.

A complex concept that is unique to each individual; is dependent upon a person's culture, development, life experiences, beliefs, and ideas about life; and is a unifying theme in peoples' lives is called a. Spirituality. b. Religion. c. Self-transcendence. d. Faith.

Spirituality rationale: Spirituality is a complex concept that is unique to each individual; is dependent upon a person's culture, development, life experiences, beliefs, and ideas about life; and is a unifying theme in peoples' lives. Religion refers to the system of organized beliefs and worship that a person practices to outwardly express spirituality. Self-transcendence is the belief that there is a force outside of and greater than the person. Faith allows people to have firm beliefs despite lack of physical evidence.

Which of the following statement about religion and spirituality is true? a. Religion is a unifying theme in people's lives. b. Spirituality is unique to the individual. c. Spirituality encompasses religion. d. Religion and spirituality are synonymous.

Spirituality is unique to the individual. rationale: Spirituality is a complex concept that is unique to each individual. Religion refers to the system of organized beliefs and worship that a person practices to outwardly express spirituality. People from different religions view spirituality differently. Although closely associated, spirituality and religion are not synonymous. Religious practices encompass spirituality, but spirituality does not need to include religious practice.

The nurse is admitting an older patient from a nursing home. During the assessment, the nurse notes a shallow open reddish, pink ulcer without slough on the right heel of the patient. How will the nurse stage this pressure ulcer? a. Stage I b. Stage II c. Stage III d. Stage IV

Stage II rationale: This would be a Stage II pressure ulcer because it presents as partial-thickness skin loss involving epidermis and dermis. The ulcer presents clinically as an abrasion, blister, or shallow crater. Stage I is intact skin with nonblanchable redness over a bony prominence. With a Stage III pressure ulcer, subcutaneous fat may be visible, but bone, tendon, and muscles are not exposed. Stage IV involves full-thickness tissue loss with exposed bone, tendon, or muscle.

A home care nurse is inspecting a patient's house for safety issues. Which findings will cause the nurse to address the safety problems? (Select all that apply.) a. Stairway faintly lit b. Bathtub with grab bars c. Scatter rugs in the kitchen d. Absence of smoke alarms e. Low-pile carpeting in the living room f. Level thresholds between bathroom and bedroom

Stairway faintly lit Scatter rugs in the kitchen Absence of smoke alarms rational: Assess the patient's home for common hazards, including the following: (1) loose area rugs and runner placed over carpeting, (2) poor lighting in stairways, and (3) absence of smoke alarms. Because of reduced depth perception, patients can trip on throw rugs, runners, or the edge of stairs. A bathtub with grab bars is safe and does not need to be addressed. Teach patients and family members to keep all flooring in good repair, and advise them to use low-pile carpeting. Thresholds between rooms need to be level with the floor.

A nurse is developing a drinking prevention presentation for adolescents. Which areas should the nurse include in the teaching session? a. Stressful life events and scholarships b. Very high self-esteem and work failure c. Health problems and avoidance of conflict d. Stress management and improving self-esteem

Stress management and improving self-esteem rational: Drinking prevention efforts should include stress management and improving self-esteem. High self-esteem decreases risk of drinking. Stressful life events when balanced with positive issues, such as receipt of a scholarship, are less likely to induce drinking. Conflict resolution can strengthen adolescent coping strategies to decrease drinking.

A nurse is teaching a health promotion class about isotonic exercises. Which types of exercises will the nurse give as examples? a. Swimming, jogging, and bicycling b. Tightening or tensing of muscles without moving body parts c. Quadriceps set exercises and contraction of the gluteal muscles d. Push-ups, hip lifting, pushing feet against a footboard on the bed

Swimming, jogging, and bicycling rationale: Examples of isotonic exercises are walking, swimming, dance aerobics, jogging, bicycling, and moving arms and legs with light resistance. Isometric exercises involve tightening or tensing of muscles without moving body parts. Examples include quadriceps set exercises and contraction of the gluteal muscles. Examples of resistive isometric exercises are push-ups and hip lifting, as well as placing a footboard on the foot of the bed for patients to push against with their feet.

The nurse is caring for a patient in the emergency department with an injured shoulder. Which type of joint will the nurse assess? a. Fibrous b. Synovial c. Synergistic d. Cartilaginous

Synovial rationale: Synovial joints, or true joints, such as the hinge type at the elbow, are freely movable and the most mobile, numerous, and anatomically complex body joints. Fibrous joints fit closely together and are fixed, permitting little, if any, movement such as the syndesmosis between the tibia and the fibula. Synergistic is a type of muscle, not joint. Cartilaginous joints have little movement but are elastic and use cartilage to unite separate bony surfaces such as the synchondrosis that attaches the ribs to the costal cartilage.

An older-adult patient has developed acute confusion. The patient has been on tranquilizers for the past week. The patient's vital signs are normal. What should the nurse do? a. Take into account age-related changes in body systems that affect pharmacokinetic activity. b. Increase the dose of tranquilizer if the cause of the confusion is an infection. c. Note when the confusion occurs and medicate before that time. d. Restrict phone calls to prevent further confusion.

Take into account age-related changes in body systems that affect pharmacokinetic activity. rationale: Some sedatives and tranquilizers prescribed for acutely confused older adults sometimes cause or exacerbate confusion. Carefully administer drugs used to manage confused behaviors, taking into account age-related changes in body systems that affect pharmacokinetic activity. When confusion has a physiological cause (such as an infection), specifically treat that cause, rather than the confused behavior. When confusion varies by time of day or is related to environmental factors, nonpharmacological measures such as making the environment more meaningful, providing adequate light, etc., should be used. Making phone calls to friends or family members allows older adults to hear reassuring voices, which may be beneficial.

A young mother has been hospitalized for an irregular heartbeat (dysrhythmia). The night nurse makes rounds and finds the patient awake. Which action by the nurse is most appropriate? a. Inform the patient that it is late and time to go to sleep. b. Ask the patient if she would like medication for sleep. c. Recommend a good movie that is on television tonight. d. Take time to sit and talk with the patient about her inability to sleep.

Take time to sit and talk with the patient about her inability to sleep rationale: A nurse on the night shift needs to take time to sit and talk with patients unable to sleep. This helps to determine the factors keeping patients awake. Assessment is the first step of the nursing process; therefore assessment needs to be done first and involves ascertaining the cause of the patient's inability to sleep. Patients who are admitted to the hospital for uncertain diagnoses can be stressed and worried about the testing and outcomes. In addition, a young mother can be worried about the care of her children and those caring for the children. This uncertainty and change in routine can cause difficulty in resting or falling asleep. A distraction such as a television may or may not work for the patient. After assessment is completed, a sedative may or may not be in order. Telling the patient that it is late and time to go to sleep is not a therapeutic response for an adult who is under stress

A nurse is reviewing a patient's history. Which priority finding will alert the nurse to assess the patient for possible sexual dysfunction? a. Takes vacations out of the country b. Takes antianxiety medication c. Takes exercise classes d. Takes afternoon naps

Takes antianxiety medication rational: Medications that can affect sexual functioning include antihypertensive, antipsychotics, antidepressants, and antianxiety. Taking vacations out of the country, exercise classes, and afternoon naps are not as priority for sexual functioning as medications.

During the admission assessment, the nurse assesses the patient for fall risk. Which of the following has the greatest potential to increase the patient's risk for falls? a. The patient is 59 years of age. b. The patient walks 2 miles a day. c. The patient takes Benadryl (diphenhydramine) for allergies. d. The patient recently became widowed.

The patient takes Benadryl (diphenhydramine) for allergies. rationale: Benadryl (diphenhydramine) has the potential to cause drowsiness and dizziness as a side effect, thereby increasing the risk for falls. Over 60 is the age typically found on fall assessments that increase the risk for falls. Walking has many benefits, including increasing strength, which would be beneficial in decreasing risk. Becoming widowed would increase stress and may affect concentration but is not the greatest risk.

A nurse takes the history of a middle-aged patient in a health clinic. Which information indicates the patient has achieved generativity? a. Married for 30 years b. Teaches preschoolers c. Has no regrets with life choices d. Cares for aging parents after work

Teaches preschoolers rationale: Teaching preschoolers indicates generativity. Middle-aged adults achieve success (generativity) in this stage by contributing to future generations through parenthood, teaching, mentoring, and community involvement. Married for 30 years indicates achievement of intimacy. Has no regrets is ego integrity. Caring for aging parents is admirable but it does not indicate development of the next generation (generativity).

A male older-adult patient expresses concern and anxiety about decreased penile firmness during an erection. What is the nurse's best response? a. Tell the patient that libido will always decrease, as well as the sexual desires. b. Tell the patient that touching should be avoided unless intercourse is planned. c. Tell the patient that heterosexuality will help maintain stronger libido. d. Tell the patient that this change is expected in aging adults.

Tell the patient that this change is expected in aging adults. rationale: Aging men typically experience an erection that is less firm and shorter acting and have a less forceful ejaculation. Testosterone lessens with age and sometimes (not always) leads to a loss of libido. However, for both men and women sexual desires, thoughts, and actions continue throughout all decades of life. Sexuality involves love, warmth, sharing, and touching, not just the act of intercourse. Touch complements traditional sexual methods or serves as an alternative sexual expression when physical intercourse is not desired or possible. Clearly not all older adults are heterosexual, and there is emerging research on older adult, lesbian, gay, bisexual, and transgender individuals and their health care needs.

A homeless adult patient presents to the emergency department. The nurse obtains the following vital signs: temperature 94.8° F, blood pressure 100/56, apical pulse 56, respiratory rate 12. Which of the vital signs should be addressed immediately? a. Respiratory rate b. Temperature c. Apical pulse d. Blood pressure

Temperature rationale: Hypothermia is defined as a core body temperature of 95° F or below. Homeless individuals are more at risk for hypothermia owing to exposure to the elements.

A nurse is assessing an 18-month-old toddler. The nurse distinguishes normal from abnormal findings by remembering Gesell's theory of development. Which information will the nurse consider? a. Growth in humans is determined solely by heredity. b. Environmental influence does not influence development. c. The cephalocaudal pattern describes the sequence in which growth is fastest at the top. d. Gene activity influences development but does not affect the sequence of development.

The cephalocaudal pattern describes the sequence in which growth is fastest at the top rationale: Gesell's theory of development states that the cephalocaudal pattern describes the sequence in which growth is fastest at the top (e.g., head/brain develop faster than arm and leg coordination). Growth in humans is determined by heredity, genes, and environment. Environment does influence development. Gesell's theory states that genes direct the sequence of development, but environmental factors also influence development, resulting in developmental changes.

A nurse is administering a vaccine to a child who is visually impaired. After the needle enters the arm, the child says, "Ow, that was sharp!" How will the nurse interpret the finding when the child said that it was sharp? a. The child's sensation is intact. b. The child's reception is intact. c. The child's perception is intact. d. The child's reaction is intact.

The child's perception is intact. rational: When a person becomes conscious of a stimulus and receives the information, perception takes place. Perception includes integration and interpretation of stimuli based on the person's experiences. Sensation is a general term that refers to awareness of sensory stimuli through the body's sense mechanisms. Reception begins with stimulation of a nerve cell called a receptor, which is usually for only one type of

A nurse is assessing temperaments of children. Which terms should the nurse use to describe findings? (Select all that apply.) a. The easy child b. The defiant child c. The difficult child d. The slow-to-warm up child e. The momma's boy or daddy's girl

The easy child The difficult child The slow to warm up child rationale: Psychiatrists identified three basic classes of temperament: the easy child; the difficult child; and the slow-to-warm up child. There is no momma's boy or daddy's girl or defiant child.

The nurse is caring for an elderly patient admitted with nausea, vomiting, and diarrhea. Upon completing the health history, which priority concern would require collaboration with social services to address the patient's health care needs? a. The electricity was turned off 2 days ago. b. The water comes from the county water supply. c. A son and family recently moved into the home. d. The home is not furnished with a microwave oven.

The electricity was turned off 2 days ago. rationale: Electricity is needed for refrigeration of food, and lack of electricity could have contributed to the nausea, vomiting, and diarrhea—potential food poisoning. This discussion about the patient's electrical needs can be referred to social services. The water supply, the increased number of individuals in the home, and not having a microwave may or may not be concerns but do not pertain to the current health care needs of this patient.

A nurse is caring for a patient with presbycusis. Which assessment finding indicates an adaptation to the sensory deficit? a. The patient frequently cleans out eyes with saline washes. b. The patient applies different spices during mealtime to food. c. The patient turns one ear toward the nurse during conversation. d. The patient isolates self from social situations with groups of people.

The patient turns one ear toward the nurse during conversation. rational: Presbycusis is impaired hearing due to the aging process. Adaptation for a sensory deficit indicates that the patient alters behavior to accommodate for the sensory deficit, such as turning the unaffected ear toward the speaker. Cleaning the eye and applying spices to food would not have an effect for a patient with presbycusis. Avoiding others because of a sensory deficit is maladaptive.

A nurse is working with a patient who wants needs to be met and is impatient and demanding when these needs are not met immediately. How should the nurse interpret this finding according to Freud? a. The id is functioning. b. The ego is functioning. c. The superego is functioning. d. The Oedipus complex is functioning.

The id is functioning rationale: The id is functioning. The id (i.e., basic instinctual impulses driven to achieve pleasure) is the most primitive part of the personality and originates in the infant. The infant, in this case the patient, cannot tolerate delay and must have needs met immediately. The ego represents the reality component, mediating conflicts between the environment and the forces of the id. The ego helps people judge reality accurately, regulate impulses, and make good decisions. The third component, the superego, performs regulating, restraining, and prohibiting actions. Often referred to as the conscience, the superego is influenced by the standards of outside social forces (e.g., parent or teacher). The child fantasizes about the parent of the opposite sex as his or her first love interest, known as the Oedipus or Electra complex. By the end of this stage, the child attempts to reduce this conflict by identifying with the parent of the same sex as a way to win recognition and acceptance.

A nurse lets the transplant coordinator make a request for organ and tissue donation from the patient's family. What is the primary rationale for the nurse's action? a. The nurse is not as knowledgeable as the coordinator. b. The nurse is uncomfortable asking the family. c. The nurse does not want to upset the family. d. The nurse is following a federal law.

The nurse is following a federal law. rationale: In accordance with federal law, a specially trained professional (e.g., transplant coordinator or social worker) makes requests for organ and tissue donation at the time of every death. Given the complex and sensitive nature of such requests, only specially trained personnel make the requests. Although the nurse may be less knowledgeable than the coordinator, uncomfortable asking the family, or not wanting to upset the family, the primary rationale is to be in accordance with federal law.

When caring for a terminally ill patient, the nurse should focus on the fact that a. Spiritual care is possibly the least important nursing intervention. b. Spiritual needs often need to be sacrificed for physical care priorities. c. The nurse's relationship with the patient allows for an understanding of patient priorities. d. Members of the church or synagogue play no part in the patient's plan of care.

The nurse's relationship with the patient allows for an understanding of patient priorities. rationale: The nurse's relationship with the patient allows the nurse to understand the patient's priorities. Spiritual priorities do not need to be sacrificed for physical care priorities. When a patient is terminally ill, spiritual care is possibly the most important nursing intervention. If the patient participates in a formal religion, involve in the plan of care members of the clergy or members of the church, temple, mosque, or synagogue.

The nurse is caring for a hospitalized patient. Which of the following behaviors alerts the nurse to consider the need for restraint? a. The patient refuses to call for help to go to the bathroom. b. The patient continues to remove the nasogastric tube. c. The patient gets confused regarding the time at night. d. The patient does not sleep and continues to ask for items.

The patient continues to remove the nasogastric tube. rationale: Restraints are utilized only when alternatives have been exhausted, the patient continues a behavior that can be harmful to himself or others, and the restraint is clinically justified. In this circumstance, continuing to remove a needed nasogastric tube would meet these criteria. Refusing to call for help, although unsafe, is not a reason for restraint. Getting confused at night regarding the time or not sleeping and bothering the staff to ask for items is not a reason for restraint.

The nurse is discussing with a patient's physician the need for restraint. The nurse indicates that alternatives have been utilized. What behaviors would indicate that the alternatives are working? a. The patient continues to get up from the chair at the nurses' station. b. The patient apologizes for being "such a bother." c. The patient folds three washcloths over and over. d. The sitter leaves the patient alone to go to lunch.

The patient folds three washcloths over and over. rationale: Offering diversionary activities such as something to hold is a way to keep the hands busy and provides an alternative to restraints. Assigning a room near the nurses' station or a chair at the desk can be an alternative for continuous monitoring. Getting up constantly can be cause for concern. Apologizing is not an alternative to restraints. Having a sitter sit with the patient to keep him occupied can be an alternative to restraints, but the sitter needs to be continuous.

Which nursing observation will indicate the patient is at risk for pressure ulcer formation? a. The patient has fecal incontinence. b. The patient ate two thirds of breakfast. c. The patient has a raised red rash on the right shin. d. The patient's capillary refill is less than 2 seconds.

The patient has fecal incontinence. rationale: The presence and duration of moisture on the skin increase the risk of ulcer formation by making it susceptible to injury. Moisture can originate from wound drainage, excessive perspiration, and fecal or urinary incontinence. Bacteria and enzymes in the stool can enhance the opportunity for skin breakdown because the skin is moistened and softened, causing maceration. Eating a balanced diet is important for nutrition, but eating just two thirds of the meal does not indicate that the individual is at risk. A raised red rash on the leg again is a concern and can affect the integrity of the skin, but it is located on the shin, which is not a high-risk area for skin breakdown. Pressure can influence capillary refill, leading to skin breakdown, but this capillary response is within normal limits.

A patient follows all the instructions a nurse provides because the patient wants to be perceived as a "good" patient. How should the nurse interpret this information according to moral development? a. The patient is in postformal thought reasoning. b. The patient is in postconventional reasoning. c. The patient is in preconventional reasoning. d. The patient is in conventional reasoning.

The patient is in conventional reasoning rationale: The patient is in conventional reasoning, specifically stage 3: Good Boy-Nice Girl Orientation. The patient wants to win approval from the nurse by "being good." Developmentalists proposed a fifth stage of cognitive (not moral) development termed postformal thought. Within this stage, adults demonstrate the ability to recognize that answers vary from situation to situation and that solutions need to be sensible. The person finds a balance between basic human rights and obligations and societal rules and regulations in the level of postconventional reasoning. Individuals move away from moral decisions based on authority or conformity to groups to define their own moral values and principles. Preconventional reasoning is the premoral level, in which there is limited cognitive thinking and the individual's thinking is primarily egocentric. At this stage, thinking is mostly based on likes and pleasures.

The nurse is evaluating outcomes for the patient with insomnia. Which key principle will the nurse consider during this process? a. The patient is the best evaluator of sleep. b. The nurse is the best evaluator of sleep. c. Effective interventions are the best evaluators of sleep. d. Observations of the patient are the best evaluators of sleep

The patient is the best evaluator of sleep rationale: With regard to problems with sleep, the patient is the source for evaluating outcomes. The patient is the only one who knows whether sleep problems have improved and what has been successful. Interventions are not the best indicator; achievement of goals according to the patient is the best. Observations do provide needed data, but in the case of insomnia, the patient is the source for evaluating the restfulness of sleep

The nurse is assessing a patient for opioid tolerance. Which finding supports the nurse's assessment? a. The patient needed a substantial dose of naloxone (Narcan). b. The patient needs increasingly higher doses of opioid to control pain. c. The patient no longer experiences sedation from the usual dose of opioid. d. The patient asks for pain medication close to the time it is due around the clock.

The patient needs increasingly higher doses of opioid to control pain. rational: Opioid tolerance occurs when a patient needs higher doses of an opioid to control pain. Naloxone (Narcan) is an opioid antagonist that is given to reverse the effects of opioid overdose. Taking pain medications regularly around the clock is an effective way to control pain. The pain medication for this patient is most likely effectively managing the patient's pain because the patient is not asking for the medication before it is due. A patient no longer experiencing a side effect (sedation) of an opioid does not indicate opioid tolerance.

A patient who had a motor vehicle crash 2 days ago is experiencing pain and is receiving patient-controlled analgesia (PCA). Which assessment finding indicates effective pain management with the PCA? a. The patient is sleeping and is difficult to arouse. b. The patient rates pain at a level of 2 on a 0 to 10 scale. c. The patient has sufficient medication left in the PCA syringe. d. The patient presses the control button to deliver pain medication.

The patient rates pain at a level of 2 on a 0 to 10 scale. rational: A level of 2 on a scale of 0 to 10 is evidence of effective pain management. The effectiveness of pain-relief measures is determined by the patient. If the patient is satisfied with the amount of pain relief, then pain measures are effective. A patient who is sleeping and is difficult to arouse is possibly oversedated; the nurse needs to assess this patient further. The amount of medication left in the PCA syringe does not indicate whether pain management is effective or not. Pressing the button shows that the patient knows how to use the PCA but does not evaluate pain management.

The nurse is completing discharge education for the patient regarding home medications. Which patient behavior is an indication that the patient understands the directions regarding the antibiotic medication? a. The patient nods throughout the educational session. b. The patient reads the medication prescription out loud. c. The patient states, "I will finish the antibiotic in ten days." d. The patient asks where to get the prescription filled.

The patient states, "I will finish the antibiotic in ten days." rationale: The patient stating the time frame for when the medication will be complete is the best answer. Nodding, reading the prescription out loud, or knowing where to get the prescription filled does not indicate understanding regarding directions for taking the antibiotic

The nurse has placed a patient on high-risk alert for falls. Which of the following observations by the nurse would indicate that the patient has an understanding of this alert? a. The patient removes the high alert armband to bathe. b. The patient wears the red nonslip footwear. c. The call light is kept on the bedside table. d. The patient insists on taking a "water" pill on home schedule in the evening.

The patient wears the red nonslip footwear rationale: Red nonslip footwear helps to grip the floor and decreases the chance of falling. The communication armband should stay in place and should not be removed, so that all members of the interdisciplinary team have the information about the high risk for falls. Call lights should be kept within reach of the patient. Taking diuretics early in the day assists with decreasing the number of bathroom trips at night—the time when falls are most frequent.

The nurse is administering pain medication for several patients. Which patient does the nurse administer medication to first? a. The patient who needs to be premedicated before walking b. The patient who has a PCA running that needs the syringe replaced c. The patient who needs to take a scheduled dose of maintenance pain medication d. The patient who is experiencing 8/10 pain and has an immediate order for pain medication

The patient who is experiencing 8/10 pain and has an immediate order for pain medication rational: Immediate (STAT) medications need to be given as soon as possible. In addition, this patient is the priority because of the report of severe pain. The other patients need pain medication, but their situations are not as high a priority as that of the patient with the STAT medication order.

The nurse is caring for a patient with a Stage II pressure ulcer and has assigned a nursing diagnosis of Risk for infection. The patient is unconscious and bedridden. The nurse is completing the plan of care and is writing goals for the patient. Which is the best goal for this patient? a. The patient will state what to look for with regard to an infection. b. The patient's family will demonstrate specific care of the wound site. c. The patient's family members will wash their hands when visiting the patient. d. The patient will remain free of odorous or purulent drainage from the wound.

The patient will remain free of odorous or purulent drainage from the wound. rationale: Because the patient has an open wound and the skin is no longer intact to protect the tissue, the patient is at increased risk for infection. The nurse will be assessing the patient for signs and symptoms of infection, including an increase in temperature, an increase in white count, and odorous and purulent drainage from the wound. The patient is unconscious and is unable to communicate the signs and symptoms of infection. It is important for the patient's family to be able to demonstrate how to care for the wound and wash their hands, but these statements are not goals or outcomes for this nursing diagnosis.

A male patient states, "I'm such a loser. I only had that job for a month." Which outcome criteria will the nurse add to the patient's care plan? a. The patient will verbalize two life areas in which he functions well. b. The patient will find new employment before the next clinic visit. c. The patient will confront a former boss about previous work problems. d. The patient will identify two reasons why he is considered a bad employee.

The patient will verbalize two life areas in which he functions well. rational: Verbalizing two life areas in which a person functions well is an individualized measurable outcome that is realistic. Confronting a former boss could have physical and emotional repercussions for the patient. If the patient is voicing that he has problems obtaining employment, then putting extra pressure to obtain employment would be detrimental to the patient and does not reflect a supportive and caring nursing outcome. Focusing on the negative of why the patient is considered a bad employee is not as beneficial as focusing on strengths.

Which goal is most appropriate for a patient who has had a total hip replacement? a. The patient will ambulate briskly on the treadmill by the time of discharge. b. The patient will walk 100 feet using a walker by the time of discharge. c. The nurse will assist the patient to ambulate in the hall 2 times a day. d. The patient will ambulate by the time of discharge.

The patient will walk 100 feet using a walker by the time of discharge. rationale: "The patient will walk 100 feet using a walker by the time of discharge" is individualized, realistic, and measurable. "Ambulating briskly on a treadmill" is not realistic for this patient. The option that focuses on the nurse, not the patient, is not a measurable goal; this is an intervention. "The patient will ambulate by the time of discharge" is not measurable because it does not specify the distance. Even though we can see that the patient will ambulate, this does not quantify how far.

A 55-year-old female presents to the outpatient clinic describing irregular menstrual periods and hot flashes. Which information should the nurse share with the patient? a. The patient's assessment points toward normal menopause. b. Those symptoms are normal when a woman undergoes the climacteric. c. An assessment is not really needed because these problems are normal for older women. d. The patient should stop regular exercise because that is probably causing these symptoms.

The patient's assessment points toward normal menopause. rationale: The most significant physiological changes during middle age are menopause in women and the climacteric in men. Menopause typically occurs between 45 and 60 years of age. The nurse should continue with the examination because a comprehensive assessment offers direction for health promotion recommendations, as well as for planning and implementing any acutely needed interventions. Exercise should not be stopped, especially in middle-aged adults.

The nurse is evaluating care of a patient for crutches. Which finding indicates a successful outcome? a. The top of the crutch is three to four finger widths from the armpit. b. The elbows are slightly flexed at 30 to 35 degrees when the patient is standing. c. The tip of the crutch is 4 to 6 inches anterior to the front of the patient's shoes. d. The position of the handgrips allows the axilla to support the patient's body weight.

The tip of the crutch is 4 to 6 inches anterior to the front of the patient's shoes. rationale: When crutches are fitted, the tip of the crutch is 4 to 6 inches anterior to the front of the patient's shoes, and the length of the crutch is two to three finger widths from the axilla. Position the handgrips so the axillae are not supporting the patient's body weight. Pressure on the axillae increases risk to underlying nerves, which sometimes results in partial paralysis of the arm. Determine correct position of the handgrips with the patient upright, supporting weight by the handgrips with the elbows slightly flexed at 20 to 25 degrees.

.A nurse is caring for a patient who recently had spinal surgery. The nurse knows that patients usually experience acute pain following this type of surgery. The patient refuses to get up and walk and is not moving around in the bed. However, the patient is stoic and denies experiencing pain at this time. What most likely explains this patient's behavior? a. The surgery successfully cured the patient's pain. b. The patient's culture is possibly influencing the patient's experience of pain. c. The primary health care provider did not prescribe the correct amount of medication. d. The nurse is allowing personal beliefs about pain to influence pain management at this time.

The patient's culture is possibly influencing the patient's experience of pain. rational: A patient's culture or beliefs about pain often influence the patient's expression of pain. In this case, the patient has just had surgery, and the nurse knows that this surgical procedure usually causes patients to experience pain. It is important at this time for the nurse to examine cultural and ethnic factors that are possibly affecting the patient's lack of expression of pain at this time. Even if surgery corrects neurological factors that create chronic pain, surgery causes pain in the acute period. The patient has not taken any pain medication so this is an unrealistic assumption; most pain medications have standard dosages. The nurse is not allowing personal beliefs to influence pain management because the nurse is attempting to determine the reason why the patient is not verbalizing the experience of pain.

The nurse is evaluating the effectiveness of guided imagery for pain management as used for a patient who has second- and third-degree burns and needs extensive dressing changes. Which finding best indicates the effectiveness of guided imagery? a. The patient's facial expressions are stoic during the procedure. b. The patient rates pain during the dressing change as a 6 on a scale of 0 to 10. c. The patient's need for analgesic medication decreases during the dressing changes. d. The patient asks for pain medication during the dressing changes only once throughout the procedure.

The patient's need for analgesic medication decreases during the dressing changes. rationale: If the patient needs less pain medication during dressing changes, then guided imagery is helping to manage the patient's pain. The purpose of guided imagery is to allow the patient to alter the perception of pain. Guided imagery works in conjunction with analgesic medications, potentiating their effects. A rating of 6 on a 0 to 10 scale indicates that the patient is having moderate pain and shows that this patient is not experiencing pain relief at this time. A person who is stoic is not showing feelings, which makes it difficult to know whether or not the patient is experiencing pain. Having to ask for pain medication during the dressing changes indicates the guided imagery is not effective.

The patient is brought to the emergency department with possible injury to the left shoulder. Which area will the nurse assess to best determine joint mobility? a. The patient's gait b. The patient's range of motion c. The patient's ethnic influences d. The patient's fine-motor coordination

The patient's range of motion rationale: Assessing range of motion is one assessment technique used to determine the degree of joint mobility and injury to a joint. Gait is the manner or style of walking. It has little bearing on the shoulder damage. Assessing fine-motor coordination would be beneficial in helping to assess the patient's ability to perform tasks such as feeding and dressing but would not help in evaluating the shoulder. Ethnic influences would not have a direct bearing on the amount of mobility in the joint.

The nurse is caring for a young-adult patient on the medical-surgical unit. When doing midnight checks, the nurse observes the patient awake, putting a puzzle together. Which information will the nurse consider to best explain this finding? a. The patient misses family and is lonely. b. The patient was waiting to talk with the nurse. c. The patient has been kept up with the noise on the unit. d. The patient's sleep-wake cycle preference is late evening

The patient's sleep-wake cycle preference is late evening rationale: his patient is awake and alert enough to do a puzzle. The individual's sleep-wake preference is probably late evening. All persons have biological clocks that synchronize their sleep-wake cycle. This explains why some individuals fall asleep in the early evening, whereas others go to bed at midnight or early morning. Waiting to talk with the nurse, being lonely, and noise on the unit may contribute to lack of sleep, but the best explanation for the patient being awake is the biological clock

A nurse is assessing body alignment. What is the nurse monitoring? a. The relationship of one body part to another while in diӄerent positions b. The coordinated eӄorts of the musculoskeletal and nervous systems c. The force that occurs in a direction to oppose movement d. The inability to move about freely

The relationship of one body part to another while in diӄerent positions rationale: The terms body alignment and posture are similar and refer to the positioning of the joints, tendons, ligaments, and muscles while standing, sitting, and lying. Body alignment means that the individual's center of gravity is stable. Body mechanics is a term used to describe the coordinated eӄorts of the musculoskeletal and nervous systems. Friction is a force that occurs in a direction to oppose movement. Immobility is the inability to move about freely.

The nurse is caring for a patient who has experienced a total abdominal hysterectomy. Which nursing observation of the incision will indicate the patient is experiencing a complication of wound healing? a. The site is hurting. b. The site is approximated. c. The site has started to itch. d. The site has a mass, bluish in color.

The site has a mass, bluish in color. rationale: A hematoma is a localized collection of blood underneath the tissues. It appears as swelling, change in color, sensation, or warmth or a mass that often takes on a bluish discoloration. A hematoma near a major artery or vein is dangerous because it can put pressure on the vein or artery and obstruct blood flow. Itching is not a complication. Incisions should be approximated with edges together; this is a sign of normal healing. After surgery, when nerves in the skin and tissues have been traumatized by the surgical procedure, it is expected that the patient will experience pain.

In discussing spiritual well-being, the nurse identifies that the vertical dimension involves a. The positive relationships and connections people have with others. b. The transcendent relationship between a person and God. c. Confidence in something for which there is no proof. d. Providing an attitude of something to live for and look forward to.

The transcendent relationship between a person and God. rational: The concept of spiritual well-being is often described as having two dimensions. The vertical dimension supports the transcendent relationship between a person and God or some other higher power. The horizontal dimension describes positive relationships and connections people have with others. Faith provides confidence in something for which there is no proof. When a person has the attitude of something to live for and look forward to, hope is present.

The patient is eager to begin an exercise program with a 2-mile jog. The nurse instructs the patient to warm up. The patient does not want to waste time with a "warm-up." Which information will the nurse share with the patient? a. The warm-up in this case can be done after the 2-mile jog. b. The warm-up prepares the body and decreases the potential for injury. c. The warm-up allows the body to readjust gradually to baseline functioning. d. The warm-up should be performed with high intensity to prepare for the coming challenge.

The warm-up prepares the body and decreases the potential for injury. rationale: The warm-up activity prepares the body for activity and decreases the potential for injury and should not be omitted. It usually lasts about 5 to 10 minutes and may include stretching, calisthenics, and/or aerobic activity performed at a lower intensity. The warm-up is before the exercise, while the cool-down period is after the exercise. The cool-down, not the warm-up, allows the body to readjust gradually to baseline functioning and provides an opportunity to combine movement such as stretching with relaxation-enhancing mind-body awareness. The warm-up should not be a high-intensity workout.

The nurse is caring for an older-adult patient with a diagnosis of urinary tract infection (UTI). Upon assessment the nurse ӄnds the patient confused and agitated. How will the nurse interpret these assessment findings? a. These are normal signs of aging. b. These are early signs of dementia. c. These are purely psychological in origin. d. These are common manifestation with UTIs.

These are common manifestation with UTIs. rationale: The primary symptom of compromised older patients with an acute urinary tract infection or fever is confusion. Acute confusion in older adults is not normal; a thorough nursing assessment is the priority. With the diagnosis of urinary tract infection, these are not early signs of dementia and they are not purely psychological.

A nurse is caring for a patient who has some immobility from noninflammatory joint degeneration. The nurse is teaching the patient about this process. Which information will the nurse include in the teaching session? a. This will affect synovial fluid. b. This will affect the body systemically. c. This involves mostly non-weight-bearing joints. d. This involves overgrowth of bone at the articular ends.

This involves overgrowth of bone at the articular ends rationale: Joint degeneration, which can occur with inflammatory and noninflammatory disease, is marked by changes in articular cartilage combined with overgrowth of bone at the articular ends. Degenerative changes commonly affect weight-bearing joints. Synovial fluid is normal in noninflammatory diseases. Inflammatory joint disease (e.g., arthritis) is characterized by inflammation or destruction of the synovial membrane and articular cartilage and by systemic signs of inflammation.

A patient asks the nurse what the term polypharmacy means. Which information should the nurse share with the patient? a. This is multiple side effects experienced when taking medications. b. This is many adverse drug effects reported to the pharmacy. c. This is the multiple risks of medication effects due to aging. d. This is concurrent use of many medications.

This is concurrent use of many medications. rationale: Polypharmacy refers to the concurrent use of many medications. It does not have anything to do with side effects, adverse drug effects, or risks of medication use due to aging.

A patient with an intravenous infusion requests a new gown after bathing. Which of the following actions is most appropriate? a. Disconnect the intravenous tubing, thread the end through the sleeve of the old gown and through the sleeve of the new gown, and reconnect. b. Thread the intravenous bag and tubing through the sleeve of the old gown and through the sleeve of the new gown without disconnecting. c. Inform the patient that a new gown is not an option while receiving an intravenous infusion in the hospital. d. Call the charge nurse for assistance because linen use is monitored and this is not a common procedure.

Thread the intravenous bag and tubing through the sleeve of the old gown and through the sleeve of the new gown without disconnecting rationale: Procedure-related accidents such as contamination of sterile items can occur in the health care setting. Keeping the intravenous tubing intact without breaks in the system is imperative to decrease the risk of infection while changing a patient's gown and satisfying the patient's request.

A nurse is documenting end-of-life care. Which information will the nurse include in the patient's electronic medical record? (Select all that apply.) a. Reason for the death b. Time and date of death c. How ethically the family grieved d. Location of body identification tags e. Time of body transfer and destination

Time and date of death Location of body identification tags Time of body transfer and destination rationale: Documentation of end-of-life care includes the following: time and date of death, location of body identification tags, time of body transfer and destination and personal articles left on and secured to the body. Reason for the death is not appropriate; this is a medical judgment and not a nursing judgment. How ethically the family grieved is judgmental and does not belong in the chart. We must remain open to the varying views and beliefs of grieving that are in contrast to our own in order to best support and care for our patients and their families.

A nurse is choosing an appropriate topic for a young-adult health fair. Which topic should the nurse include? a. Retirement b. Menopause c. Climacteric factors d. Unplanned pregnancies

Unplanned pregnancies rationale: Unplanned pregnancies are a continued source of stress that can result in adverse health outcomes for the mother (young adult), infant, and family. Retirement is an issue for middle-aged, not young adults. The onset of menopause and the climacteric affect the sexual health of the middle-aged adult, not the young adult.

The patient is unable to move self and needs to be pulled up in bed. What will the nurse do to make this procedure safe? a. Place the pillow under the patient's head and shoulders. b. Do by self if the bed is in the flat position. c. Place the side rails in the up position. d. Use a friction-reducing device

Use a friction-reducing device. rationale: This is not a one-person task. Helping a patient move up in bed without help from other co-workers or without the aid of an assistive device (e.g., friction-reducing pad) is not recommended and is not considered safe for the patient or the nurse. Remove the pillow from under head and shoulders and place it at the head of the bed to prevent striking the patient's head against the head of the bed. When pulling a patient up in bed, the bed should be ӄat to gain gravity assistance, and the side rails should be down.

A terminally ill patient is experiencing constipation secondary to pain medication. Which is the best method for the nurse to improve the patient's constipation problem? a. Contact the health care provider to discontinue pain medication. b. Administer enemas twice daily for 7 days. c. Massage the patient's abdomen. d. Use a laxative.

Use a laxative rationale: Opioid medication is known to slow peristalsis, which places the patient at high risk for constipation. Laxatives are indicated for opioid-induced constipation. Massaging the patient's abdomen may cause further discomfort. Discontinuing pain medication is inappropriate for a terminally ill patient. Enema administration twice a day is not the best step in the treatment of opioid-induced constipation.

A nurse is assessing the skin of an immobilized patient. What will the nurse do? a. Assess the skin every 4 hours. b. Limit the amount of fluid intake. c. Use a standardized tool such as the Braden Scale. d. Have special times for inspection so as to not interrupt routine care.

Use a standardized tool such as the Braden Scale. rationale: Consistently use a standardized tool, such as the Braden Scale. This identiӄes patients with a high risk for impaired skin integrity. Skin assessment can be as often as every hour. Limiting ӄuids can lead to dehydration, increasing skin breakdown. Observe the skin often during routine care.

The patient has been diagnosed with a spinal cord injury and needs to be repositioned using the logrolling technique. Which technique will the nurse use for logrolling? a. Use at least three people. b. Have the patient reach for the opposite side rail when turning. c. Move the top part of the patient's torso and then the bottom part. d. Do not use pillows after turning

Use at least three people. rationale: At least three to four people are needed to perform this skill safely. Have the patient cross the arms on the chest to prevent injury to the arms. Move the patient as one unit in a smooth, continuous motion on the count of three. Gently lean the patient as a unit back toward pillows for support.

The nurse is caring for a patient who has suffered a stroke and has residual mobility problems. The patient is at risk for skin impairment. Which initial actions should the nurse take to decrease this risk? a. Use gentle cleansers, and thoroughly dry the skin. b. Use therapeutic bed and mattress. c. Use absorbent pads and garments. d. Use products that hold moisture to the skin.

Use gentle cleansers, and thoroughly dry the skin. rationale: Use cleansers with nonionic surfactants that are gentle to the skin. After you clean the skin, make sure that it is completely dry. Absorbent pads and garments are controversial and should be considered only when other alternatives have been exhausted. Depending on the needs of the patient, a specialty bed may be needed, but again, this does not provide the initial defense for skin breakdown. Use only products that wick moisture away from the patient's skin.

The nurse is caring for a 4-year-old child who has pain. Which technique will the nurse use to best assess pain in this child? a. Use the FACES scale. b. Check to see what previous nurses have charted. c. Ask the parents if they think their child is in pain. d. Have the child rate the level of pain on a 0 to 10 pain scale.

Use the FACES scale. rational: The FACES scale assesses pain in children who are verbal. Because a 4-year-old is verbal, this is an appropriate scale to use with this child. Assessing pain intensity in children requires special techniques. Young children often have difficulty expressing their pain. Parents' statement of pain is not an effective way to assess pain in children because children's statements are the most important. The 0 to 10 pain scale is too difficult for a 4-yearold child to understand. Previous documentation by nurses will tell you what the child's pain has been but will not tell you the child's current pain intensity.

The patient is having a difficult time dealing with his AIDS diagnosis. He states, "It's not fair. I'm totally isolated from my family because of this. Even my father hates me for this. He won't even speak to me." The nurse needs to a. Assure the patient that his father will accept his situation soon. b. Use therapeutic communication to establish trust and caring. c. Point out that the patient has no control and that he has to face the consequences. d. Tell the patient, "If your father can't get over it, forget it. You have to move on."

Use therapeutic communication to establish trust and caring. rationale: The nurse needs to use therapeutic communication to establish trust and a caring presence because providing spiritual care requires caring, compassion, and respect. The nurse should not offer false hope. The nurse should help the patient maintain feelings of control. The nurse should encourage renewing relationships if possible and establishing connections with self, significant others, and God.

A nurse is caring for a postoperative mastectomy patient. Which action is a priority for increasing self-awareness? a. Solving problems for the patient before developing insight b. Using communication skills to clarify family and patient expectations c. Telling the patient that it will be fine because many others have survived d. Rotating nursing personnel in the patient's care, so the patient can talk to many people

Using communication skills to clarify family and patient expectations rational: Increase the patient's self-awareness by allowing him or her to openly explore thoughts and feelings. A priority nursing intervention is the expert use of therapeutic communication skills to clarify the expectations of a patient and family. Interventions designed to help a patient reach the goal of adapting to changes in self-concept or attaining a positive self-concept are based on the premise that the patient first develops insight and self-awareness concerning problems and stressors and then acts to solve the problems and cope with the stressors. Reassurance that a person will do fine dismisses any potential concerns the patient may have. Rotating nursing personnel does not allow time for the patient to build rapport with any one nurse.

The nurse is caring for a patient who is immobile and is at risk for skin impairment. The plan of care includes turning the patient. Which is the best method for repositioning the patient? a. Place the patient in a 30-degree supine position. b. Utilize a transfer device to lift the patient. c. Elevate the head of the bed 45 degrees. d. Slide the patient into the new position.

Utilize a transfer device to lift the patient. rationale: When repositioning the patient, obtain assistance and utilize a transfer device to lift rather than drag the patient. Sliding the patient into the new position will increase friction. The patient should be placed in a 30-degree lateral position, not a supine position. The head of the bed should be elevated less than 30 degrees to prevent pressure ulcer development from shearing forces.

A nurse is assessing a patient who started to have severe pain 3 days ago. When the nurse asks the patient to describe the pain, the patient states, "The pain feels like it is in my stomach. It is a burning pain, and it spreads out in a circle around the spot where it hurts the most." Which type of pain does the nurse document the patient is having at this time? a. Superficial pain b. Idiopathic pain c. Chronic pain d. Visceral pain

Visceral pain rational: Visceral pain arises from visceral organs, such as those from the gastrointestinal tract. Visceral pain is diffuse and radiates in several directions and can have a burning quality. Superficial pain has a short duration and is usually a sharp pain arising from the skin. Pain of an unknown cause is called idiopathic pain. Chronic pain lasts longer than 6 months

The nurse is completing a sleep assessment on a patient. Which tool will the nurse use to complete the assessment? a. Visual analog scale b. Cataplexy scale c. Polysomnogram d. RAS scale

Visual analog scale rationale: The visual analog scale is utilized for assessing sleep quality. Cataplexy, or sudden muscle weakness during intense emotions such as anger, sadness, or laughter, occurs at any time during the day; there is no cataplexy scale for sleep assessment. A polysomnogram involves the use of EEG, EMG, and EOG to monitor stages of sleep and wakefulness during nighttime sleep; this is used in a sleep laboratory study. Researchers believe that the ascending reticular activating system (RAS) located in the upper brainstem contains special cells that maintain alertness and wakefulness; however, there is no assessment tool called the RAS scale.

The nurse is caring for an older adult who presents to the clinic after a fall. The nurse reviews fall prevention in the home. Which of the following should the patient avoid? (Select all that apply.) a. Watering outdoor plants with a nozzle and hose b. Purchasing light bulbs with strength greater than 60 watts c. Missing yearly eye examinations d. Using bathtubs without safety strips e. Unsecured rugs throughout the home f. Walking to the mailbox in the summer

Watering outdoor plants with a nozzle and hose Missing yearly eye examinations Using bathtubs without safety strips Unsecured rugs throughout the home rationale: Unsecured rugs, using a hose to water plants, missing yearly eye examinations, and using tubs without safety strips are all items the patient should avoid to help in the prevention of falls in the home. Exercise is beneficial and increases strength, which helps with the prevention of falls. It is important that the home is well lit, so encourage the purchase of bulbs with strength of 60 watts or higher for the home.

The nurse is teaching a young-adult couple about promoting the health and psychosocial development of their 8-year-old child. Which information from the parent indicates a correct understanding of the teaching? a. "We will provide consistent, devoted relationships to meet needs." b. "We will limit choices and provide punishment for mistakes." c. "We will provide proper support for learning new skills." d. "We will instill a strong identity of who our child is."

We will provide proper support for learning new skills rationale: An 8-year-child would be in the industry versus inferiority stage of development. During this stage, the child needs to be praised (proper support) for accomplishments such as learning new skills. Developing a strong identity is part of the identity versus role confusion stage, usually occurring during puberty. During the autonomy versus shame and doubt stage, limiting choices and harsh punishment lead to feelings of shame and doubt. Providing consistent, devoted relationship to meet needs is usually a part of the trust versus mistrust stage.

The nurse knows that four categories of risk have been identified in the health care environment. Which of the following provides the best examples of those risks? a. Tile floors, cold food, scratchy linen, and noisy alarms b. Carpeted floors, ice machine empty, unlocked supply cabinet, and call light in reach c. Wet floors, pinching fingers in door, failure to use lift for patient, and alarms not functioning properly d. Dirty floors, hallways blocked, medication room locked, and alarms set

Wet floors, pinching fingers in door, failure to use lift for patient, and alarms not functioning properly rationale: The four categories are falls, patient-inherent accidents, procedure-related accidents, and equipment-related accidents. Wet floors contribute to falls, pinching finger in door is patient inherent, failure to use the lift is procedure related, and an alarm not functioning properly is equipment related. Tile floors and carpeted or dirty floors do not necessarily contribute to falls. Cold food, ice machine empty, and hallways blocked are not patientinherent issues in the hospital setting but are more of patient satisfaction or infection control issues or fire safety issues. Scratchy linen, unlocked supply cabinet, and medication room locked are not procedure-related accidents. These are patient satisfaction issues and control of supply issues, and are examples of actually following a procedure correctly. Noisy alarms, call light within reach, and alarms set are not equipment-related accidents but are patient satisfaction issues and examples of following a procedure correctly.

When caring for a middle-aged adult exhibiting maladaptive coping skills, the nurse is trying to determine the cause of the patient's behavior. Which information from a growth and development perspective should the nurse consider when planning care? a. Individuals have uniform patterns of growth and development. b. Culture usually has no effect on predictable patterns of growth and development. c. Health is promoted based on how many developmental failures a patient experiences. d. When individuals experience repeated developmental failures, inadequacies sometimes result.

When individuals experience repeated developmental failures, inadequacies sometimes result. rationale: If individuals have repeated development failures, inadequacies sometimes result should be considered. Developmental failures could manifest with ineffective coping skills. However, when an individual experiences successes, health is promoted. Patients have unique patterns of growth and development that are not uniform. Nurses must consider the influence of culture and context on growth and development.

A nurse is assessing the body alignment of a standing patient. Which ӄnding will the nurse report as normal? a. When observed laterally, the spinal curves align in a reversed "S" pattern. b. When observed posteriorly, the hips and shoulders form an "S" pattern. c. The arms should be crossed over the chest or in the lap. d. The feet should be close together with toes pointed out.

When observed laterally, the spinal curves align in a reversed "S" pattern. rationale: When the patient is observed laterally, the head is erect and the spinal curves are aligned in a reversed "S" pattern. When observed posteriorly, the shoulders and hips are straight and parallel. The arms hang comfortably at the sides. The feet are slightly apart to achieve a base of support, and the toes are pointed forward.

Which question will be most appropriate for a nurse to ask when assessing an adult patient for growth and developmental delays? a. "How many times per week do you exercise?" b. "Are you able to stand on one foot for 5 seconds?" c. "Would you please describe your usual activities during the day?" d. "How many hours a day do you spend watching television or sitting in front of a computer?"

Would you please describe your usual activities during the day? rationale: Asking the patient to describe usual daily activities will provide the nurse with useful information about the patient's current patterns. Understanding normal growth and development helps nurses predict, prevent, and detect deviations from patients' own expected patterns. How many hours are spent watching television or in front of a computer and how many times the patient exercises in a week would not provide the nurse with as much information about the patient's expected patterns when stated patterns are compared with expected patterns for the patient's age group to detect delays. The question about standing on one foot is for a child, not an adult.

A patient has had two family members die during the past 2 days. Which coping strategy is most appropriate for the nurse to suggest to the patient? a. Writing in a journal b. Drinking alcohol to go to sleep c. Exercising vigorously rather than sleeping d. Avoiding talking with friends and family members

Writing in a journal rationale: Coping strategies may be healthy and effective like talking, journaling, and sharing emotions with others. They may also be unhealthy and ineffective like increased use of alcohol, drugs, and violence. Although exercise is important for self-care, sleep is also important. Shutting oneself away from friends and family by not talking about the sadness is not effective; the patient should spend time with people who are nurturing.

The nurse will be most concerned about the risk of malnutrition for a patient with which sensory deficit? a. Xerostomia b. Dysequilibrium c. Diabetic retinopathy d. Peripheral neuropathy

Xerostomia rational: Xerostomia is a decrease in production of saliva; this decreases the ability and desire to eat and can lead to nutritional problems. The other options do not address taste- or nutrition-related concerns. Dysequilibrium is balance. Diabetic retinopathy affects vision. Peripheral neuropathy includes numbness and tingling of the affected areas and stumbling gait.

Family members gather in the emergency department after learning that a family member was involved in a motor vehicle accident. After learning of the family member's unexpected death, the surviving family members begin to cry and scream in despair. Which phase does the nurse determine the family is in according to the Attachment Theory? a. Numbing b. Reorganization c. Yearning and searching d. Disorganization and despair

Yearning and searching rationale: Yearning and searching characterize the second bereavement phase in the Attachment Theory. Emotional outbursts of tearful sobbing and acute distress are common in this phase. During the numbing phase, the family is protected from the full impact of the loss. During disorganization and despair, the reason why the loss occurred is constantly examined. Reorganization is the last stage of the Attachment Theory in which the person accepts the change and builds new relationships.

A nurse is planning care for young-adult patients. Which information should the nurse consider when planning care? a. Fertility issues do not occur in young adulthood. b. Young adults tend to suffer more from severe illness. c. Substance abuse is easy to observe in young-adult patients. d. Young adults are quite active but are at risk for illness in later years.

Young adults are quite active but are at risk for illness in later years rationale: Young adults are generally active and experience severe illnesses less frequently. However, their lifestyles may put them at risk for illnesses or disabilities during their middle or older-adult years. An estimated 10% to 15% of reproductive couples are infertile, and many are young adults. Substance abuse is not always diagnosable, particularly in its early stages.

A nurse is teaching a community group of school-aged parents about safety. The most important item to prioritize and explain is how to check the proper fit of a. a bicycle helmet. b. swimming goggles. c. soccer shin guards. d. baseball sliding shorts.

a bicycle helmet. rationale: Bicycle-related injuries are a major cause of death and disability among children. Proper fit of the helmet helps to decrease head injuries resulting from bicycle accidents. Goggles, shin guards, and sliding shorts are important sports safety equipment and should fit properly, but they do not protect from this leading cause of death.

A nurse is assessing a 17-year-old adolescent's cognitive development. Which behavior indicates the adolescent has reached formal operations? a. Uses play to understand surroundings b. Discusses the topic of justice in society c. Hits other students to deal with environmental change d. Questions where the ice is hiding when ice has melted in a drink

discusses the topic of justice in society rationale: Discussing the topic of justice demonstrates that the adolescent is concerned about issues that affect others besides self. In the formal operations period, as adolescents mature, their thinking moves to abstract and theoretical subjects. They have the capacity to reason with respect to possibilities. Hitting would be a common schema during the sensorimotor stage of development. Using play to learn about the environment is indicative of the preoperational stage. During the concrete operations stage (ages 6 to 12 years), children are able to coordinate two concrete perspectives in social and scientific thinking, such as understanding the difference between "hiding" and "melting."

A nurse is using Jean Piaget's developmental theory to focus on cognitive development. Which area will the nurse assess in this patient? a. Latency b. Formal operations c. Intimacy versus isolation d. The postconventional level

formal operations rationale: Jean Piaget's theory includes four stages in sequential order: sensorimotor, preoperational, concrete operations, and formal operations. Intimacy versus isolation is part of Erik Erikson's psychosocial theory of development. Latency is stage 4 of Freud's five-stage psychosexual theory of development. The postconventional level of reasoning is part of Kohlberg's theory of moral development.

The nurse is observing the way a patient walks. Which aspect is the nurse assessing? a. Activity tolerance b. Body alignment c. Range of motion d. Gait

gait rationale: Gait describes a particular manner or style of walking. Activity tolerance is the type and amount of exercise or work that a person is able to perform. Body alignment refers to the position of the joints, tendons, ligaments, and muscles while standing, sitting, and lying. Range of motion is the maximum amount of movement available at a joint in one of the three planes of the body: sagittal, frontal, or transverse.

A nurse is preparing a care plan for a patient who is immobile. Which psychosocial aspect will the nurse consider? a. Loss of bone mass b. Loss of strength c. Loss of weight d. Loss of hope

loss of hope rationale: Loss of hope is a psychosocial aspect. Patients with restricted mobility may have some depression. Depression is an aӄective disorder characterized by exaggerated feelings of sadness, melancholy, dejection, worthlessness, emptiness, and hopelessness out of proportion to reality. All the rest are physiological aspects: bone mass, strength, and weight.

A nurse is assessing the risk of intimate partner violence (IPV) for patients. Which population should the nurse focus on most for IVP? a. White males b. Pregnant females c. Middle-aged adults d. Nonsubstance abusers

pregnant females rationale: The greatest risk of violence occurs during the reproductive years. A pregnant woman has a 35.6% greater risk of being a victim of IPV than a nonpregnant woman. White males, middle-aged adults, and nonsubstance abusers are not as high risk as pregnant women.

The nurse is caring for a patient who was involved in an automobile accident 2 weeks ago. The patient sustained a head injury and is unconscious. Which priority element will the nurse consider when planning care to decrease the development of a decubitus ulcer? a. Resistance b. Pressure c. Weight d. Stress

pressure rationale: Pressure is the main element that causes pressure ulcers. Three pressure-related factors contribute to pressure ulcer development: pressure intensity, pressure duration, and tissue tolerance. When the intensity of the pressure exerted on the capillary exceeds 15 to 32 mm Hg, this occludes the vessel, causing ischemic injury to the tissues it normally feeds. High pressure over a short time and low pressure over a long time cause skin breakdown. Resistance, stress, and weight are not the priority causes of pressure ulcers.

The nurse is completing an assessment of the patient's skin's integrity. Which assessment is the priority? a. Pressure points b. Breath sounds c. Bowel sounds d. Pulse points

pressure points rationale: Observe pressure points such as bony prominences. The nurse continually assesses the skin for signs of ulcer development. Assessment for tissue pressure damage includes visual and tactile inspection of the skin. Assessment of pulses, breath sounds, and bowel sounds is part of a head-to-toe assessment and could influence the function of the body and ultimately skin integrity; however, this assessment is not a specific part or priority of a skin assessment.

A nurse is caring for a 4-year-old patient. Which object will the nurse allow the child to play with safely to foster cognitive development? a. The pump administering intravenous fluids b. A book to read alone in a quiet place c. The blood pressure cuff d. A baseball bat

the blood pressure cuff rationale: Children should be allowed to play with any equipment that is safe, like a blood pressure cuff. A 4-year-old child would be in the preoperational period of cognitive development. Children at this stage are still egocentric. Play is very important to foster cognitive development. The IV pump and bat are not safe pieces of equipment for a 4-year-old child to play with. A 4-year-old child is of preschool age and more than likely is not able to read yet. Also, the book does not allow for any human interaction and communication if he or she reads alone.


संबंधित स्टडी सेट्स

Western Civ Marxism and WWI Test

View Set

Intro to Business Chapter 6 Review

View Set

Driving Segment 1 Chapter 3 (quiz)

View Set

Finance final quizzes (13,14,16,22,26)

View Set